Download as pdf or txt
Download as pdf or txt
You are on page 1of 217

Mc lc

1 Dy s v cc bi ton v dy s
1.1 Gii thiu . . . . . . . . . . . . . . . . . . . . . . . . . . . . . . . .
1.2 nh ngha v cc nh l c bn . . . . . . . . . . . . . . . . . . .
1.3 Mt s phng php gii bi ton v dy s . . . . . . . . . . . . .
1.3.1 Dy s thc: mt s dng dy s c bit . . . . . . . . . .
1.3.2 Dy s nguyn . . . . . . . . . . . . . . . . . . . . . . . . .
1.3.3 Dy s v phng trnh . . . . . . . . . . . . . . . . . . . .
1.3.4 Mt vi th thut khc . . . . . . . . . . . . . . . . . . . .
1.4 Mt s phng php xy dng h thng bi tp . . . . . . . . . . .
1.4.1 Xy dng dy hi t bng phng trnh . . . . . . . . . . .
1.4.2 Xy dng dy truy hi t cp nghim ca phng trnh bc 2
1.4.3 Xy dng cc dy s nguyn t li gii cc phng trnh
nghim nguyn . . . . . . . . . . . . . . . . . . . . . . . . .
1.4.4 Xy dng dy s l nghim ca mt h phng trnh ph
thuc bin n . . . . . . . . . . . . . . . . . . . . . . . . . .
1.5 L thuyt dy s di con mt ton cao cp . . . . . . . . . . . . .
1.5.1 Ri rc ha cc khi nim v nh l ca l thuyt hm
bin s thc . . . . . . . . . . . . . . . . . . . . . . . . . . .
1.5.2 Phng php hm sinh v bi ton tm s hng tng qut .
1.5.3 i s tuyn tnh v phng trnh sai phn . . . . . . . . .
1.5.4 S dng xp x trong d on kt qu . . . . . . . . . . . .
1.6 Bi tp . . . . . . . . . . . . . . . . . . . . . . . . . . . . . . . . .

4
4
5
8
8
12
17
18
23
23
24

2 Phng trnh sai phn


2.1 Sai phn . . . . . . . . . . . . . . . . . . . . . . . . . . . .
2.1.1 nh ngha . . . . . . . . . . . . . . . . . . . . . .
2.1.2 Tnh cht . . . . . . . . . . . . . . . . . . . . . . .
2.2 Phng trnh sai phn tuyn tnh . . . . . . . . . . . . . .
2.2.1 Mt s khi nim chung v phng trnh sai phn
2.3 Phng trnh sai phn tuyn tnh bc nht . . . . . . . .

41
41
41
41
43
43
44

.
.
.
.
.
.

.
.
.
.
.
.

.
.
.
.
.
.

.
.
.
.
.
.

.
.
.
.
.
.

25
26
27
27
29
30
31
32

MC LC
2.3.1
2.3.2
2.3.3

nh ngha . . . . . . . . . . . . . . . . . . . . . . . . . .
Phng php gii . . . . . . . . . . . . . . . . . . . . . . .
Phng php tm nghim ring ca phng trnh sai phn
tuyn tnh cp 1 khng thun nht khi v phi f (n) c
dng c bit . . . . . . . . . . . . . . . . . . . . . . . . .
2.3.4 Bi tp . . . . . . . . . . . . . . . . . . . . . . . . . . . .
2.4 Phng trnh sai phn tuyn tnh cp 2 . . . . . . . . . . . . . .
2.4.1 nh ngha . . . . . . . . . . . . . . . . . . . . . . . . . .
2.4.2 Cch gii . . . . . . . . . . . . . . . . . . . . . . . . . . .
2.5 Phng trnh sai phn tuyn tnh cp 3 . . . . . . . . . . . . . .
2.5.1 nh ngha . . . . . . . . . . . . . . . . . . . . . . . . . .
2.5.2 Phng php gii . . . . . . . . . . . . . . . . . . . . . . .
2.5.3 V d . . . . . . . . . . . . . . . . . . . . . . . . . . . . .
2.5.4 Phng trnh sai phn tuyn tnh cp k . . . . . . . . . .
3 Xc nh s hng tng qut ca mt dy s
3.1 Tm s hng tng qut ca dy (dng a thc) khi bit cc s
hng u tin . . . . . . . . . . . . . . . . . . . . . . . . . . . . .
3.2 Cng thc truy hi l mt biu thc tuyn tnh . . . . . . . . . .
3.2.1 V d . . . . . . . . . . . . . . . . . . . . . . . . . . . . .
3.3 Cng thc truy hi l mt h biu thc tuyn tnh . . . . . . . .
3.3.1 V d . . . . . . . . . . . . . . . . . . . . . . . . . . . . .
3.4 Cng thc truy hi l biu thc tuyn tnh vi h s bin thin .
3.5 Cng thc truy hi dng phn tuyn tnh vi h s hng . . . . .
3.6 H thc truy hi phi tuyn . . . . . . . . . . . . . . . . . . . . .
3.6.1 Quy trnh tuyn tnh ho mt phng trnh sai phn . . .
3.6.2 V d . . . . . . . . . . . . . . . . . . . . . . . . . . . . .
3.6.3 Mt s v d khc . . . . . . . . . . . . . . . . . . . . . .
3.6.4 Bi tp. . . . . . . . . . . . . . . . . . . . . . . . . . . . .
4 Phng trnh hm sai phn bc hai
4.1 Hm tun hon v phn tun hon cng tnh . . . . . . . . . . .
4.2 Phng trnh hm sai phn bc hai vi hm tun hon v phn
tun hon . . . . . . . . . . . . . . . . . . . . . . . . . . . . . . .
4.3 Phng trnh vi hm s tun hon, phn tun hon nhn tnh .
4.3.1 nh ngha . . . . . . . . . . . . . . . . . . . . . . . . . .
4.3.2 Mt s bi ton . . . . . . . . . . . . . . . . . . . . . . . .
4.3.3 Mt s v d p dng . . . . . . . . . . . . . . . . . . . .

2
. 44
. 44

.
.
.
.
.
.
.
.
.
.

45
47
47
47
48
55
55
55
56
58
60

.
.
.
.
.
.
.
.
.
.
.
.

61
63
64
70
70
72
78
81
82
83
87
96

99
. 99
.
.
.
.
.

100
108
109
109
125

MC LC
5 Dy s sinh bi hm s
5.1 Hm s chuyn i php tnh s hc v i s . .
5.2 V cc dy s xc nh bi dy cc phng trnh
5.3 nh l v ba mnh tng ng . . . . . . .
5.4 Mt s bi ton v c lng tng v tch . . . .
5.5 Bi tp . . . . . . . . . . . . . . . . . . . . . . .

.
.
.
.
.

.
.
.
.
.

.
.
.
.
.

.
.
.
.
.

.
.
.
.
.

.
.
.
.
.

.
.
.
.
.

.
.
.
.
.

.
.
.
.
.

128
128
135
141
142
144

6 Mt s lp hm chuyn i cc cp s
6.1 Cp s cng, cp s nhn v cp s iu ho . . . .
6.2 Dy s tun hon . . . . . . . . . . . . . . . . . . . .
6.3 Hm s chuyn i cp s cng . . . . . . . . . . . .
6.4 Hm s chuyn i cp s cng vo cp s nhn . . .
6.5 Hm s chuyn i cp s nhn vo cp s cng . . .
6.6 Hm s chuyn i cp s nhn vo cp s iu ho

.
.
.
.
.
.

.
.
.
.
.
.

.
.
.
.
.
.

.
.
.
.
.
.

.
.
.
.
.
.

.
.
.
.
.
.

.
.
.
.
.
.

.
.
.
.
.
.

145
145
146
152
154
155
156

.
.
.
.
.

7 Mt s lp hm chuyn i cc cp s trong tp ri rc
158
7.1 Hm s chuyn i cp s cng thnh cp s cng . . . . . . . . . 158
7.2 Hm s chuyn i cp s nhn thnh cp s nhn . . . . . . . . . 161
8 Mt s bi ton xc nh dy s trong lp dy tun hon cng
tnh v nhn tnh.
167
8.1 Mt s bi ton xc nh dy s trong lp dy tun hon cng tnh 167
8.2 Hm s xc nh trn tp cc s nguyn . . . . . . . . . . . . . . . 170
8.2.1 Hm s chuyn i cc php tnh s hc . . . . . . . . . . 170
8.2.2 Hm s chuyn tip cc i lng trung bnh . . . . . . . . 172
8.2.3 Phng trnh trong hm s vi cp bin t do . . . . . . . 177
8.2.4 Mt s dng ton lin quan n dy truy hi . . . . . . . . 180
8.3 Hm s xc nh trn tp cc s hu t . . . . . . . . . . . . . . . 184
8.4 Phng trnh trong hm s vi cp bin t do . . . . . . . . . . . . 191
8.5 S dng gii hn gii phng trnh hm . . . . . . . . . . . . . 198
Ti liu tham kho . . . . . . . . . . . . . . . . . . . . . . . . . . . . . 217

Chng 1

Dy s v cc bi ton v dy
s
1.1

Gii thiu

Chn ti v dy s, chng ti t trc mnh mt nhim v v cng kh


khn, bi y l mt lnh vc rt kh v rt rng, s dng nhiu kin thc khc
nhau ca ton hc. Hn th, trc c kh nhiu cun sch chuyn kho v
ti ny. D vy, chng ti vn mun c gng ng gp mt s kinh nghim v
ghi nhn ca mnh thu lm c trong qu trnh ging dy nhng nm qua.
Tp ti liu ny khng phi l mt gio trnh v dy s, li cng khng phi
l mt cm nang hng dn gii cc bi ton dy s. Tp ti liu ny ng hn
ht l nhng cp nht ca tc gi v nhng phng php gii cc bi ton dy
s cng vi nhng nhn nh i khi mang y tnh ch quan ca tc gi. V vy,
hy coi y l mt ti liu m. Hy tip tc trin khai, lin h v c kt kinh
nghim, ghi nhn nhng ci hay v gp cho nhng ci cha hay, thm ch cha
chnh xc.
Trong ti liu ny, khng phi tt c cc vn ca dy s u c cp
ti. V d phn dy s v bt ng thc ch c ni n rt s si, cc bi ton
dy s m thc cht l cc bi ton v ng d cng khng c xt ti... Hai
mng ln m tp ti liu ny ch n nht l bi ton tm s hng tng qut
ca mt dy s v bi ton tm gii hn dy s.
Trong tp ti liu ny, cc vn v cc bi ton c mc kh d khc
nhau. C nhng bi c bn, c nhng bi kh hn v c nhng bi rt kh. V
vy, cn phi la chn vn vi mc thch hp (v d c mt s vn v
bi ton ch ng phi mc k thi chn i tuyn hoc quc t).
Vit tp ti liu ny, tc gi s dng rt nhiu ngun ti liu khc nhau,
tuy nhin ch c mt s bi c ghi ngun gc, mt s bi khng th xc nh c.

1.2. nh ngha v cc nh l c bn

Tc gi cng s dng cc bi ging ca cc thy Phan c Chnh, Nguyn


Vn Mu, L nh Thnh, ng Hng Thng, Nguyn Minh c... trong bi vit
ca mnh.
Cui cng, tp ti liu ny khng khi c nhng nhm ln v thiu st, tc
gi rt mong nhn c s gp ca tt c cc thy c gio. V rt mong rng,
vi n lc chung ca tt c chng ta, tp ti liu s tip tc c hon thin v
b sung.

1.2

nh ngha v cc nh l c bn

nh ngha 1.1. Dy s l mt hm s t N vo mt tp hp s (N, Q, R, C)


hay mt tp con no ca cc tp hp trn). Cc s hng ca dy s thng
c k hiu l un , vn, xn , yn thay v u(n), v(n), x(n), v(n). Bn thn dy s c
k hiu l {xn }.
V dy s l mt trng hp c bit ca hm s nn n cng c cc tnh
cht ca mt hm s.
nh ngha 1.2. Dy s {xn } c gi l dy tng (gim) nu vi mi n ta c
xn+1 xn (xn+1 xn ). Dy s tng hoc dy s gim c gi chung l dy n
iu.
Dy s {xn} c gi l b chn trn nu tn ti s thc M sao cho vi mi n
ta c xn M .
Dy s {xn } c gi l b chn di nu tn ti s thc m sao cho vi mi n
ta c xn m.
Mt dy s va b chn trn, va b chn di c gi l dy b chn.
Dy s xn c gi l tun hon vi chu k k nu xn+k = xn vi mi n N. Dy
s tun hon vi chu k 1 gi l dy hng.
nh ngha 1.3. Ta ni dy s {xn } c gii hn hu hn a khi n dn n v
cng nu vi mi  > 0, tn ti s t nhin N0 (ph thuc vo dy s xn v )
sao cho vi mi n > N0 ta c |xn a| nh hn .
lim xn = a  > 0N0 N : n > N0|xn a| < 

Ta ni dy s {xn } dn n v cng khi n dn n v cng nu vi mi s


thc dng M ln tu , tn ti s t nhin N0 (ph thuc vo dy s xn v M )
sao cho vi mi n > N0 ta c |xn | ln hn M .
lim xn = M > 0N0 N : n > N0 |x| > M.

Dy s c gii hn hu hn c gi l dy hi t. Dy s khng c gii hn


hoc dn n v cng khi n dn n v cng gi l dy phn k.

1.2. nh ngha v cc nh l c bn

nh l 1.1 (Tng, hiu, tch, thng cc dy hi t). Nu {xn }, {yn } l cc


dy hi t v c gii hn tng ng l a, b th cc dy s {xn + yn }, {xn yn },
{xn yn } v {xn /yn } cng hi t v c gii hn tng ng l a + b, a b, a.b, a/b.
(Trong trng hp dy s thng, ta gi s yn v b khc khng)
nh l 1.2 (Chuyn qua gii hn trong bt ng thc). Cho dy s {xn } c
gii hn hu hn l, nu N0 N : n > N0 ta c a xn b th a xn b.
nh l 1.3 (nh l kp). Cho ba dy s {xn }, {yn}, {zn} trong xn v zn c
cng gii hn hu hn 1, v N0 N : n > N0 ta c xn yn zn . Khi yn
cng c gii hn l 1.
nh l 1.4 (Dy n iu). Mt dy tng v b chn trn hay mt dy gim
v b chn di th hi t. Ni ngn gn hn, mt dy s n iu v b chn th
hi t.
nh l 1.5 (V dy cc on thng lng nhau). Cho hai dy s thc {an }, {bn}
sao cho
a) n N, an bn ;
b) nN, [an+1 , bn+1] [an , bn];
c) bn an 0 khi n .
Khi tn ti duy nht s thc l sao cho [an , bn] = 1.
nh l 1.6 (Bolzano Veierstrass). T mt dy b chn lun c th trch ra mt
dy con hi t.
nh ngha 1.4. Dy {xn } c gi l dy Cauchy nu  > 0N0 N: m, n >
N0 |xm xn | < .
nh ngha 1.5 (Tiu chun Cauchy). Dy s {xn } c gii hn hu hn khi v
ch khi n l dy Cauchy.
Cp s cng. Dy s {xn } c gi l mt cp s cng khi v ch khi tn
ti d sao cho
n N, xn+1 = xn + d.
d c gi l cng sai ca cp s cng, x0 l s hng u, xn l s hng th n.
Ta c cc cng thc c bn sau:
xn = x0 + nd
Sn = x0 + x1 + + xn1
= nx0 + n(n 1)d/2
= n(x0 + xn1 )/2

1.2. nh ngha v cc nh l c bn

Cp s nhn. Dy s {xn } c gi l mt cp s nhn khi v ch khi tn ti


q sao cho
n N, xn+1 = qxn .
d c gi l cng bi ca cp s nhn, x0 l s hng u, xn l s hng th n.
Ta c cc cng thc c bn sau:
x n = q n x0
Sn = x0 + x1 + + xn1 = (q n 1)x0/(q 1)
Nu |q| < 1 th {xn } c gi l cp s nhn li v hn. Tng ca cp s nhn
li v hn c tnh theo cng thc
S = x0 /(1 q)
Dy Fibonacci. Dy s Fibonacci l dy s c nh ngha bi
f0 = 0, f1 = 1, n N, fn+2 = fn+1 + fn .
Dy s Fibonacci c rt nhiu tnh cht th v v xut hin mt cch t nhin
trong nhiu lnh vc khc nhau. Chng ta c cng thc sau y tm s hng
tng qut ca dy s Fibonacci:
Cng thc Binet.
 n   n
1+ 5
12 5
2

fn =
.
5
Ni chung, cc dy s xc nh bi cng thc truy hi fn+2 = fn+1 + fn (vi
f0 , f1 bt k) c gi l dy Fibonacci m rng.
Dy Farey. Dy Farey Fn vi mi s nguyn dng n l tp hp cc phn s
ti gin dng a/b vi 0 a b n v (a, b) = 1 xp theo th t tng dn.
V d 1.1.
F5 = {0/1, 1/5, 1/4, 1/3, 2/5, 1/2, 3/5, 2/3, 3/4, 4/5, 1/1}.
Ngoi tr F1 , Fn c s l cc phn t v 1/2 lun nm gia. Gi p/q, p0 /q 0 v
p00/q 00 l cc s hng lin tip trong dy Farey th
pq 0 qp0 = 1, v p0 /q 0 = (p + p00)/(q + q 00).
S cc s hng N (n) trong dy Farey c tnh theo cng thc
N (n) = 1 +

n
X
k=1

(k) = 1 + (n).

1.3. Mt s phng php gii bi ton v dy s

1.3

Mt s phng php gii bi ton v dy s

Phng php gii cc bi ton dy s rt a dng nh chnh yu cu ca


chng. c th l mt tnh cht s hc, mt tnh cht i s hay mt tnh cht
gii tch. Di y chng ta s xem xt nhng phng php c bn nht.
Tuy nhin, c th a ra hai nguyn l chung gii cc bi ton dy s l
- ng ngi vit ra cc s hng u tin ca dy s
- ng ngi tng qut ha bi ton

1.3.1

Dy s thc: mt s dng dy s c bit

Dy s dng xn+1 = f (xn )


y l dng dy s thng gp nht trong cc bi ton v gii hn dy s.
Dy s ny s hon ton xc nh khi bit f v gi tr ban u x0 . Do vy s hi
t ca dy s s ph thuc vo tnh cht ca hm s f (x) v x0 . Mt c im
quan trng khc ca dy s dng ny l nu a l gii hn ca dy s th a phi l
nghim ca phng trnh x = f (x). Chng ta c mt s kt qu c bn nh sau:
nh ngha 1.6. Hm s f : D D c gi l mt hm s co trn D nu tn
ti s thc q, 0 < q < 1 sao cho |f (x) f (y)| q|x y| vi mi x, y thuc D.
nh l 1.7. Nu f (x) l mt hm s co trn D th dy s {xn } xc nh bi
x0 = a D, xn+1 = f (xn ) hi t. Gii hn ca dy s l nghim duy nht trn
D ca phng trnh x = f (x).
Chng minh. Vi mi n > m th p dng nh ngha hm s co, ta c
|xn xm | = |f (xn1 ) f (xm1 )| q|xn1 xm1 | qm |xnm x0 | (1.1)
T y |xn x0 | |xn xn1 | + + |x1 x0 | (qn1 + + 1)|x1 x0|, suy
ra {xn } b chn. Xt  > 0. T (1.1), do q < 1 v |xnm x0| b chn nn ta suy
ra tn ti N sao cho q N |xnm x0| < . Suy ra {xn } l dy Cauchy v do hi
t.
V d 1.2 (Vit Nam, 2000). Cho dy s {xn } xc nh nh sau
q

x0 = 0, xn+1 = c c + xn .
Tm tt c cc gi tr ca c vi mi gi tr x0 (0, c), xn xc nh vi mi n
v tn ti gii hn hu hn limn xn .

1.3. Mt s phng php gii bi ton v dy s

Gii. x1 tn ti th ta th c c + xn 0 vi mi x0 (0, c) hay c(c1) x0

vi mi x0 (0, c), suy ra c 2. Vi c 2 th 0 < x1 < c. Nu 0 < xn < c

th c c + xn p
> c 2 c, suy ra xn+1 tn ti v ta
pcngc 0 < xn+1 < c.

t f (x) = c c + x th f 0 (x) = 14 x + x c c + x.
p

Vi
mi
x

(0,
c)
ta
c
(c
+
x)(c

c
+
x)
>
c(c

c + c) 2(2
p

2 + 2) > 14 . T suy ra |f 0 (x)| q < 1 vi mi x (0, c), tc f (x) l

hm s co trn (0, c), suy ra dy s cho hi t. Vy tt c cc gi tr c cn


tm l c 2.
Mt trng hp na cng c th xt c s hi t ca dy s {xn } l trng
hp f n iu. C th l
Nu f l hm s tng trn D th {xn } s l dy n iu. Dy s ny tng
hay gim tu theo v tr ca x0 so vi x1 .
Nu f l hm gim trn D th cc dy con {x2p}, {x2p+1} l cc dy n iu
(v ngc chiu nhau).
V d 1.3 (V ch sinh vin Moskva, 1982). Cho dy s {xn } xc nh bi
x0 = 1982, xn+1 = 1/(4 3xn ). Hy tm limn xn
Gii. Tnh ton trc tip ta thy 0 < x2 < 1, x3 > x2 . V f (x) = 1/(4 3x) l
mt hm s tng t [0, 1] vo [0, 1] nn t y, {xn }n2 l mt dy s tng v b
chn trn bi 1 do c gii hn. Gi s gii hn l a th ta c a = 1/(4 3a)
hay a = 1 (gi tr a = 1/3 loi do dy tng).
Cu hi: Vi nhng gi tr no ca x0 th dy s xc nh vi mi x v c gii
hn? Khi no th gii hn l 1? Khi no th gii hn l 1/3?
Trong trng hp f l hm gim, ta c th chng minh dy hi t bng cch
chng minh hai dy con trn cng hi t v mt gii hn.
Tuy nhin, kh khn nht l gp cc hm s khng n iu. Trong trng
hp ny, ta phi xt tng khong n iu ca n v s hi t ca hm s s ty
thuc vo gi tr ban u.
V d 1.4. Tm tt c cc gi tr ca a dy s {xn } xc nh bi x0 = a, xn+1 =
2 x2n c gii hn hu hn.
Gii. Hm s f (x) = 2 x2 tng trn (, 0) v gim trn (0, +). Phng
trnh f (x) = x c hai nghim l x = 2 v x = 1. l nhng d kin quan
trng trong li gii bi ton ny.
u tin, ta nhn xt rng nu a < 2 th do f : (, 2) (, 2) v
l hm tng, x1 = 2 a2 < x0 nn dy s {xn } gim. Nu dy {xn } b chn di
th n hi t v nghim ca phng trnh x = 2 x2 , iu ny mu thun v dy
gim v x0 < 2. Vy {xn } khng b chn di, tc khng c gii hn hu hn.
Nu a > 2 th x1 < 2 v ta cng suy {xn } khng c gii hn hu hn.

1.3. Mt s phng php gii bi ton v dy s

10

Vi a = 2, 1 th dy s c gii hn. Xt x0 [2, 2]. Ta chng minh dy s


c gii hn hu hn khi v ch khi tn ti n sao cho xn = 2 hoc xn = 1. Tht
vy, gi s xn c gii hn hu hn l b v xn
/ {2, 1} vi mi n. Khi b = 2
hoc b = 1. Gi s b = 2 th tn ti N0 sao cho xn nm trong ln cn 2 vi
mi n N0. Nhng nu xn = 2 +  th xn+1 = 2 + 4 2 > xn , suy ra dy xn
tng k t N0 v khng th dn v 2. Nu b = 1 k t n N0 no xn thuc
ln cn 1. Xt
xn+2 xn = 2 (2 x2n )2 xn = (2 xn x2n )(x2n xn 1)
Ti ln cn 1 th x2n xn 1 < 0. V nu xn < 1 th xn+1 > 1 (v ngc li
xn > 1 th xn+1 < 1 - chng ta ang xt trong ln cn im 1!) nn c th gi
s xn > 1. Khi 2 xn x2n < 0 suy ra xn+2 > xn . Tip tc nh vy, suy ra
1 < xn < xn+2 < < xn+2k < mu thun vi gi thit b = 1. Vy iu gi
s l 2, tc l dy s ch c gii hn khi tn ti n sao cho xn = 2 hoc
xn = 1.
1
Sau khi thu c kt qu ny, ta s dng hm ngc f (x) = 2 x
xy dng tt c cc gi tr a tha mn iu kin u bi.
Trong v d trn, ta s dng gi thit tn ti gii hn thu gn min D,
t mt hm c bin thin phc tp tr thnh mt hm n iu.
Dy s dng xn+1 = xn (xn ) v nh l trung bnh Cesaro
y l trng hp c bit ca dy s dng xn+1 = f (xn ). Tuy nhin, vi
dy s dng ny vn hi t ca xn thng khng c t ra (v qu n gin
v gii hn ch c th l 0 hoc ). y, ta s c mt yu cu cao hn l tm
bc tim cn ca xn , c th l tm b sao cho xn = O(n ). Vi cc dy s c dng
ny, nh l trung bnh Cesaro s t ra rt hu hiu.
nh l 1.8 (Trung bnh Cesaro). Nu dy s {xn } c gii hn hu hn l a
th dy s cc trung bnh {x1 + x2 + + xn )/n} cng c gii hn l a.
nh l ny c th pht biu di dng tng ng nh sau: Nu lim n (xn+1
xn ) = a th limn xn /n = a.
Ta chng minh nh l cch pht biu 2. R rng ch cn chng minh cho
trng hp a = 0. V limn (xn+1 xn ) = 0 nn vi mi  > 0 tn ti, N0 sao
cho vi mi n N0 ta c |xn+1 xn | < . Khi , vi mi n > N0
|xn /n| [|xN0 | + |xN0 +1 xN 0 | + + |xn xn1 |]/n < |xN0 |/n + (n N0)/n.
Gi c nh N0, ta c th tm c N1 > N0 sao cho |xN 0|/N1 < . Khi vi
mi n > N1 ta s c |xn /n| < 2. Vy limn xn /n = 0.
nh l trung bnh Cesaro c nhiu ng dng quan trng trong vic tm gii
hn dy s v c th pht biu cho cc trung bnh khc nh trung bnh nhn,

11

1.3. Mt s phng php gii bi ton v dy s

trung bnh iu ha, trung bnh ly tha. Tuy nhin, y ta ch khai thc cch
pht biu 2 ca nh l p dng cho cc dy s c dng xn+1 = xn (xn ) .
tm s sao cho xn /n c gii hn hu hn, theo nh l trung bnh Cesaro, ta
ch cn tm g sao cho xn+1 xn c gii hn hu hn a. Khi , limn xn /n = a,
suy ra lim xn /n1 = a1 , tc l = 1/.
V d 1.5. Cho dy s {xn } c xc nh bi x0 = 1/2, xn+1 = xn x2n . Chng
minh rng limn nxn = 1.
Gii. Trong bi ny, = 1 do ta s th vi = 1. D dng chng minh
c limn xn = 0. Ta c
1/xn+1 1/xn = (xn xn+1 )/xn+1 xn = x2n /(xn x2n )xn = 1/(1 xn ) 1.
T p dng nh l trung bnh Cesaro, suy ra lim1/nxn = 1, suy ta lim nxn =
1.
V d 1.6. Cho dy s{xn } c xc nh bi x0 = 1, xn+1 = sin(xn ). Chng

minh rng lim nxn = 3.


Gii. Dy s cho khng c dng xn+1 = xn (xn ) (?) nhng kt lun ca
bi ton gi cho chng ta n nh l trung bnh Cesaro. V = 1 nn ta s
th vi = 2. D dng chng minh c rng lim xn = 0. Xt
1/x2n 1/x2n = [x2n sin2 (xn )]/x2n sin2 (xn ) 1/3
(Dng quy tc LHopitale)

2
T , theo nh l trung bnh Cesaro lim 1/nxn = 1/3, suy ra lim lim n.xn =
3.
Nh vy, ta c th tm nu bit . Trong trng hp khng bit th ta
phi d on.
V d 1.7 (Chn i tuyn Vit Nam, 1993). Dy s {an } c xc nh bi

a1 = 1 v an+1 = an + 1/ an . Hy tm tt c cc s thc dy s (an ) /n


c gii hn hu hn khc 0.
Gii. Trc ht ta chng minh an dn ti v cng khi n dn ti v cng. Tht

vy, ta c a2n+1 = a2n + 2 an + 1/an > a2n + 2. Suy ra a2n+1 > 1 + 2n suy ra
(pcm). Tr li bi ton, xt
3/2
3/2
3/2 3/2
an+1 a3/2
a3/2
/(1/a3/2
n = (an + 1/ an )
n = (1 + 1/an )
n )
3/2

t x = 1/an

3/2

3/2

th x 0 khi n . Do limn (an+1 an ) =


3/2

limx0(1 + x)3/2/x = 3/2 (Quy tc LHopitale) T suy ra lim an /n = 3/2.

1.3. Mt s phng php gii bi ton v dy s

12

Vi > 3/2 suy ra gii hn bng , vi < 3/2 suy ra gii hn bng 0. Vy
= 3/2 l gi tr duy nht tho mn yu cu bi ton.
Cu hi:
1) Lm sao c th d on c gi tr ?
2) v c mi quan h g?

1.3.2

Dy s nguyn

Dy s nguyn l mt phn quan trng trong l thuyt dy s. Ngoi cc vn


chung nh tm s hng tng qut ca dy s, tm cng thc tnh tng n s
hng u tin... cc bi ton v dy s nguyn cn quan tm n tnh cht s
hc ca dy s nh chia ht, ng d, nguyn t, chnh phng, nguyn t cng
nhau... Cc bi ton v dy s nguyn rt a dng. Trong nhiu trng hp, dy
s ch l ci b ngoi, cn bn cht bi ton l mt bi ton s hc. Trong cc
phn di y, chng ta s t cp n nhng bi ton nh vy m chuyn
chng vo phn bi tp.
Nguyn l Dirichlet v dy s nguyn
Nguyn l Dirichlet l mt nguyn l ht sc n gin nhng li v cng hu
hiu trong cc bi ton chng minh, c bit l chng minh s tn ti ca mt
i tng tho mn mt iu kin no . S dng nguyn l ny, ngi ta
chng minh c nhiu kt qu rt mnh, v d nh nh l Fermat-Euler v
tng hai bnh phng, nh l Weil v phn b u... y ta nu ra hai kt
qu lin quan n dy s:
nh l 1.9 (Weil, v phn b u). Nu l s v t th dy {n}n=1 phn
b u trn khong (0, 1).
nh l 1.10 (V s tun hon ca cc s d). Cho dy s nguyn {xn }
xc nh bi cng thc truy hi xn+k = a1 xn+k1 + + ak xn v k s hng u
tin nguyn. Khi , vi mi s nguyn dng N , dy s d ca xn khi chia cho
N s tun hon.
Tip theo ta xt mt vi v d v vic s dng nguyn l Dirichlet trong cc
bi ton dy s.
V d 1.8. Chng minh rng nu 1 a1 , a2, ..., an+1 2n th tn ti i < j sao
cho ai | aj .
Gii. Mi s ai c th vit di dng ai = 2si ri vi ri l s l. Cc s ri ch c
th nhn n gi tr t 1, 3, ..., 2n 1. V c n + 1 s nn theo nguyn l Dirichlet,
tn ti i < j sao cho ri = rj v tng ng ta c ai | aj .

1.3. Mt s phng php gii bi ton v dy s

13

V d 1.9 (Tp chi AMM). Xt n s nguyn dng a1 < a2 < < an 2n


sao cho [ai , aj ] > 2n vi mi i 6= j. Chng minh rng a1 > 2n/3.
Gii. Nu a1 2n/3, ta xt n + 1 s 2a1 , 3a1, a2, . . ., an . Cc s ny u khng
ln hn 2n v khng c s no l bi ca s no. iu ny mu thun vi kt qa
bi ton trn.
V d 1.10. (Canada, 2000) Cho A = (a1, a2, ..., an) l dy cc s nguyn thuc
on [1000, 1000]. Gi s tng cc s hng ca A bng 1. Chng minh rng tn
ti mt dy con (cha t nht 1 phn t) ca A c tng bng 0.
Gii. Ta c th gi s trong A khng c phn t no bng 0, v nu ngc
li th bi ton hin nhin. Ta sp xp dy A thnh dy B = (b1, ..., b2000) bng
cch chn dn t cc s hng ca dy A theo quy tc sau: b1 > 0, b2 < 0. Vi
mi i 3 chn bi l s c du ngc vi du ca tng si1 = b1 + + bi1
(v sao lun thc hin c?). Bng cch xy dng nh th, ta c 2000 s
s1 , s2 , ..., s2000 nm trong on [999, 1000]. Nu trong s si c mt s bng 0
th bi ton ng. Trong trng hp ngc li, theo nguyn l Dirichlet tn ti
i < j sao cho si = sj . Khi bi+1 + + bj = 0.
H m c s v dy s nguyn
H m c s c th dng xy dng nhiu dy s c tnh cht rt th
v. Nhn trn phng din ca mt c s khc, c th rt kh nhn ra quy lut,
nhng nu chn ng c s th bi ton tr nn v cng n gin.
Xin nhc li l vi b l mt s nguyn dng ln hn hay bng 2 th mi s
nguyn dng N u c th biu din mt cch duy nht di dng
N = a1 ...ak (b) = a1 bk1 + + ak vi 1 a1 b 1, 0 a2 , . . . , ak b 1.
l nh ngha h m c s dng c bn nht. Tuy nhin, c th ly mt dy
s nguyn bt k (c tr tuyt i tng nghim ngt) lm h m c s v d h
m c s (2), h m c s Fibonacci (3 = 4 2 + 1, 17 = 13 + 3 + 1...)
Cc h m thng s dng nht l h m c s 2 v c s 3. Di y ta xt
mt vi v d:
V d 1.11 (IMO 1983). Chng minh hoc ph nh mnh sau: T tp hp
105 s nguyn dng u tin lun c th chn ra mt tp con gm 1983 s sao
cho khng c ba s no lp thnh mt cp s cng.
Gii. Ta chng minh mnh tng qut: T 3n s t nhin u tin lun c
th chn ra 2n s sao cho khng c ba s no lp thnh mt cp s cng. Tht
vy, xt trong h m c s 3 tp hp tt c cc s c n ch s. Chn cc s
m trong biu din tam phn ca n ch cha ch s 2 v ch s 0. Khi c 2n
s nh vy v khng c ba s no trong chng lp thnh mt cp s cng.

1.3. Mt s phng php gii bi ton v dy s

14

V d 1.12 (Singapore 1995). Cho dy s {fn } xc nh bi f1 = 1, f2n = fn


v f2n+1 = f2n+1 .
(i) Tnh M = max{f1 , ..., f1994}
(ii) Tm tt c cc gi tr n, 1 n 1994 sao cho fn = M .
Gii. Kinh nghim mt cht ta thy ngay fn chnh l tng cc ch s ca n
trong h m nh phn. T y do 1994 < 2048 = 211 suy ra M = 10.
V d 1.13. Dy s {fn } c xc nh bi f1 = 1, f2n = 3fn , f2n+1 = f2n+1 .
Hy tnh f100.
Gii. fn c xc nh nh sau: Xt biu din nh phn ca n ri tnh gi
tr ca s nh phn ny trong h tam phn. V 100 = 26 + 25 + 22 nn f100 =
36 + 35 + 32 = 981.
V d 1.14. Dy s {an } c xc nh bi 0 a0 < 1, an = 2an1 nu 2an1 <
1 v an = 2an1 1 nu 2an1 1. Hi c bao nhiu gi tr a0 a5 = a0 .
Gii. Phn tch: Khi tnh an theo an1 ta c th la chn mt trong hai cng
thc. Tt nhin, vi a0 chn ri th tt c cc bc tip theo u xc nh mt
cch duy nht. Tuy nhin, ta c th chn a0 nh th no sau cc cng
thc tnh theo ng kch bn cho. C 25 = 32 kch bn nh vy. V d vi kch
bn (1, 1, 2, 1, 2) ta c x1 = 2x0, x2 = 2x1 = 4x0, x3 = 2x2 1 = 8x0 1, x4 =
2x3 = 16x0 2, x5 = 2x4 1 = 32x0 3.
Gii phng trnh x0 = x5 ta c x0 = 3/31. Tt nhin, c c mt li
gii hon chnh, ta cn phi lp lun cht ch thy rng cc x0 thu c l
khc nhau v vi mi x0 thu c, dy s s "i" ng nh kch bn nh.
Tuy nhin, phn tch ny gi chng ta hng n h nh phn. V ta c li gii
p mt sau:
Nu a0 = 0, d1d2d3 . . . l biu din nh phn ca a0 th a1 = 0, d2d3 d4 . . . Tht
vy, nu 2a0 < 1 th d1 = 0 v a1 = 2a0 = 0, d2d3d4 . . . cn nu 2a0 1 th d1 = 1
v a1 = 2a0 1 = 0, d2d3d4 . . .
Hon ton tng t, a2 = 0, d3d4 d5 . . . , . . . , a5 = 0, d6d7d8 . . . Nh vy a5 = a0
khi v ch khi a0 l phn s nh phn tun hon chu k 5. C 25 = 32 chu k tun
hon nh vy, trong chu k 11111 cho chng ta a0 = 1 (loi). Vy tt c c
31 gi tr a0 tha mn yu cu bi. l 0, (00000), 0, (00001), . . ., (0, 11110).
Tnh sang h thp phn l cc gi tr 0, 1/31, 2/31, . . . , 30/31.
S phc v dy s nguyn
S phc c nhng ng dng rt quan trng trong ton hc ni chung v trong
l thuyt dy s ni chung. Nh s phc, chng ta c th thy c mi quan h
gia hm lng gic v hm m. Nh s phc, mi a thc bc n u c n

1.3. Mt s phng php gii bi ton v dy s

15

nghim v v vy nh l Vit mi pht huy c tc dng. Di y ta xt mt


s v d v ng dng ca s phc trong cc bi ton tnh tng v dy truy hi.
V d 1.15. Vi s nguyn dng n, hy tnh
A(n) = Cn0 + Cn3 + + Cn3[n/3] .
Gii. C th t B(n) = Cn1 + Cn4 + + C(n) = Cn2 + Cn5 + ri s dng cc
cng thc
A(n) + B(n) = B(n + 1), B(n) + C(n) = C(n + 1), C(n) + A(n) = A(n + 1)
tm cng thc tnh A(n). Tuy nhin da theo cch tnh Cn0 +Cn2 + +Cn2 [n/2]
bng cch thay x = 1, y = 1 v x = 1, y = 1 vo cng thc nh thc Newton, ta
c cch gii khc kh p nh sau: Gi  l s tha mn phng trnh 2 ++1 = 0.
Do 3 = 1 nn ta c
(1 + 1)n = A(n) + B(n) + C(n)
(1 + )n = A(n) + B(n) + 2 C(n)
(1 + 2 )n = A(n) + 2 B(n) + C(n)
T y suy ra 3A(n) = 2n + (1 + )n + (1 + 2 )n . T y, dng cng thc
Moivre ta tm c
A(n) = [2n + 2 cos(np/3)]/3.
V d 1.16. Tnh tng Sn (x) = Cn0 + Cn1 cos x + + Cnn cos nx.
Gii. t T n(x) = 0 + Cn1 sin x + + Cnn sin nx th Sn (x) + iTn (x) = Cn0 +
Cn1 (cos x+i sin x)+ +Cnn (cos x+i sin x)n = (1+cos x+i sin x)n = 2[cos(x/2)[cos(x/2)+
i sin(x/2)]]n = 2n cosn (x/2)[cos(nx/2) + i sin(nx/2)].
T suy ra Sn (x) = 2n cosn (x/2) cos(nx/2).
V d 1.17 (AMM). Cho dy s {un } xc nh bi u0 = 3, u1 = 0, u2 =
2, un+3 = un+1 + un . Chng minh rng up lun chia ht cho p nu p l s nguyn
t.
Gii. Phng trnh c trng ca dy s c dng x3 x 1 = 0. Nu phng
trnh c trng ny c nghim nguyn th ta c th s dng nh l nh Fermat
chng minh kt lun ca bi ton. Tuy nhin, cc nghim ny khng nguyn,
thm ch phng trnh ch c 1 nghim thc. Ta phi cu cu n s tr gip ca
s phc.
Gi u, v, w l ba nghim ca phng trnh th u+v+w = 0, uv+vw+wu = 1,
suy ra u2 + v 2 + w2 = (u + v + w)2 2(uv + vw + wu) = 2. T ta c th kt
lun
u n = un + vn + wn

1.3. Mt s phng php gii bi ton v dy s

16

Pp1
Vi p l s nguyn t l th up = (v + w)p = v p wp i=1 Cpi v iwpi .
P
P
i i pi
Tng t v p = wp up i = 1p1 Cpi wi upi , wp = up v p p1
.
i=1 Cp u v
P
p1
p
p
p
i
i
pi
i
pi
i
pi
T suy ra 3(u + v + w ) = i=1 Cp (v w
+w u
+uv )
By gi, ch rng Cpi chia ht cho p vi 1 i p 1i (v p l s nguyn t)
v (v iwpi + wi upi + ui v pi ) l s nguyn (biu thc i xng i vi u, v, w)
nn v phi l mt s nguyn chia ht cho p. Vy vi p nguyn t, p > 3 bi ton
c chng minh. Cui cng ch u2 = 2, u3 = 3 ta c bi ton ng vi mi
p.
Dy s dng [n]
Dy s dng xn = [n] c nhiu tnh cht s hc th v. Nu a > 1 th
{[n ]}n1 l dy cc s nguyn dng phn bit, c s bin thin gn ging mt
cp s cng nhng li khng phi l mt cp s cng. Dy s ny c bit th v
khi a l s v t bc hai. Ta c mt kt qa quen thuc sau y
nh l 1.11. Nu a, b l cc s v t dng tho mn iu kin 1/a + 1/b = 1
th hai dy s xn = [n], yn = [n], n = 1, 2, 3, ... lp thnh mt phn hoch ca
tp hp cc s nguyn dng.
Chng minh. Xt hai dy s , 2, 3, ...v , 2, 3, ... Khng mt s hng
no trong cc s hng trn l s nguyn. Vi mi s nguyn dng N , c [N/] s
hng ca dy th nht nm bn tri N v [N/] s hng ca dy th hai. Nhng
N/ + N/ = N , v , l cc s v t, phn l ca cc s N/ v N/ l cc
s dng c tng bng 1 (do ng thc trn). Suy ra c [N/] + [N/] = N 1
s hng ca c hai dy nm bn tri N . V bn tri N + 1 c N s hng ca c
hai dy nn gia N v N + 1 c ng mt s hng ca mt trong hai dy, t
suy ra iu phi chng minh.
Cu hi: C th pht biu v chng minh nh l o nh th no?
Hai dy s trn vt ht tp hp cc s nguyn dng. iu ny cho chng ta
mt hng suy ngh: nu hai dy s vt ht tp hp cc s nguyn dng th c
kh nng chng s c dng trn. V nhiu bi ton c xy dng theo hng
ny. Chng ta xt mt v d
V d 1.18 (AMM). Gi s {fn } v {gn } l hai dy s nguyn dng c xc
nh nh sau
1) f1 = 1
2) gn = na 1 fn , trong a l s nguyn ln hn 4,
3) fn+1 l s nguyn dng nh nht khc cc s f1 , f2, ..., fn, g1, g2, ..., gn.
Chng minh rng tn ti cc hng s , sao cho fn = [n], gn = [n] vi
mi n = 1, 2, 3, ...

1.3. Mt s phng php gii bi ton v dy s

17

Gii. Theo cch xy dng {fn } v {gn } lp thnh mt phn hoch ca N . Gi


s ta tm c a, b tha mn iu kin u bi, khi , ta phi c 1/+1/ = 1.
Ngoi ra, khi n ln th na 1 = fn + gn n + n, suy ra + = a. Vy
, phi l nghim ca phng trnh x2 ax + a = 0.
Xt phng trnh x2 ax + a = 0 c hai nghim < . V a > 4, , l cc
s v t. Dy s {fn } v {gn } c xc nh mt cch duy nht, do chng
minh khng nh ca bi ton, ta ch cn chng minh {[n]} v {[n]} tha mn
cc iu kin 1), 2), 3).
R rng [a] = 1, [n] = [n(a )] = n + [n)] = na [n] 1 (do n v
t).
Gi s [n] = [m] = k, t n = k + r, m = k + s vi 0 < r, s < 1 th
n + m = k(1/ + 1/) + r/ + s/ = k + r/ + s/,
iu ny khng th xy ra v 0 < r/ + s/ < 1. Nh vy vi mi m, n ta c
[n] 6= [m].
Tip theo,
[(n + 1)] [n] + 1, [(n + 1)] [n] + 2 > [n] + 1.
Cui cng gi s k l mt s nguyn bt k v n = [(k + 1)/]. Nu n > k/
th k < n < (k + 1)/ = k + 1 v [n] = k. Nu n < k/ th (k n) >
k k/ = k(1 1/) = k, (k n) < k ((k + 1)/ 1) = k + 1, suy ra
[(k n)] = k.
T cc nhn xt trn ta suy ra mi s nguyn dng k c mt trong dy s
ng mt ln v hai dy s {[n]} v {[n]} tha mn iu kin 3) (pcm)
Ghi ch: Trong li gii trn, ta khng dng n kt qu ca nh l trn
v cng chnh l mt cch chng minh khc cho nh l.
Cc bi ton v dy s dng {[n]} thng lin quan n phn hoch v cc
dy s gn tuyn tnh (xm+n xm + xn ). Xin xem thm mt s v d trong phn
bi tp.

1.3.3

Dy s v phng trnh

Dy s c mi quan h rt cht ch vi phng trnh. iu ny c th thy


rt r qua hai v d c bn: phng trnh sai phn tuyn tnh c gii bng vic
xt nghim ca phng trnh c trng, gii hn ca dy s cng thng c
gii ra t mt phng trnh. V vn ny, xin c thm cc mc tng ng
trong bi ny. y l mt trong nhng ni dung quan trng nht trong phn dy
s.

1.3. Mt s phng php gii bi ton v dy s

1.3.4

18

Mt vi th thut khc

Sp xp li th t
Sp xp li th t l mt th thut thng c p dng trong cc bi ton
lin quan n bt ng thc trong dy s. Vic sp xp li th t cc s trn
ng thng dn n cc tnh cht c bit m mt dy s bt k khng c,
chng hn nu a < b < c th |c a| = |c b| + |b a|. Cng nh cc nguyn l
c bn khc, nguyn l n gin ny t ra kh hu hiu trong nhiu trng hp.
V d 1.19 (Vit Nam 1998). Tn ti hay khng mt dy s thc {xn } tha
mn iu kin
1) |xn | 0, 666 vi mi n = 1, 2, 3, ...
2) |xm xn | 1/n(n + 1) + 1/m(m + 1) vi mi s nguyn dng m 6 n.
Gii. Gi s tn ti dy s nh vy. Vi mi s nguyn dng N , ta sp xp li
cc s x1 , ..., xN theo th t tng dn
xi1 xi2 xiN
Khi |xiN xi1 | = |xiN xiN 1 |+
P +|xi2 xi1 |1/iN (iN +1)+1/iN 1(iN 1 +
1)+ +1/i2(i2 +1)+1/i1(i1 +1) = 2 1/ik (ik +1)1/iN (iN +1)1/i1(i1 +1) =
A(N ).
V i1, i2, ..., iN ch l mt hon v ca 1, 2, ..., N nn ta c
X
A(N ) = 2
1/k(k + 1) 1/iN (iN + 1) 1/i1(i1 + 1)
= 2(1 1/(N + 1)) 1/iN (iN + 1) 1/i1(i1 + 1)
2(1 1/(N + 1)) 1/1.2 1/2.3 = 4/3 2/(N + 1)
By gi ch rng |xiN xi1 | 2x0, 666 < 4/3. Chn N ln sao cho 4/3
2/(N + 1) > 2x0, 666, ta suy ra mu thun. Vy khng tn ti dy s tha mn
yu cu bi.
V d 1.20 (Lin X 1986). Gi s a1 , a2, ..., an l cc s dng tu . Chng
minh bt ng thc
1/a1 + 2/(a1 + a2 ) + + n/(a1 + + an ) < 4(1/a1 + 1/a2 + + 1/an )
Gii. V phi khng thay i nu ta thay i th t ca ai do ta ch cn
(v phi) chng minh bt ng thc ng cho trng hp tng bn tri ln
nht. iu ny xy ra khi ai c sp theo th t tng dn. Tht vy, gi s
0 < b1 b2 ... bn l cc s ai c sp xp li. Khi r rng vi mi k ta
c b1 + + bk a1 + + ak v
1/a1 +2/(a1 +a2 )+ +n/(a1 + +an ) 1/b1 +2/(b1 +b2 )+ +n/(b1 + +bn )

1.3. Mt s phng php gii bi ton v dy s

19

Vi mi k, ghp cc s hng ca tng bn phi thnh cp ta c nh gi sau


(2k1)/(b1+ +b2k1 )+2k/(b1+ +b2k1) < (2k1)/kbk +2k/(k+1)bk < 4/bk
T suy ra bt ng thc cn chng minh.
Php th lng gic
Nhiu dy s i s vi cng thc phc tp c th tr thnh cc dy s n
gin nh php th lng gic. Th thut ny c bit hiu quan trong cc bi
ton chng minh mt dy s l tun hon hay khng tun hon. p dng
c th thut ny, iu cn thit l bit cc cng thc lng gic v mt cht
nhy cm ton hc.
V d 1.21 (Vit
p Nam, 1990). Cho {xn } l dy s tha mn iu kin |x1| < 1,
xn+1 = (xn + 3 3x2n )/2 (n 1)
a) x1 phi tha mn iu kin g tt c cc s hng ca dy s u dng?
b) Dy s trn c tun hon khng?
iu kin |x1| < 1 v dng ca hm s gi ngay cho chng ta php t
x1 = cos vi thuc (0, ) khi x2 = ( cos + 3 sin )/2 = cos( 2/3).
T suy ra xn+1 = cos( 2n/3). T y c th d dng tr li cc cu hi
ca bi.
V d 1.22 (KVANT). Cho dy s un xc nh bi: u1 = 2, un+1 = (2 +
un )/(1 2un ).
a) Chng minh rng un 6= 0 vi mi n nguyn dng
b) Chng minh dy khng tun hon
Gii. t = arctan 2, tan = 2. Khi nu un = tan x th un+1 = tan( + x),
suy ra un = tan(n). S dng cng thc tan 2x = 2 tan x/(1 tan2 x) suy ra
u2n = 2un /(1 u2n ). T y nu u2n = 0 th un = 0. Nu tn ti n sao cho
un = 0 th s dng tnh cht ny, ta suy ra tn ti s sao cho u2s + 1 = 0 hay
(2 + u2s )/(1 2u2s ) = 0 hay u2s = 2, 2us /(1 us2 ) = 2. Suy ra us v t. iu
ny v l. Phn b) l h qu ca cu a).
V d 1.23. Tm cng thc tng qut tnh s hng ca dy s x0 = a, xn+1 =
2 x2n .
Gii. Nu |a| 2 th t a = 2 cos , ta c xn = 2 cos(2n). Nu |a| > 2,
n
n
t a = (a + 1/a) th ta c xn = (2 + 1/2 ).
V d 1.24 (Th Nh K 1997). Hai dy {an }, {bn} c xc nh bi a1 =
, b1 = , an+1 = an bn , bn+1 = an + bn . C bao nhiu cp (a, b) tha mn
a1997 = b1, b1997 = a1 ?

1.3. Mt s phng php gii bi ton v dy s

20

Gii. Ta c a2n+1 + b2n+1 = (a2 + b2)(a2n + b2n ) nn yu cu bi ton xy ra ch


khi 2 + 2 = 1. t a = cos , = sin th an = cos(n), bn = sin(n). T
suy ra li gii ca bi ton.
Php th lng gic thng c p dng trong cc bi ton c cng thc
"gi nh" n cc cng thc lng gic hoc c kt qu ging tnh cht hm
lng gic (chng hn tnh tun hon hoc tnh b chn). Tuy nhin, php th
lng gic c th xut hin nhng trng hp m tng chng khng dnh dng
g n vi lng gic.
V d 1.25. Vi mi s t nhin n > 1 v n s thc dng x1 , x2, ..., xn t
f = max{x1 , 1/x1 + x2, ..., 1/xn1 + xn , 1/xn}.
Hy tm min f .
Gii. Tng chng nh bi ton ny khng lin quan g n lng gic. V hn
th, cng chng lin quan g n dy s. Tuy nhin, iu kin t gi tr nh nht
ca f s to ra mt dy s! Ta chng minh rng nu x1 , x2, ..., xn l n s thc
m ti f t min th ta phi c x1 = 1/x1 + x2 = ... = 1/xn1 + xn = 1/xn .
V bi ton dy s xut hin: Vi mi s nguyn dng n, xt dy s {xk }nk=1
xc nh bi x1 = a v xk = x1 1/xk1, vi k = 2, ..., n. Hy tm a sao cho
1/xn = x1 . V bi ton cui cng ny c th gii nh sau. t x1 = 2 cos th
x2 = 2 cos 1/2 cos = (4 cos2 1)/2 cos = sin3 / sin2 , x3 = 2 cos
sin 2/ sin 3 = sin 4/ sin 3... Tip tc nh vy suy ra xn = sin(n+1)/ sin n.
T ng thc 1/xn = x1 sin n/ sin(n + 1) = 2 cos sin(n + 2) = 0. n
y, t iu kin xk dng ta suy ra = /(n + 2) v min f = 2 cos(/(n + 2)).
Cu hi:
1) Ti sao c th khng nh khi f t min th cc gi tr trn y phi bng
nhau?
2) Ti sao c th t x1 = 2 cos ?
3) Lm sao c th d on ra cch t trn?
4) Php gii trn cn cha cht ch im no?
5) Mi s thc x u c th biu din di dng x = 2 cos hoc, x = a+1/a.
iu c ngha g?
Dy s ph
Khi kho st s hi t ca mt dy s ta thng nh l v dy n iu v b
chn. Nu dy khng n iu th c th th xt dy vi ch s chn v dy vi
ch s l. Tuy nhin, c nhng dy s c "hnh vi" phc tp hn nhiu. Chng
tng gim rt bt thng. Trong mt s trng hp nh th, ta c th xy dng
mt (hoc 2) dy s ph n iu, chng minh cc dy s ph c gii hn v

1.3. Mt s phng php gii bi ton v dy s

21

sau chng minh dy s ban u c cng gii hn. Tt nhin, dy s ph phi


c xy dng t dy s chnh.
V d 1.26. Dy s {an } c xc nh bi a1 > 0, a2 > 0 v an+1 = 2/(an +
an1 ). Chng minh rng dy s {an } hi t v tm gii hn ca dy s .
Gii. Xt hai dy
Mn = max{an , an+1, an+2 , an+3 }
mn = min{an , an+1 , an+2, an+3 }
Ta chng minh Mn l dy s gim v mn l dy s tng. Tht vy, ta s chng
minh an+4 max{an+1 , an+3}. T y suy ra Mn+1 = an+1 hoc an+2 hoc
an+3 v r rng khi Mn = max{an , an+1 , an+2 , an+3 } Mn+1 . Tht vt
nu an+4 an+3 th 2/(an+3 + an+2 ) an+3 suy ra 2 (an+3 + an+2 )an+3 .
Khi an+1 = 2/an+3 an+2 = 2/an+3 2/(an+2 + an+3 ) an+2 + an+4 =
2an+2 /(an+3 + an+2 )an+3 an+2 + an+4 an+4 suy ra pcm. Vy ta chng
minh c Mn gim. Tng t mn tng. Hai dy s ny u b chn nn hi t.
Cui cng, ta ch cn cn chng minh hai gii hn bng nhau.

V d 1.27. Dy s {an } c xc nh bi a1 > 0, a2 > 0 v an+1 = an +

an1 . Chng minh rng dy s {an } hi t v tm gii hn ca dy s .


Gii. Xt dy s Mn = max{an , an+1 , 4}.
Nu Mn = 4 th an , an+1 4, suy ra an+2 4, t Mn+1 = 4.

Nu Mn = an+1 th an+1 an , 4. Khi an1 = an+1 an+1 an+1 ,

suy ra an+2 = an + an+1 an + an1 = an+1 suy ra Mn+1 = max{an+1 , an+2 , 4} =


an+1 .

Nu Mn = an th an an+1 , 4. Khi an+2 = an + an+1 2 an . Suy ra


Mn+1 an = Mn .
Vy trong mi trng hp th Mn+1 Mn , tc l dy {Mn } l dy s gim.
Do Mn b chn di bi 4 nn dy ny c gii hn. Ta chng minh gii hn ny
bng 4. Thc vy, gi s gii hn l M > 4. Khi vi mi  > 0, tn ti N sao
cho vi mi n N th M  < Mn < M + . Chn n N sao cho Mn+2 = an+2
(theo cc lp lun trn v do M > 4 th tn ti ch s n nh vy). Ta c

M  < Mn+2 = an+2 = an + an1 < 2 M + 


hay M (M 4) (2M + 4 ) < 0
Mu thun v M > 4 v  c th chn nh tu .

1.3. Mt s phng php gii bi ton v dy s

22

Phng php sai phn


tnh tng n s hng u tin ca mt dy s, mt trong nhng phng
php hiu qu nht l phng php sai phn: tnh tng n s hng u tin
ca dy s {an }, ta tm hm s f (n) sao cho an = f (n + 1) f (n). Khi
a0 + + an1 = f (n) f (0).
Mt trong nhng v d kinh in chnh l phng php m Bernoulli v
cc nh ton hc th k 18 a ra tm cng thc tnh tng S(k, n) =
1k + 2k + + nk . Dng phng php h s bt nh, h tm a thc fk (n) sao
cho nk = fk (n + 1) fk (n) v t tm c S(k, n) = fk (n + 1) fk (n). Phng
php ny hiu qu hn phng php xy dng cng thc truy hi, v tnh Sk
ta khng cn phi dng n cc cng thc tnh Sk1 , Sk2
Khi d on cc hm f , ta c th s dng tch phn ri tng t ha qua.
V d tch phn ca a thc bc k l a thc bc k + 1. Vy th fk = nk suy
ra fk phi c bc k + 1.
Tuy nhin, khc vi tch phn, i khi cc hm ri rc khng c "nguyn
hm". Trong trng hp ta khng tnh c tng m ch c th nh gi tng
bng cc bt ng thc.

V d 1.28. Tm phn nguyn ca tng S = 1/1 + 1/ 2 + + 1/ 100.

Gii. Ta cn tm mt nh gi cho S. Nhn xt rng hm 1/ x


c nguynhm

l 2 x, ta xt hm s f (n) = 2 n. Khi f (n + 1) f (n) = 2 n + 1 2 n =

2/( n + 1 + n).

Suy ra, 1/ n + 1 < f (n + 1) f (n) < 1/ n. T , 2( 101 1) < S <


2( 100 1) + 1, suy ra [S] = 18.
V d 1.29 ( ngh Ton quc t 2001). Cho x1 , x2, ..., xn l cc s
thc bt k. Chng minh rng

x1/(1 + x21 ) + x2 /(1 + x21 + x22) + + xn /(1 + x21 + + x2n ) < n.


Gii. t v tri ca bt ng l A. p dng bt ng thc Bunhiacopsky ta
c
A2 n[x21 /(1 + x21 )2 + x22 /(1 + x21 + x22)2 + + x2n /(1 + x21 + + x2n )2 ]
chng minh bt ng thc u bi, ta ch cn chng minh
x21 /(1 + x21)2 + x22 /(1 + x21 + x22 )2 + + x2n /(1 + x21 + + x2n )2 < 1.
Nhng iu ny l hin nhin do bt ng thc
x2k /(1 + x21 + + x2k )2 1/(1 + x21 + + x2k1 ) 1/(1 + x21 + + x2k ).

1.4. Mt s phng php xy dng h thng bi tp

23

V d 1.30. Xt dy s {xn }n=1 cho bi: xn+2 = [(n 1)xn+1 + xn ]/n. Chng
minh rng vi mi gi tr ban u x1 , x2, dy s cho hi t. Tm gii hn ca
dy nh mt hm s theo x1, x2.
Gii. Ta c t cng thc ca dy s xn+2 xn+1 = (xn+1 xn )/n = (xn
xn1 )/n(n 1) = = (1)n (x2 x1 )/n!. T suy ra xn+2 = (xn+2 xn+1 ) +
(xn+1 xn ) + + (x2 x1) + x1 = x1 + (x2 x1 )Kn , trong Kn = 1 1/1! +
1/2! + (1)n/n!. T y suy ra dy s c gii hn v gii hn bng
x1 + (x2 x1)/e.
Cu hi:
1) C th tng qut ha bi ton trn nh th no?
2) Hy tm sai phn ca cc hm s arctan(n). T t ra bi ton tnh
tng tng ng.
3) Tm sai phn ca hm s ln(n). T tm nh gi cho tng 1 + 1/2 +
+ 1/n.
4) T cng thc sin 3x = 3 sin x 4 sin 3x c th lp ra cng thc tnh tng
no?

1.4
1.4.1

Mt s phng php xy dng h thng bi tp


Xy dng dy hi t bng phng trnh

C th xy dng dy s hi t v mt s a xut pht t mt phng trnh c


nghim l a theo cch sau:

V d 1.31. Xt a = 2, l nghim ca phng trnh 2 = 2. Ta vit li di


dng
= 2/ 2 = + 2/ = ( + 2/)/2
v ta thit lp dy s xn tho mn x0 = a, xn+1 = (xn + 2/xn )/2. Nu dy ny
hi t th gii hn s l 2. Tng t nh vy, ta c th xy dng c dy s
tin v cn bc k ca m nh sau:
x0 = a, xn+1 = (xn + m/xk1
n )/2

Cng vi gii hn cn n l 2, ta c th xy dng mt dy s khc theo


"phong cch" nh vy:
x0 = a, xn+1 = 1 + xn x2n /2
Tt nhin, trong tt c cc v d trn, ta ch c c phng trnh vi nghim
theo mun khi chng minh c s hi t ca dy s. V vy, cn cn thn
vi cch thit lp bi ton kiu ny. V d, vi dy s xn+1 = 1 + xn x2n /2 th
khng phi vi x0 no dy cng hi t, v khng phi lc no gii hn cng l.

1.4. Mt s phng php xy dng h thng bi tp

24

Mt cch tng qut, ta c th dng phng php tm nghim xp x Newton


xy dng cc dy s. tm nghim ca phng trnh F (x) = 0, phng php
Newton ngh chn x0 tng i gn nghim v xy dng dy truy hi
xn+1 = xn F (xn )/F 0(xn )
khi dy xn s dn n nghim ca phng trnh F (x) = 0.
V d 1.32. Xt hm s F (x) = x2 2, th F (x)/F 0 (x) = (x2 2)/2x v ta
c dy s xn+1 = (xn + 2/xn )/2.
Xt hm s F (x) = x3 x th F (x)/F 0 (x) = (x3 x)/(3x2 1) v ta c dy
s
xn+1 = 2x3n /(3x2n 1)

1.4.2

Xy dng dy truy hi t cp nghim ca phng trnh


bc 2

Chng ta thy, t hai nghim ca mt phng trnh bc 2 c th xy dng


ra cc dy truy hi tuyn tnh bc 2 (kiu dy s Fibonacci). Tng t nh th,
c th xy dng cc dy truy hi tuyn tnh bc cao t nghim ca cc phng
trnh bc cao. Trong phn ny, chng ta s i theo mt hng khc: xy dng
cc dy truy hi phi tuyn bc nht t cp nghim ca phng trnh bc 2.
Xt phng trnh bc 2: x2 mx 1 = 0 c hai nghim l v . Xt mt s
n
n
n+
n+1
thc a bt k. Xt dy s xn = a(2 + 2 ). Khi x2n = a2 (2 + 2 + 2) =
axn+1 + 2a2 , t suy ra dy s xn tho cng thc truy hi: xn+1 = x2n /a 2a.
V d chn a = 1/2, m = 4, ta c bi ton: Tm cng thc tng qut ca dy
s xn c xc nh bi x0 = 2, xn+1 = 2x2n 1.
n
n
n+1
n+1
Tng t nh vy, nu xt xn = a(3 + 3 ) th x3n = a3 (3
+ 3

n
n
3(3 + 3 ) = a2 (xn+1 3xn ). T suy ra dy s xn tho cng thc truy hi
xn+1 = x3n /a2 (3xn ).
V d xt , l hai nghim ca phng trnh x2 4x 1 = 0, a = 1/4,
ta c bi ton: Tm cng thc tng qut ca dy s xn c xc nh bi
x0 = 1, xn+1 = 16x3n + 3xn . Hon ton tng t, c th xy dng cc dy truy
hi phi tuyn dng a thc bc 4, 5. Bng php di trc, ta c th thay i dng
ca cc phng trnh ny.
V d 1.33. nu trong dy x0 = 2, xn+1 = 2x2n 1 ta t xn = yn 1/2 th ta
c dy yn tho: y0 = 5/2, yn+1 = 2(yn2 yn ).
Nu , l cc s thc th trong hai s c t nht mt s c tr tuyt i ln
hn 1, v vy dy s khng hi t (Tr trng hp hai nghim i nhau v dy
l dy hng). Tuy nhin, nu chn , l cp s phc lin hp c mun nh
hn hay bng 1, ta c th to ra cc dy tun hon hoc dy hi t. Ch rng

1.4. Mt s phng php xy dng h thng bi tp

25

chn , y chnh l chn m v cng chnh l chn x0 . Do tnh cht ca


dy s s ph thuc rt nhiu vo x0 .

2 1, nu x = 2 th x = [(2 + 3)2n + (2
V
d
vi
dy
s
tho
x
=
2x
n+1
0
n
n
2n
3) ]/2; nu x0 = 1 th xn l dy hng; nu x0 = cos th xn = cos(2n ).
Cu hi:
1) Xt xem vi nhng a, b, c no th phng trnh sai phn xn+1 = ax2n +bxn +c
gii c bng phng php trn?
n
n
2) Hy tm dng ca cc dy truy hi to c bng cch xt xn = a(k + k )
vi k = 4, 5.

1.4.3

Xy dng cc dy s nguyn t li gii cc phng trnh


nghim nguyn

Mt dy truy hi tuyn tnh vi h s nguyn v cc s hng u u nguyn


s cha ton s nguyn. l iu hin nhin. Th nhng c nhng dy s m
trong cng thc truy hi c phn s, thm ch c c cn thc nhng tt c cc
s hng ca n vn nguyn. y mi l iu bt ng. Tuy nhin, nu xem xt
k, ta c th thy chng c mt mi quan h rt trc tip.
Chng ta hy bt u t bi ton quen thuc sau: Chng
p minh rng mi s
hng ca dy s {an } xc nh bi a0 = 1, an+1 = 2an + 3a2n 2 u nguyn.
Chuyn v v bnh phng cng thc truy hi, ta c
a2n+1 4an+1 an + 4a2n = 3a2n 2
a2n+1 4an+1 an + a2n + 2 = 0
Thay n bng n 1, ta c
a2n 4an an1 + a2n1 + 2 = 0
T y suy ra an1 , an+1 l hai nghim ca phng trnh
x2 4an x + a2n + 2 = 0
Suy ra: an+1 + an1 = 4an hay an+1 = 4an an1 . T y suy ra tt c cc s
hng trong dy u nguyn.
C cng thc ban u ln cng thc h qu an+1 = 4an an1 u gi cho
chng ta n vi phng trnh Pell. Qu tht l c th xy dng hng lot dy
s tng t bng cch xt phng trnh Pell.
Xt phng trnh x2 Dy 2 = k. Gi s phng trnh c nghim khng tm
thng (x0, y0) v (, ) l nghim c s ca phng trnh x2 Dy 2 = 1. Khi ,
nu xt hai dy {xn }, {yn} xc nh bi xn+1 = xn + Dyn , yn+1 = xn + yn
th xn , yn l nghim ca x2 Dy 2 = k.

1.4. Mt s phng php xy dng h thng bi tp

26

T h phng trnh trn, ta c th tm c


p
p
xn+1 = xn + D(x2n k); yn+1 = yn + k + Dyn2
v nh vy xut hin hai dy s nguyn c cho bi mt cng thc khng
nguyn.
V d, vi D = 4a(a + 1), k = 1 th ta c x0 = = 2a + 1, y0 = = 1. Ta
c hai dy s nguyn sau y:
p
x0 = 2a + 1, xn+1 = 2a + 1 + 4a(a + 1)(x2n 1)
p
y0 = 1, yn+1 = 2a + 1 + 4a(a + 1)yn2 + 1
Cui cng, ch rng ta c th to ra mt kiu dy s khc t kt qu an1 , an+1
l hai nghim ca phng trnh
x2 4an x + a2n + 2 = 0
trn y: Theo nh l Viet th an+1 an1 = a2n + 2, suy ra
an+1 = (a2n + 2)/an1
v ta c bi ton: Cho dy s {an } xc nh bi a0 = 1, a1 = 3 v an+1 =
(a2n + 2)/an1 . Chng minh rng an nguyn vi mi n.

1.4.4

Xy dng dy s l nghim ca mt h phng trnh ph


thuc bin n

Xt mt h phng trnh F (n, x) = 0. Nu vi mi n, phng trnh F (n, x) =


0 c nghim duy nht trn mt min D no th dy s xn c xc nh.
T mi lin h gia cc hm F (n, x), dy s ny c th c nhng tnh cht rt
th v.
V d 1.34. Vi mi s t nhin n 3, gi xn l nghim dng duy nht ca
phng trnh xn x2 x1 = 0. Chng minh rng lim xn = 1 v tm lim n(xn 1).
V d 1.35. Chng minh rng vi mi n nguyn dng, phng trnh
1/x + 2/(x 1) + 2/(x 4) + + 2/(x n2 ) = 0
c nghim duy nht xn thuc khong (0, 1). Tm limn xn .
V d 1.36. Chng minh rng vi mi n nguyn dng, phng trnh
1/x + 2/(x 1) + 2/(x 4) + + 2/(x n2 ) = 0
c nghim duy nht xn thuc (0, 1). Tm limn xn .

1.5. L thuyt dy s di con mt ton cao cp

27

to ra cc phng trnh c nghim duy nht trn mt khong no , c


th s dng tng ca cc hm n iu. Ring vi hm a thc ta c th s dng
quy tc -cc v s nghim dng ca phng trnh: Nu dy cc h s ca
phng trnh i du k ln th phng trnh c khng qu k nghim dng.
V d phng trnh x4 x2 nx 1 = 0 c nghim dng duy nht x0 , cn
phng trnh x4 x2 + nx 1 = 0 c nhiu nht hai nghim dng.
Khi xy dng cc hm F (n, x), c th s dng cng thc truy hi. Nh trong
v d trn th F (n + 1, x) = F (n, x) + 1/(x n 1). Xy dng F (n, x) kiu ny,
dy nghim xn s d c nhng quy lut th v hn. V d, vi dy s trn, ta c
F (n + 1, xn) = F (n, xn ) + 1/(xn n 1) < 0. T y, do F (n + 1, 0+ ) = ta
suy ra xn+1 nm gia 0 v xn , tc dy xn gim.
Cu hi:
1) C th xy dng dy s no vi h hm s F (x) = x(x 1) . . .(x n)?
2) Cho 0 < a1 < a2 < < an < l mt dy s dng tng nghim ngt.
Xt h phng trnh 1/x + 1/(x1 a1 ) + + 1/(x an ) = 0 c nghim duy nht
xn thuc (0, a1). Khi no th xn dn v 0 khi n dn n v cng?

1.5
1.5.1

L thuyt dy s di con mt ton cao cp


Ri rc ha cc khi nim v nh l ca l thuyt hm
bin s thc

Dy s l hm s, do n c y cc tnh cht chung ca hm s. Tuy


nhin, do tnh cht c bit ca N , mt s khi nim nh o hm, tch phn
khng c nh ngha cho cc dy s. Nhng thc ra, dy s cng c cc khi
nim tng ng vi cc khi nim ny. Bng cch so snh v php tng t, ta
c th tm c nhng nh l th v ca l thuyt dy s. l qu trnh ri
rc ha.
Ri rc ha ca o hm f 0 (x) chnh l sai phn xn = xn xn1 ca dy s.
Cng nh o hm ca hm bin s thc, sai phn dng xt tnh tng gim
ca dy s. Tng t nh vy, ta nh ngha sai phn cp 2 v dng o tnh
li lm ca dy. Ri rc ha ca khi nim tch phn chnh l khi nim tng:
S(xn ) = x0 + + xn . Hai khi nim ny ngc nhau: (S(xn)) = xn , S(xn) =
xn .
V d 1.37 (nh l Stolz). Xt hai dy s {xn } v {yn } trong {yn } l dy
s dng tng v dn n v cng. Th th lim xn /yn = lim(xn xn1 )/(yn yn1 )
vi gi thit l gii hn v phi tn ti. (So snh vi quy tc LHopitale)
Chng minh: t lim(xn xn1 )/(yn yn1 ) = A. Vi mi  > 0 tn ti
N1 sao cho vi mi n N1 ta c |(xn xn1 )/(yn yn1 ) A| < , suy ra

28

1.5. L thuyt dy s di con mt ton cao cp

A  < (xn xn1 )/(yn yn1 ) < A + . T y, do yn l dy tng nn ta c


(A )(yN1 yN1 1 ) < xN1 xN1 1 < (A + )(yN1 yN1 1 )
...
(A )(yn yn1 ) < xn xn1 < (A + )(yn yn1 )
Cng cc bt ng thc trn li, ta c
(A )(yn yN1 1 ) < xn xN1 1 < (A + )(yn yN1 1 )
Chia hai v cho yn , ta c
A  + [xN1 (A )yN1 1 ]/yn < xn /yn < A +  + [xN1 (A + )yN1 1 ]/yn
V yn dn n v cng nn tn ti N2 > N1 sao cho
[xN1 (A )yN1 1 ]/yn >  v [xN1 (A + )yN1 1 ]/yn < 
vi mi n N2. Khi vi mi n N2 ta c A 2 < xn /yn < A + 2 v iu
ny c ngha l lim xn /yn = A.
Cu hi: iu kin yn tng v dn n v cng c cn thit khng?
V d 1.38. Chng minh rng nu dy s {xn } tho mn iu kin xn+1 2xn +
xn1 0 v k1, k2, . . ., kr l cc s t nhin tho mn iu kin k1 +k2 + +kr =
r.k th
xk1 + + xkr r.xk
(So snh vi bt ng thc Jensen)
V d 1.39. Cho dy s {xn } tho mn iu kin xk+1 2xk + xk1 0 vi
mi k = 1, . . ., n. Ngoi ra x0 = xn+1 = 0. Chng minh rng xk 0 vi mi
k = 1, . . . , n.
(o hm bc 2 khng m, suy ra o hm bc nht l hm tng v ch c
nhiu nht 1 nghim, suy ra chiu bin thin ca hm s ch c th l 0 gim
cc tiu ri tng 0)
V d 1.40. Cho dy s dng {an }. Bit rng tn ti gii hn
n
X
1
= A < .
n
ak

lim

k=1

t sn = a1 + a2 + + an . Chng minh rng tng limn


c gii hn hu hn khi n .

Pn

k=1

k2 ak /s2k cng

1.5. L thuyt dy s di con mt ton cao cp

29

Gii. Dch sang ngn ng hm s, ta c bi ton sau "Nu f (x) l hm s tng


R dx
t R+ vo R+ v tn ti tch phn suy rng 0
th cng tn ti tch phn
f (x)
R x2f (x)dx
trong F (x) l nguyn hm ca f (x)". Bi ny c th gii bng
0
F 2 (x)
phng php tch phn tng phn nh sau:
Z A
Z A 2
Z
xdx
x f (x)dx
1 A d(x2 ) 1 x2 A
=
=
+
2 0 F (x)
2 F (x) 0
F 2 (x)
0 F (x)
0
nh vy ch cn chng minh tn ti

R xdx
x2
v
lim
.
x
0 F (x)
F (x)

Cu hi:
1) nh l Rolle c dng ri rc nh th no?
2) Cng thc tnh tch phn tng phn c dng ri rc nh th no?

1.5.2

Phng php hm sinh v bi ton tm s hng tng qut

Cho dy s a0, a1, . . . , an , . . . Hm sinh F (x) ca dy s ny l biu thc hnh


thc
F (x) = a0 + a1x + + an xn +
Cc php ton trn hm sinh c thc hin mt cch t nhin v chng ta
khng quan tm n tnh cht gii tch ca chng (bn knh hi t ca chui
tng ng c th bng 0). Php ton c bit nht ca hm sinh l php nhn:
Nu F (x), G(x) l hm sinh ca cc dy {an }, {bn} tng ng th F (x).G(x)
P
l hm sinh ca dy {cn } trong cn = n0 ai bni .
S ng dng ca hm sinh vo bi ton tm s hng tng qut ca dy s
nh sau: Gi s ta cn tm s hng tng qut ca dy s {an } cho bi mt cng
thc truy hi no . Ta thit lp hm sinh F (x) ca {an }. Da vo h thc
truy hi, ta tm c mt phng trnh cho F (x), gii phng trnh, ta tm c
F (x). Khai trin F (x) theo lu tha x (Khai trin Taylor), ta tm c an vi
mi n.
V d 1.41. Tm s hng tng qut ca dy s {an } xc nh bi: a0 = 3, a1 =
2, an+2 = 5an+1 6an .
Gii. Xt hm sinh F (x) = a0 + a1 x + a2 x2 + + an+2 xn+2 + . Vi mi n
t nhin, ta thay an+2 bng 5an+1 6an th c
F (x) = a0 + a1 x + (5a1 6a0 )x2 + + (5an+1 6an )xn+2 +
= a0 + a1 x + 5x(a1 x + + an+1 xn+1 + ) 6x2 (a0 + a1x + + an xn + )
= a0 + a1 x + 5x(F (x) a0 ) 6x2 F (x)

1.5. L thuyt dy s di con mt ton cao cp

30

Suy ra F (x) = (3 13x)/(6x2 5x + 1) = 7/(1 2x) 4/(1 3x) = 7(1 + 2x +


(2x)2 + + (2x)n + ) 4(1 + 3x + (3x)2 + + (3x)n + )
T an = 7.2n 4.3n.
Trn l thuyt, khi tm c F (x), ta phi dng cng thc Taylor tm khai
trin ca F (x). y l mt bi ton phc tp. Tuy nhin, trong nhiu trng
hp, cng thc nh thc Newton tng qut di y dng:
(1 + x) = 1 + x + [( 1)/2]x2 + + [( 1) . . .( n + 1)/n!]xn +
V d 1.42. Dy s {an } xc nh bi a0 = 1, a0an + a1 an1 + + an a0 = 1
vi mi n. Hy tm cng thc tng qut ca an .
Gii. Xt hm sinh F (x) = a0 + a1x + a2 x2 + + an xn + . T cng
thc truy hi ta suy ra F 2 (x) = 1 + x + x2 + + xn + = (1 x)1 . T
y F (x) = (1 x)1/2. Khai trin F (x) theo cng thc Newton, ta tm c
n
an = C2n
/22n .

1.5.3

i s tuyn tnh v phng trnh sai phn

Trong phn trn, chng ta s dng phng php hm sinh gii bi ton
tm cng thc tnh s hng tng qut ca mt dy s. Trong phn ny, ta s xem
xt cu trc nghim ca phng trnh sai phn di gc i s tuyn tnh.
Xt phng trnh sai phn thun nht: xn+k = a1 xn+k1 + + ak xn . D
thy rng nu dy s {xn }, {yn} tho mn phng trnh ny th {axn + byn } cng
tho mn phng trnh vi mi a, b. Nh vy tp hp tt c cc dy s tho mn
phng trnh sai phn trn lp thnh mt khng gian vc-t. Hn th, ta c nh
l:
nh l 1.12. Tp hp tt c cc dy s tho mn phng trnh sai phn
xn+k = a1xn+k1 + + ak xn .
l mt khng gian vct k chiu.
Chng minh nh l ny kh n gin: Dy s s hon ton xc nh nu
bit k s hng u tin. Gi {xin }(i = 0, k 1) l dy s c xij = 0 nu i 6= j v
xii = 1. Khi c th chng minh d dng rng cc dy {x1n }, . . . , {xkn} c lp
tuyn tnh v vi mi dy {xn } ta c
xn = x0x0n + + xk1 xk1
n
Nh th, cu trc nghim ca phng trnh sai phn tuyn tnh thun nht
l r. Ta ch cn tm mt c s no ca khng gian nghim l c th m
t c tt c cc nghim ca phng trnh sai phn. C s m chng ta a ra
trn khng c tnh tng minh, do kh c th s dng trong vic thit lp
cng thc tng qut. xy dng mt c s khc tt hn, ta c nh l:

1.5. L thuyt dy s di con mt ton cao cp

31

nh l 1.13. Nu l nghim bi r ca phng trnh c trng


xk a1xk1 ak = 0
th cc dy s {n }, . . ., {nr1n } tho mn phng trnh sai phn xn+k =
a1 xn+k1 + + ak xn .
Vi nh l ny, ta c th tm k dy s tng minh to thnh mt c s
ca khng gian nghim.
Cui cng, nu ta gp phng trnh sai phn tuyn tnh khng thun nht
xn+k = a1 xn+k1 + + ak xn + f (n).
th nghim tng qut ca phng trnh ny s c dng l tng ca nghim tng
qut ca phng trnh sai phn tuyn tnh thun nht tng ng vi mt nghim
ring ca phng trnh khng thun nht.
tm nghim ring, ta vn dng phng php hon ton tng t nh trong
phng trnh vi phn: Nu f (n) l a thc th ta xn tm di dng a thc, l
hm m th tm di dng hm m... y, trng hp c s l nghim kp ca
phng trnh c trng cng c x l tng t nh trong phng trnh vi phn.

1.5.4

S dng xp x trong d on kt qu

Trong nhiu trng hp, d on c kt qu l mt na, thm ch 2/3


li gii. Chng ta gp nhiu tnh hung l li gii u tin thu c mt cch
rt kh khn, nhng sau th hng lot li gii p hn, gn hn xut hin. V
sao chng ta khng ngh ngay c nhng li gii p? V chng ta cha bit p
s. Khi bit ri th c th nh hng d dng hn rt nhiu. Di y, chng ta
s xem xt mt s ng dng ca xp x trong vic d on kt qu.
Trong v d v dy s xn+1 = sin(xn ), chng ta p dng nh l trung

bnh Cesaro tm gii hn nxn , mc d dy s khng c dng quen thuc


xn+1 = xn (xn ) . Th nhng, nu rng xn 0 khi n , m ti ln
cn 0 th sin x x x3 /6 th ta s thy tnh quy lut ca kt qu tm c
trn.
Vi phng php tng t, ta c th thy dy dng xn+1 = xn (xn )
hng lot cc dy s c b ngoi khc hn nh: xn+1 = ln(1 + xn ), xn+1 =
xn cos xn , xn+1 = arctg(xn) . . . (D nhin, phi kim tra iu kin xn 0 khi
n ).

Ta cng c th gii thch c v sao trong bi ton an+1 = an +1/ an phn

3/2
trn, ta tm c s 3/2. Ta c an+1 = an +1/ an = an (1+1/an ). V an
3/2
3/2
khi n nn vi mi ta c an+1 = an (1 + 1/an ) an (1 + /an ) =
3/2

an + an
. Do hiu s ny xp x hng s, ta chn b = 3/2.
Ta xt mt v d khc

1.6. Bi tp

32

V d 1.43 (HSP, 2000). Cho dy s {an } xc nh bi: a1 = a2 = 1, an+1 =


an + an1 /n(n + 1). Chng minh rng dy {an } c gii hn.
Gii. D thy {an } l dy tng. V vy ta ch cn chng minh dy {an } b chn
trn. Ta c
an+1 = an + an1 /n(n + 1) < an [1 + 1/n(n + 1)]
T y suy ra
an+1 < [1 + 1/n(n + 1)] . . .[1 + 1/2.3]a2 = [1 + 1/n(n + 1)] . . .[1 + 1/2.3]
Nh vy ta ch cn chng minh tch [1 + 1/n(n + 1)] . . . [1 + 1/2.3] b chn. Kt
qu ny khng phc tp v c th chng minh hon ton s cp. Tuy nhin,
nhng kinh nghim v dy s 1/n(n + 1) gi cho chng ta ti mi quan h gia
tch trn v tng 1/2.3 + + 1/n(n + 1). Theo hng , chng ta c th a ra
mt kt qu tng qut hn v kt qu c d on t vic s dng xp x.
Gi s rng {xn } l dy s thc sao cho tng x1 + + xn c gii hn hu hn
khi n . Khi xn 0 khi n . V vy, vi n ln th xn ln(1 + xn ).
Do tng ln(1 + x1 ) + + ln(1 + xn ) cng c gii hn hu hn khi n v
c ngha l tch (1 + x1 ) . . . (1 + xn ) cng vy. Ta c nh l
nh l 1.14. Cho dy s thc {xn }. Khi nu tng x1 + + xn c gii hn
hu hn khi n th tch (1 + x1) . . . (1 + xn ) cng c gii hn hu hn khi
n .
Cu hi:
1) Mnh o ca nh l trn c ng khng?
2) Cho n > 3 v xn l nghim dng duy nht ca phng trnh xn x2
x 1 = 0. C th d on c limn n(xn 1)?

1.6

Bi tp

Bi 1.1 (Canada 1998). Cho m l s nguyn dng. Xc nh dy a0 , a1, a2, . . .


nh sau: a0 = 0, a1 = m v am+1 = m2an an1 vi n = 1, 2, . . . Chng minh
rng vi mi cp sp th t cc s t nhin (a, b) vi a b l nghim ca phng
trnh (a2 + b2)/(ab + 1) = m2 khi v ch khi (a, b) = (an , an+1 ) vi n l mt s t
nhin no .
Bi 1.2 (Bulgari 1978). Cho dy s {an } xc nh bi an+1 = (a2n + c)/an1 .
Chng minh rng nu a0 , a1 v (a20 + a21 + c)/a0 a1 l s nguyn th an nguyn vi
mi n.
Bi 1.3. Trong mt dy v hn cc s nguyn dng, mi mt s hng sau ln
hn s hng trc hoc l 54 hoc l 77. Chng minh rng trong dy ny tn
ti s hng c hai ch s tn cng ging nhau.

33

1.6. Bi tp

Bi 1.4 (Sc-Slovakia 1997). Chng minh rng tn ti dy s tng {an }


n=1
cc s nguyn dng sao cho vi mi s t nhin k, dy {k + an } cha hu hn
s nguyn t.
Hng dn: Dng nh l Trung hoa v s d.
Bi 1.5 (Putnam 1995). t S() = {[n]|n = 1, 2, 3, . . .}. Chng minh
rng tp hp cc s nguyn dng N khng th phn hoch thnh 3 tp hp
S(), S(), S().
Bi 1.6 (Putnam 1999). Dy s {an }n=1 c xc nh bi a1 = 1, a2 = 2, a3 =
24 v vi n 4.
an = (6a2n1 an3 8an1 a2n2 )/an2 an3
Chng minh rng vi mi n, an l s nguyn chia ht cho n.
Bi 1.7. Trong dy s nguyn dng {ak }k=1 tng ca 10 s hng u tin bng
100, cn t a11 , mi an bng s cc ch s i < n sao cho ai + i n. Bit rng
a11 = 10. Chng minh rng k t mt ch s no , tt c cc s hng ca dy
bng nhau.
Bi 1.8 (Balkan). Cho x0 x1 x2 xn l dy s khng gim
cc s t nhin sao cho vi mi s t nhin k, s cc s ca dy ny khng vt
qu k l hu hn (v k hiu l yk ). Chng minh rng vi mi m, n
n
X
0

xi +

m
X

yi (n + 1)(m + 1)

Bi 1.9 (Bulgari 87). Xt dy s {xn } xc nh bi x1 = x2 = 1, xn+2 =


14xn+1 xn 4. Chng minh rng vi mi n, xn l bnh phng ca mt s
nguyn.
Hng dn: Xt dy u1 = u2 = 1, un+2 = 4un+1 un . Chng minh rng
un+2 un u2n+1 = 2 sau chng minh rng xn = u2n . C th dng tng bi
ny xy dng cc bi ton khc nh th no?
Bi 1.10 (Canada 1988). Cho hai dy s {xn }, {yn} xc nh bi xn+1 =
4xn xn1 , x0 = 0, x1 = 1 v yn+1 = 4yn yn1 , y0 = 1, y1 = 2. Chng minh
rng vi mi n, yn2 = 3x2n + 1.
Bi 1.11 (Canada 1993). Cho y1 , y2, y3, . . . l dy s xc nh bi y1 = 1 v
vi mi s nguyn dng k
y4k = 2y2k , y4k+1 = 2y2k + 1, y4k+2 = 2y2k+1 + 1, y4k+3 = 2y2k+1
Chng minh rng dy s y1 , y2, y3 . . . nhn tt c cc gi tr nguyn dng, mi
gi tr ng mt ln.

34

1.6. Bi tp

Bi 1.12. Gi s rng sn l dy s nguyn dng tho mn iu kin 0


sn+m sn sm K vi K l mt s nguyn dng cho trc. Vi s nguyn
dng N c tn ti cc s thc a1 , a2, . . . , aK sao cho
sn = [a1n] + + [aK n] vi mi n = 1, 2, ...N?
Bi 1.13. Cho a1 = 1, b1 = 2, c1 = 3. Gi S(n) l tp hp cc s nguyn dng
ai , bi, ci vi i n. Xy dng an , bn, cn nh sau:
an+1 = s nguyn dng nh nht khng thuc S(n);
bn+1 = s nguyn dng nh nht khng thuc S(n) v khc an+1 ;
cn+1 = an+1 + bn+1 ;
Gi dk l dy tng cc ch s n sao cho bn = an + 2. Chng minh rng
a) dk /k 6 khi k dn n v cng
b) Nu B l s nguyn th (dk 6k)/2 = B vi v s cc ch s k.
Bi 1.14 (AMM). Cc dy s {an }, {bn}, {cn} c xc nh nh sau: a1 =
1, b1 = 2, c1 = 4 v
an = s nguyn dng nh nht khng thuc {a1, . . . , an1 , b1, . . . , bn1, c1, . . . , cn1}
bn = s nguyn dng nh nht khng thuc {a1 , . . ., an1 , an , b1, .
. . , bn1, c1, . . . , cn1}
cn = 2bn +nan . Hy chng minh hoc ph nh rng 0 < n(1+ 3)bn < 2
vi mi n.

Bi 1.15 (AMM). Cho a1 = 1 v an+1 = an + [ an ] vi n = 1, 2, . . . Chng


minh rng an l s chnh phng khi v ch khi n = 2k + k 2 vi k l s nguyn
dng no .
Bi 1.16 (Bulgari 1973). Cho dy s {an }n=1 c xc nh bi a1 = 2, an+1 =
a2n an + 1.
a) Chng minh rng (an , am) = 1 vi mi m 6= n.
P
b) Chng minh rng lim n1 1/ak = 1.
Hng dn:
a) am 1 = am1 . . . an (an 1)
b) 1/ak = 1/(ak 1) 1/(ak+1 1)
Bi 1.17 (Ba Lan 2002). Cho trc s nguyn dng k. Dy s {an } c xc
nh bi a1 = k + 1, an+1 = a2n kan + k vi mi n 1. Chng minh rng vi
mi m 6= n ta c (am , an) = 1.
Bi 1.18 (KVANT). Cho 1 a0 < a1 < < an l cc s nguyn dng.
Chng minh rng
1/[a0, a1] + 1/[a1, a2] + + 1/[an1 , an] 1 1/2n

35

1.6. Bi tp

Hng dn: Vi a < b, 1/[a, b] = (a, b)/ab (b a)/ab = 1/a 1/b.


Bi 1.19 (Ba Lan 1997). Dy s a1 , a2, . . . xc nh bi
a1 = 0, an = a[n/2] + (1)n(n+1)/2
Vi mi s t nhin k, tm s cc ch s n sao cho 2k n < 2k+1 v an = 0.
Hng dn: Dng h m c s.
Bi 1.20 (Vit Nam, 1998). Cho dy s {an } c xc nh bi a0 = 20, a1 =
100, an+2 = 4an+1 + 5an + 20 vi n = 0, 1, 2, . . . Tm s nguyn dng h nh nht
tho mn iu kin an+h an chia ht cho 1998 vi mi n = 0, 1, 2, . . .
Bi 1.21 (Chn i tuyn VN, 1993). Gi (n) l hm Euler (ngha l (n)
l s cc c s nguyn dng khng ln hn b v nguyn t cng nhau vi n).
Tm tt c cc s nguyn dng k > 1 tho mn iu kin:
Vi a l s nguyn >1 bt k, t x0 = a, xn+1 = k(xn ) vi n = 0, 1, . . . th
(xn ) lun b chn.
Bi 1.22 (M 1997). Cho dy s t nhin a1, a2, . . . , a1997 tho
ai + aj ai+j ai + aj + 1
vi mi i, j nguyn dng tho i + j 1997. Chng minh rng tn ti s thc x
sao cho an = [nx] vi mi n = 1, 2, ..., 1997.
Hng dn: Chng minh rng an /n < (am + 1)/m vi mi m, n.
Bi 1.23. Cho dy s {an }
a) [Lin X 1977] Chng minh rng nu lim(an+1 an /2) = 0 th lim an = 0.
b) Tm tt c cc gi tr a sao cho nu lim(an+1 an ) = 0 th lim an = 0.
Bi 1.24 (CRUX). Tm s hng tng qut ca dy s {pn } xc nh bi p0 =
1, pn+1 = 5pn (5p4n 5p2n + 1)

Bi 1.25. Dy s {an } c xc nh bi a1 > 0, a2 > 0 v an+1 = an + an1 .


Chng minh dy s {an } hi t v tm gii hn.
Bi 1.26 (LMO 1989). Dy s thc {ak }k=1 tho mn iu kin ak+1 = (kak +
1)/(k ak ). Chng minh rng dy s cha v hn s hng dng v v hn s
hng m.
Bi 1.27 (LMO 1989). Dy s thc {ak }k=1 tho mn iu kin |am + an
am+n | 1/(m + n) vi mi m, n. Chng minh rng {ak } l cp s cng.

36

1.6. Bi tp

Bi 1.28. Vi n 2, gi xn l nghim dng duy nht ca phng trnh xn =


xn1 + xn2 + + x + 1.
a) Chng minh rng lim xn = 2.
b) Hy tm lim(2 xn )1/n.
Bi 1.29 (Bulgari 82). Cho x1 , . . . , xn l cc s thc thuc on [0, 2]. Chng
minh rng
n
n X
X
|xi xj | n2 .
i=1 j=1

Du bng xy ra khi no?


Hng dn: Sp li th t!
Bi 1.30 (Bulgari 86). Cho dy s thc {an }
n=1 tho mn iu kin an+1
(1 + k/n)an 1, n = 1, 2, . . . trong 0 < k < 1. Chng minh rng tn ti s t
nhin t sao cho at < 0.
Hng dn: an+1 /(n + 1) < an /n 1/(n + 1).
Bi 1.31. Hai dy s {an }, {bn} xc nh bi a1 > 0, b1 > 0, an+1 = an +
1/bn , bn+1 = bn + 1/an . Chng minh rng a50 + b50 > 20.
Hng dn: Xt cn = (an + bn )2.
Bi 1.32 (Canada 1985). Cho 1 < x1 < 2. Vi n = 1, 2, . . . tanh ngha
xn+1 = 1 + xn x2n /2. Chng minh rng vi mi n 3 ta c |xn 2| < 1/2n.
Bi 1.33
a, b > 0. Hai dy s {an }, {bn} xc nh bi
(PARABOLA). Cho
a1 = ab, b1 = (a + b)/2, an+1 = an bn , bn+1 = (an + bn )/2. Chng minh rng
vi mi n nguyn dng ta c |bn an | |b a|/2n .
Bi 1.34 (IMO 1978). Cho {an } l dy cc s nguyn dng phn bit. Chng
minh rng vi mi n ta c
n
X
k=1

ak /k2

n
X

1/k.

k=1

Bi 1.35 (Putnam 2001). Gi s {an }n=1 l dy s tng cc s thc dng


sao cho lim an /n = 0. C th tn ti v s cc s nguyn dng n sao cho
ani + an+i < 2an vi mi i = 1, 2, . . . , n 1 hay khng?
Bi 1.36 (o - Ba Lan 2001). Cho a1 , a2, . . . , a2010 l dy s tho mn iu
kin

37

1.6. Bi tp

1. Tng 20 s hng lin tip ca dy s l khng m.


2. |ai ai+1 | 1 vi mi i = 1, 2, . . ., 2009.
P
Hy tm min 2001
i=1 ai .
Bi 1.37 (Ba Lan 2001). Cho dy s {an } xc nh bi a0 = 1, an = a[7n/9] +
a[n/9] , n = 1, 2, . . . Chng minh rng tn ti k sao cho ak < k/2001!.
Bi 1.38 (Trung Quc 1997). Cho a1 , a2, . . . l dy s thc tho mn iu
kin an+m an + am vi mi m, n. Chng minh rng an ma1 + (n/m 1)am
vi mi n m.
Bi 1.39 (Singapore 1997). Cho dy s {an } xc nh bi a0 = 1/2, ak+1 =
ak + a2k /n, k = 1, 2, . . ., n 1. Chng minh rng 1 1/n < an < 1.
Hng dn: Chng minh bng quy np rng (n + 1)/(2n k + 2) < ak <
n/(2n k).
Bi 1.40 (Baltic Way). Gi s a1 , a2, . . . , a9 l cc s khng m sao cho a1 =
a9 = 0 v t nht c mt s khc 0. Chng minh rng tn ti ch s i, 2 i 8
sao cho ai1 + ai+1 < 2ai . Khng nh c cn ng khng nu thay 2 bt ng
thc cui cng bng 1.9?
Bi 1.41. Dy s an c xc nh bi cng thc truy hi
a0 = 1, an+1 =

an
, n = 0, 1, 2, . . .
1 + nan

Hy tm cng thc tng qut cho an .


Bi 1.42 (Vit Nam, 1984). Dy s u1 , u2, . . . c xc nh bi: u1 = 1, u2 =
P
2, un+1 = 3un un1 vi n = 2, 3, . . . t vn = 1kn arcotguk .
Hy tm gii hn vn khi n dn n v cng.
Hng dn: Dng sai phn.
Bi 1.43 (PTNK, 1999). Cho a > 1 v dy s {xn } c xc nh nh sau
x1 = a, xn+1 = nax vi mi n 1.
Hy xc nh tt c cc gi tr ca a dy {xn } hi t.

38

1.6. Bi tp

Bi 1.44. Cho dy s dng {an }. Bit rng tn ti gii hn


n
X
1
=A<
n
ak

lim

k=1

t sn = a1 + a2 + + an . Chng minh rng tng


n
X
k 2 ak
(sk )2
k=1

cng c gii hn hu hn khi n .


Hng dn: Dng cng thc tnh tng tng phn
Bi 1.45. Cho f : N R tho iu kin f (a+b) f (a)+f (b) vi mi |ba| k
(k l s nguyn dng c nh). Hi c tn ti gii hn f (n)/n khi n dn n v
cng khng?
Bi 1.46. Cc phn t ca dy s a1 , a2, a3, . . ., l cc s nguyn dng khc
nhau. Chng minh rng vi mi k tn ti n sao cho tn ti an n.
Bi 1.47. Chng minh rng nu a1 > 2 v an = a2n1 2 th
q
1
1
1
1
+
+
+ = [a1 a21 4].
a1 a1 a2 a1a2a3
2
Hng dn: Dng lng gic.
Bi 1.48.P
Dy s dng an tho mn iu kin an < an+1 + a2n . C th khng
nh tng ni=1 ai dn n v cng khi n dn n v cng hay khng?
Bi 1.49 (THTT). Cho s thc r > 2. Cho dy s thc dng {an } tho mn
iu kin arn = a1 + + an1 vi mi n 2. Chng minh rng dy {an /n} c
gii hn hu hn khi n v tm gii hn .
Bi 1.50 (Chn i tuyn Vit Nam, 1985). Dy s thc {xn } c xc
nh bi:
p

x1 = 29/10, xn+1 = (xn / x2n 1) + 3, n = 1, 2, 3 . . .


Hy tm s thc nh hn x2k1 v ln hn x2k vi mi k = 1, 2, . . .
Bi 1.51 (Chn i tuyn Vit Nam, 1996). Tm tt c cc gi tr ca a
dy s {xn } c xc nh bi

x0 = 1996
xn+1 = a/(1 + x2n )
c gii hn hu hn khi n dn ti v cng.

39

1.6. Bi tp

Hng dn: Chuyn v dng xn+1 = f (xn ), x0 = b.


Bi 1.52 (Vit Nam, 1997). Cho n l s nguyn >1, khng chia ht cho 1997.
t
ai = i + ni/1997 vi mi i = 1, 2, . . ., 1996,
bj = j + 1997j/n vi mi j = 1, 2, . . ., n 1.
Ta sp xp cc s {ai } v {bi} theo th t tng dn:
c1 c2 c1995+n
Chng minh rng ck+1 ck < 2 vi mi k = 1, 2, ..1994 + n.
Bi 1.53 (Vit Nam, 1998). Cho a l mt s thc khng nh hn 1. t
x1 = a, xn+1 = 1 + ln(x2n /(1 + ln(xn )) vi n = 1, 2, . . .
Chng minh rng dy s {xn } c gii hn v tm gii hn .
Bi 1.54. Cho dy s {xn } xc nh bi, x1 = a, xn+1 = (2x3n )/(3x2n 1) vi mi
n 1. Tm tt c cc gi tr ca a dy s xc nh v c gii hn hu hn.
Bi 1.55. Chng minh rng dy s xc nh bi iu kin xn+1 = xn + x2n /n2
vi n 1, trong 0 < x1 < 1 l dy b chn.
Bi 1.56. Cho dy s
s
an =

1+2 1+

1 + (n 1) 1 + n

Chng minh rng limn an = 3.


Bi 1.57. Dy a1 + 2a2, a2 + 2a3, a3 + 2a4 , . . . hi t. Chng minh rng dy
a1 , a2, a3, . . . cng hi t.
Bi 1.58. Cho dy A(n), n = 1, 2, . . . tho mn: vi mi x thc th limn A([xn ]) =
0. Chng minh rng lim A(n) = 0 khi n tin ti v cng.
Bi 1.59. Cho hm s
f (x) = x + A sin x + B cos x vi A2 + B 2 < 1.
Xt dy s
a0 = a, a1 = f (a0 ), . . ., an+1 = f (an ), . . .
Chng minh rng vi mi a, dy s {an } c gii hn v hy tm gii hn .

1.6. Bi tp

40

Bi 1.60. Cho dy s {an }, c xc nh nh sau: a0 = a, a1 = b, an+1 =


an + (an an1 )/2n. Tm limn an .
Bi 1.61 (AMM). Cho {Hn } l dy s Fibonacci tng qut, tc l H1 , H2 l
cc s nguyn bt k v vi n > 2 th Hn = Hn1 + Hn2 .
a) Hy tm T , ph thuc vo H1 v H2 sao cho cc s H2n H2n+2 +T, H2nH2n+4 +
T, H2n1H2n+1 T, H2n1H2n+3 T u l cc s chnh phng.
b) Chng minh T l duy nht.
Bi 1.62. Cho r l s thc. Xc nh dy s {xn } bi x0 = 0, x1 = 1, xn+2 =
rxn+1 xn vi n 0. Chng minh rng x1 + x3 + + x2m1 = x2m .
Bi 1.63 (IMO 1977). Trong mt dy s hu hn cc s thc, tng 7 s hng
lin tip ca dy lun m, cn tng 11 s hng lin tip lun dng. Hi dy s
c th c nhiu nht bao nhiu s hng.
Ti liu tham kho
1. Jean-Marie Monier, Gii tch 1, 2, 3, 4, NXBGD 1999-2000.
2. L Hi Chu: Tuyn tp cc thi ton quc t.
3. Titu Andreescu, Razvan Gelca: Mathematical Olympiad Challenges, Birkhauser
2000.
4. A. Gardiner, The Mathematical Olympiad Hanbook, Oxford, 1997.
5. Titu Andreescu, Zuming Feng: Mathematical Olympiads 1998-1999, 19992000, 2000-2001, MAA, 2000-2002.
6. Arthur Engel: Problem-Solving Strategies, Springer 1997. 7
7. G.Polya, G.Szego: Cc bi tp v nh l ca gii tch, Nauka 1977 (Ting
Nga).
8. Cupsov, Nesterenko . . .: Thi v ch ton ton Lin X, Prosvesenie, 1999
(Ting Nga).
9. 400 bi ton t American Mathematical monthly, Mir, 1977 (Ting Nga).
10. thi ton ca Vit Nam, cc nc v khu vc.
11. Tp ch Ton hc v Tui tr (THTT), Parabola, Kvant, American Mathematical monthly (AMM).
Trn Nam Dng - HKHTN TP H Ch Minh
227 Nguyn Vn C, Qun 5, TP H Ch Minh
Email: namdung@fpt.com.vn, namdung@fsoft.com.vn

Chng 2

Phng trnh sai phn


2.1
2.1.1

Sai phn
nh ngha

Cho hm s y = f (x) xc nh trn R, t xk = x0 + kh (k N ) vi x0


R, h R, bt k, cho trc. Gi yk = f (xk ) l gi tr ca hm s f (x) ti x = xk .
Khi , Hiu s yk := yk+1 yk (k N) c gi l sai phn cp 1 ca hm s
f (x). Hiu s 2 yk := yk+1 yk = (yk ) (k N ) c gi l sai phn cp
2 ca hm s f (x). Tng qut, i yk := i1 yk+1 i1 yk = (i1 yk ) (k
N ) c gi l sai phn cp i ca hm s f (x) (i = 1; 2; ; n; ).

2.1.2

Tnh cht

Mnh 2.1 (Biu din sai phn theo gi tr ca hm s). Sai phn mi cp
u c th biu din theo cc gi tr ca hm s:
y 0 ; y1 ; y2 ; ; y n ;
Chng minh. Tht vy, ta c
yk = yk+1 yk
2yk = yk+1 yk
= yk+2 yk+1 (yk+1 yk )
= yk+2 2yk+1 + yk .
Tng t, bng quy np ta c th chng minh c.
i
X
yk =
(1)s Cis yk+is ,
i

s=1

41

(pcm).

42

2.1. Sai phn

Mnh 2.2 (Sai phn ca hng s). Sai phn ca hng s bng 0.
Chng minh. Tht vy, vi y = f (x) = C = const ta c: f (x) = C C = 0.
Hn th na, sai phn mi cp ca hng s u bng 0.
Mnh 2.3 (Tnh cht tuyn tnh ca sai phn ). Sai phn mi cp l mt
ton t tuyn tnh trn tp cc hm s. Tc l.
i N , ; R, f (x); g(x) : R R, ta lun c:
i (f (x) + g(x)) = i f (x) + i g(x).
Chng minh. Tht vy, t fk = f (xk ) ; gk = g(xk ), ta thu c
i

(fk + gk ) =

i
X

(1)s Cis [fk+is + gk+is ]

s=0

i
X
s=0
i

(1)sCis fk+is +

i
X
(1)s Cis gk+is
s=0

= fk + gk .
Vy nn
i (f (x) + g(x)) = i f (x) + i g(x) vi mi i N (pcm).
Mnh 2.4 (Sai phn ca a thc). Sai phn cp i ca mt a thc bc n.
+) L mt a thc bc n i khi i < n. +) L hng s khi i = n. +) Bng 0 khi
i > n.
Chng minh. Do sai phn mi cp l ton t tuyn tnh nn ta ch cn chng
minh tnh cht cho a thc y = Pn (x) = xn .
+) Khi i < n ta c
1o ) Vi i = 1 th: xn = (x + h)n xn = Pn1 (x) l a thc bc n 1 i
vi x. Vy khng nh ng vi i = 1.
2o ) Gi s khng nh ng vi i = k < n, tc l k xn = Pnk (x) l a thc
bc n k i vi x. Khi
k+1 xn = (k xn ) = k ((x + h)n ) k (xn )
= Pnk (x + h) Pnk (x) = Pnk1 (x)
l a thc bc n k 1 = n (k + 1) i vi x.
Vy khng nh cng ng vi i = k + 1. T , theo nguyn l quy np ton
hc suy ra khng nh ng vi mi i N (pcm).

43

2.2. Phng trnh sai phn tuyn tnh

+) Khi i = n th theo trn, n (xn ) l a thc cp n n = 0 i vi x nn l


hng s.
+) Khi i > n th
i (xn ) = in (n (xn )) = in C

(C = const) = 0.

Vy tnh cht c chng minh trn vn.


Mnh 2.5 (Cng thc sai phn tng phn).
(fk gk ) = fk gk + gk+1 fk .
Chng minh. Ta c
(fk gk ) = fk+1 gk+1 fk gk
= fk+1 gk+1 fk gk+1 + fk gk+1 fk gk
= gk+1 (fk+1 fk ) + fk (gk+1 gk )
= fk gk + gk+1 fk .
Mnh 2.6 (Tng cc sai phn).
n
X

yk = yn+1 y1 .

k=1

Chng minh.
n
X

yk = y1 + y2 + + yn1 + yn

k=1

= y2 y1 + y3 y2 + + yn yn1 + yn+1 yn
= yn+1 y1 .

2.2
2.2.1

Phng trnh sai phn tuyn tnh


Mt s khi nim chung v phng trnh sai phn

nh ngha 2.1. Phng trnh sai phn (cp k) l mt h thc tuyn tnh cha
sai phn cc cp ti k.
f (yn ; yn; 2yn ; ; k yn ) = 0.

(1)

2.3. Phng trnh sai phn tuyn tnh bc nht

44

V sai phn cc cp u c th biu din theo gi tr ca hm s nn (1) c


dng:
a0 yn+k + a1yn+k1 + + ak yn = f (n).
(2)
trong a0 ; a1; ; ak , f (n) bit, cn yn , yn+1 , , yn+k l cc gi tr cha
bit.
Phng trnh (2) c gi l phng trnh sai phn tuyn tnh cp k.
Nu f (n) = 0 th phng trnh (2) c dng
a0 yn+k + a1 yn+k1 + + ak yn = 0.

(3)

v c gi l phng trnh sai phn tuyn tnh thun nht cp k.


Nu f (n) 6= 0 th (2) c gi l phng trnh sai phn tuyn tnh khng
thun nht.
b. Nghim.
Hm s yn bin n tho mn (2) c gi l nghim ca phng trnh sai
phn tuyn tnh (2).
Hm s yn ph thuc k tham s tho mn (3) c gi l nghim tng qut
ca (3).
Mt nghim yn tho mn (2) c gi l mt nghim ring ca (2).

2.3
2.3.1

Phng trnh sai phn tuyn tnh bc nht


nh ngha

Phng trnh sai phn tuyn tnh bc nht (cp mt) l phng trnh sai
phn dng:
u1 = , aun+1 + bun = f (n)
n N
(1)
trong ; a 6= 0 ; b 6= 0 l cc hng s v f (n) l biu thc ca n cho trc.

2.3.2

Phng php gii

A. Gii phng trnh sai phn thun nht tng ng.


1+ ) Gii phng trnh c trng: a + b = 0 tm .
2+ ) Tm nghim ca phng trnh sai phn tuyn tnh thun nht tng
ng: aun+1 + bun = 0 di dng u
n = cn (c l hng s ).

B. Tm mt nghim ring un ca phng trnh khng thun nht.


C. Tm nghim tng qut ca phng trnh (1):
un = un + u
n .

2.3. Phng trnh sai phn tuyn tnh bc nht

45

1
V d 2.1. Phng trnh un+1 = 3un +1 c u
n = C.3n ; un = nn c nghim
2
1
tng qut l un = C.3n vi C l hng s bt k.
2
Sau y ta trnh by phng php tm nghim ring.

2.3.3

Phng php tm nghim ring ca phng trnh sai phn


tuyn tnh cp 1 khng thun nht khi v phi f (n) c dng
c bit

Trng hp 1. Nu f (n) = Pm (n) l a thc bc m i vi n. Khi : +)


Nu 6= 1 th ta chn un = Qm (n) cng l a thc bc m i vi n. +) Nu
= 1 th ta chn un = nQm (n) trong Qm (n) cng l a thc bc m i vi
n.
V d 2.2. Gii phng trnh sai phn:
(
x0 = 7
xn+1 = 15xn 14n + 1
Gii. Ta c f (n) = 14n+1 l a thc bc nht, = 15 6= 1 chn xn = an+b.
Thay vo phng trnh ta c
a(n + 1) + b = 15(an + b) 14n + 1.
Suy ra a = 1 ; b = 0. Vy xn = n cn x
n = C.15n v nghim tng qut
l: xn = C.15n + n. M x0 = 7 nn C = 7. Vy phng trnh c nghim:
xn = 7.15n + n.
(
x0 = 99
V d 2.3. Gii phng trnh sai phn:
xn+1 = xn 2n 1
Gii. f (n) = 2n 1 l a thc bc nht, = 1 chn xn = n(an + b). Thay
vo (1.2) c:
(n + 1)[a(n + 1) + b] = n(an + b) 2n 1 a = 1 ; b = 0 xn = n2 .
Cn x
n = C.1n = C xn = C n2 , m x0 = 99 C 02 = 99 C = 99. Vy
phng trnh (1.2) c nghim: xn = 99 n2 .
Trng hp 2. f (n) = p. n (p; 6= 0). Khi :
+) Nu 6= th ta chn xn = d. n (d R).
+) Nu = th ta chn xn = d.n. n (d R).

46

2.3. Phng trnh sai phn tuyn tnh bc nht

V d 2.4. Gii phng trnh sai phn:

(
x0 = 8
xn+1 = 2xn + 3n

(1.3)

Gii. Do = 2 6= 3 = nn ta chn xn = d.3n. Thay vo phng trnh (1.3)


c d = 1 xn = 3n . Cn x
n = C.2n . Vy xn = C.2n + 3n . Thay vo iu kin
bin c C = 7.
Tr li: phng trnh cho c nghim xn = 7.2n + 3n .
(
x0 = 101
V d 2.5. Gii phng trnh sai phn:
.
xn+1 = 7.xn + 7n+1 (1.4)
Gii. Do = 7 = nn ta chn xn = d.n.3n. Thay vo phng trnh (1.4) c
d = 1 xn = n.7n . Cn x
n = C.7n . Vy xn = C.7n + n.7n . Thay vo iu kin
bin c C = 101.
Tr li: phng trnh cho c nghim xn = (101 + n).7n .
Trng hp 3. f (n) = . sin nx + . cos nx ( + 6= 0; x 6= k; k Z). Khi
, ta chn un = A. sin nx + B. cos nx vi A; B R l cc hng s.
(
x0 = 1

V d 2.6. Gii phng trnh sai phn:


.
n
2.xn+1 = xn sin
(1.5)
4
1
n
n
n
Gii. C = ; f (n) = sin
nn ta chn xn = A. cos
+ B. sin
.
4
4
4
2
Thay xn vo (1.5), bin i v so snh cc h s ta c A = 1 ; B = 0 xn =
n
1
1
n
cos
. Cn x
n = C.( )n xn = C.( )n + cos
. Thay vo iu kin bin
4
4
2
2
n
x0 = 1 ta c C = 0. Vy phng trnh cho c nghim xn = cos
.
4
Trng hp 4.
m
X
f (n) =
fk (n).
k=1

Khi ta chn nghim ring


ca (1) di dng: xn = m
k=1 xnk trong xnk
tng ng l nghim ring ca phng trnh sai phn (1) vi V P = fk (n).
(
x0 = 17
V d 2.7. Gii phng trnh sai phn:
xn+1 = 2xn n2 + 2n + 1 + 6.2n (1.6)
xn

Gii. C = 2 ; f1 (n) = n2 + 2n + 1 ; f2 (n) = 6.2n


x
n = C.2n ; xn1 = an2 + bn + c ; xn2 = d.n.2n.

47

2.4. Phng trnh sai phn tuyn tnh cp 2

Vy ta chn xn = an2 + bn + c + d.n.2n. Thay vo (1.6) v so snh cc h s c:


a = 1 ; b = c = 0 ; d = 3. Vy: xn = C.2n + n2 + 3n.2n . Thay vo iu kin bin
x0 = 17 ta c C = 17 v do nghim ca phng trnh sai phn cho l:
xn = 17.2n + n2 + 3n.2n .

2.3.4

Bi tp

Gii cc phng trnh sai phn tuyn tnh sau:


1.

un+1 = 3un 6n + 1 ; u1 = 1.
p s: un = 3n + 1 3n .

2.

un+1 = un + 2n2 ; u1 = 1.
p s: un =

3.

un+1 = 5un 3n ; u0 = 1.
p s: un =

1
(2n3 3n2 + n + 3).
3
1 n
(5 + 3n ).
2

4.

un+1 = 2un + 6.2n ; u0 = 1.


p s: un = (3n + 1).2n.

5.

un+1 = un + 2n.3n ; u0 = 0.
p s: un =

1
[(2n 3).3n + 3].
2

6.

un+1 2un = (n2 + 1).2n ; u0 = 1.



n(2n2 3n + 7)
p s: un =
+ 2 .2n .
6

7.

un+1 2un = n + 3n ; u0 = 1.
p s: un = 2n + 3n n 1.

2.4
2.4.1

Phng trnh sai phn tuyn tnh cp 2


nh ngha

Phng trnh sai phn tuyn tnh cp hai l phng trnh sai phn dng:
u1 = , u2 = , aun+2 + bun+1 + cun = f (n),

n N

(1)

trong a, b, c, , l cc hng s, a 6= 0, c 6= 0 v f (n) l biu thc cha n


cho trc.

48

2.4. Phng trnh sai phn tuyn tnh cp 2

2.4.2

Cch gii

+) Gii phng trnh thun nht tng ng. +) Tm nghim ring ca phng
trnh khng thun nht. +) Tm nghim tng qut ca phng trnh (1) di
dng:
un = u
n + un .
A- Gii phng trnh thun nht tng ng
aun+2 + bun+1 + cun = 0

(2)

1+ ) Gii phng trnh c trng:


a.2 + b. + c = 0. (3) tm .
2+ ) Tm nghim tng qut ca phng trnh thun nht tng ng.
Trng hp 1: Nu (3) c hai nghim phn bit: = 1 ; = 2 th:
u
n = A.n1 + B.n2 ,
trong A v B c xc nh khi bit u1 v u2.
Trng hp 2: Nu (3) c nghim kp: 1 = 2 = th:
u
n = (A + Bn).n ,
trong A v B c xc nh khi bit u1 v u2.
Trng hp 3: Nu l nghim phc, = x + i.y th ta t
r = || =

x2 + y 2 , tan =

 
y
, ;
.
x
2 2

lc = r(cos + i. sin ) v
u
n = rn (A. cos n + B. sin n),
trong A v B c xc nh khi bit u1 v u2. V d
(
x0 = 2 ; x1 = 8
V d 2.8. Gii phng trnh sai phn:
xn+2 = 8xn+1 + 9xn

49

2.4. Phng trnh sai phn tuyn tnh cp 2

Gii. Phng trnh c trng:


2 + 8 9 = 0 = 1 hoc = 9.

(Hai nghim phn bit.)

Vy: x
n = xn = A.1n + B.(9)n = A + B.(9)n .
Gii iu kin bin:
(
(
(
x0 = 2
A+B = 2
A=1

x1 = 8
A 9B = 8
B=1

Vy phng trnh cho c nghim: xn = 1 + (9)n .


(
x0 = 1 ; x1 = 16
V d 2.9. Gii phng trnh sai phn:
xn+2 = 8xn+1 16xn

Gii. Phng trnh c trng:


2 8 + 16 = 0

1 = 2 = 4 (c nghim kp).

Vy: x
n = xn = (A + Bn).4n .
Gii iu kin bin:
(
(
x0 = 1
A=1

x1 = 16
(A + B).4 = 16

A=1
B=3

Vy phng trnh cho c nghim: xn = (1 + 3n).4n.

x = 1 ; x = 1
0
1
V d 2.10. Gii phng trnh sai phn:
2
x
n+2 = xn+1 x n

Gii. Phng trnh c trng:

1+i 3
1i 3
2
+1=0=
hoc =
. (Hai nghim phc.)
2
2
C:

1+i 3

=
= cos + i. sin r = 1 ; = .
2
3
3
3
n
n
Nghim tng qut: x
n = xn = A. cos
+ B. sin
.
3
3
Gii iu kin bin:

(
x0 = 1
A = 1
A=1
1

1
x1 =
A. cos + B. sin =
B=0
2
3
3
2

50

2.4. Phng trnh sai phn tuyn tnh cp 2

Vy phng trnh cho c nghim: xn = cos

n
. Bi tp Gii cc phng trnh
3

sai phn sau:


1. yn+2 = 4yn+1 3yn
2. yn+2 = 8yn+1 16yn
3. yn+2 = 2yn+1 2yn

vi
vi
vi

y0 = 1 ; y1 = 1.
y0 = 1 ; y1 = 1.
y0 =

1
; y1 = 2.
2

B- Cc phng php tm nghim ring ca phng trnh sai phn tuyn tnh cp
hai khng thun nht
aun+2 + bun+1 + cun = f (n)
vi v phi c dng c bit Trng hp 1.
f (n) = Pk (n) l a thc bc k i vi n.
Khi : +) Nu phng trnh c trng (3) khng c nghim = 1 th ta chn
xn = Qk (n)
trong Qk (n) l a thc bc k no i vi n. +) Nu phng trnh c trng
(3) c nghim n = 1 th ta chn
xn = nQk (n)
trong Qk (n) l a thc bc k no i vi n. +) Nu phng trnh c trng
(3) c nghim kp = 1 th ta chn
xn = n2 Qk (n)
trong Qk (n) l a thc bc k no i vi n.
V d 2.11. Tm mt nghim ring xn ca phng trnh sai phn:
xn+2 = 4xn+1 + 5xn + 12n + 8.
Gii. Phng trnh c trng:
2 + 4 5 = 0 = 1 hoc = 5 ; f (n) = 12n + 8.
Chn xn = n(an + b). Thay vo phng trnh cho v so snh cc h s ta c
a = 1 ; b = 0.
Vy phng trnh cho c mt nghim ring l xn = n2 .

51

2.4. Phng trnh sai phn tuyn tnh cp 2

V d 2.12. Gii phng trnh sai phn:


2xn+2 5xn+1 + 2xn = n2 2n + 3

vi

x0 = 1 ; x1 = 3.

Gii. Phng trnh c trng:


22 5 + 2 = 0 = 2 hoc =

1
; f (n) = n2 2n + 3.
2

1
Vy phng trnh thun nht c nghim tng qut x
n = A.2n + B.( )n .
2
Chn xn = an2 + bn + c. Thay vo phng trnh cho v so snh cc h s ta
c a = 1 ; b = 4 ; c = 10.
Vy phng trnh cho c mt nghim ring l xn = n2 + 4n 10.
Do phng trnh cho c nghim tng qut l:
1
xn = A.2n + B.( )n n2 + 4n 10.
2
Thay vo cc iu kin bin ta tm c A = 3 ; B = 8. Vy nghim ca phng
trnh cho l:
1
xn = 3.2n + 8.( )n n2 + 4n 10.
2
Trng hp 2.
f (n) = Pk (n). n

trong Pk (n) l mt a thc bc k i vi n.

Khi : +) Nu khng phi l nghim ca phng trnh c trng (3) th ta


chn:
xn = Qk (n)
trong Qk (n) l mt a thc bc k no i vi n vi h s cn c xc
nh. +) Nu l mt nghim n ca phng trnh c trng (3) th ta chn:
xn = n.Qk (n)
trong Qk (n) l mt a thc bc k no i vi n . +) Nu l nghim kp
ca phng trnh c trng (3) th ta chn:
xn = n2 .Qk (n),
trong Qk (n) l mt a thc bc k no i vi n .
V d 2.13. Tm mt nghim ring ca phng trnh sai phn sau:
2xn+2 + 5xn+1 + 2xn = (35n + 51).3n.

52

2.4. Phng trnh sai phn tuyn tnh cp 2

Gii. Ta c = 3 ; Pk (n) = 35n + 51 l a thc bc nht.


Phng trnh c trng:
22 + 5 + 2 = 0 = 2 := 1 hoc =

1
:= 2
2

(1; 2 6= ).

Chn xn = (an + b).3n . Thay vo phng trnh cho v so snh cc h s ta


c: a = 1 ; b = 0.
Vy phng trnh cho c mt nghim ring l: xn = n.3n .
V d 2.14. Tm mt nghim ring ca phng trnh sai phn sau:
xn+2 5xn+1 + 6xn = (8n + 11).2n.
Gii. Ta c = 2 ; Pk (n) = 8n + 11 l a thc bc nht.
Phng trnh c trng:
2 5 + 6 = 0 = 2 := 1 hoc = 3 := 2

(1 = ; 2 6= ).

Chn xn = n(an + b).2n . Thay vo phng trnh cho v so snh cc h s ta


c: a = 4 ; b = 23.
Vy phng trnh cho c mt nghim ring l: xn = (4n2 + 23n).2n.
V d 2.15. Tm mt nghim ring ca phng trnh sai phn sau:
xn+2 10xn+1 + 25xn = (n + 2).5n+1
Gii. Ta c = 5 ; Pk (n) = 5n + 10 l a thc bc nht.
Phng trnh c trng:
2 10 + 25 = 0 = 5 := 0

(nghim kp) (1 = 2 = )

Chn xn = n2 (an + b).5n. Thay vo phng trnh cho v so snh cc h s ta


4
7
c: a =
; b=
.
50
50
n2
Vy phng trnh cho c mt nghim ring l: xn =
(4n + 7).5n .
50
Trng hp 3.
f (n) = Pm (n). cos n + Pl (n). sin n
trong Pm (n) ; Pl (n) ln lt l cc a thc bc m ; l i vi n.
K hiu k = M ax{m; l} v gi = cos + i sin (i2 = 1). Khi :
+) Nu khng l nghim ca phng trnh c trng (3) th ta chn:
xn = Tk (n). cos n + Rk (n). sin n

53

2.4. Phng trnh sai phn tuyn tnh cp 2

trong , Tk (n) ; Rk (n) l cc a thc bc k i vi n no .


+) Nu l nghim ca phng trnh c trng (3) th ta chn:
xn = nTk (n). cos n + Rk (n). sin n
trong , Tk (n) ; Rk (n) l cc a thc bc k i vi n no .
(
x0 = 1 ; x 1 = 0
V d 2.16. Gii phng trnh sai phn:
n
xn+2 = 2xn+1 xn + sin
2

Gii. Ta c Pm (n) 0 ; Pl (n) 1 ; = = i.


2
Phng trnh c trng: 2 2 + 1 = 0 c nghim kp = 1 (6= i = ).
Nghim tng qut ca phng trnh thun nht : x
n = an + b.
n
n
Nghim ring ca phng trnh cho c dng: xn = c. cos
+ d. sin
.
2
2
Thay xn vo phng trnh cho, rt gn v so snh cc h s ta c:
c=

1
1
n
; d = 0 xn = cos
2
2
2

Phng trnh cho c nghim tng qut l:


xn = an + b +
Gii cc iu kin bin:

x = b + 1 = 1
0
2
x = a + b = 0
1

1
n
cos
2
2

a = 1
2

b =
2

Vy phng trnh cho c nghim:


xn =

1
n
(1 n + cos
)
2
2

V d 2.17. Tm mt nghim ring ca phng trnh sai phn sau:


n
n
+ (3n + 1). sin
.
2
2

Gii. Ta c Pm (n) = n 2 ; Pl (n) = 3n + 1 ; = = i.


2
Phng trnh c trng:
xn+2 3xn+1 + 2xn = (n 2). cos

2 3 + 2 = 0 = 1 := 1 ; = 2 := 2

(1; 2 6= ).

54

2.4. Phng trnh sai phn tuyn tnh cp 2

Nghim ring ca phng trnh cho c dng:


n
n
xn = (an + b). cos
+ (cn + d). sin
.
2
2
Thay xn vo phng trnh cho, rt gn v so snh cc h s ta c:
n
a = 1 ; b = c = d = 0 xn = n. cos
.
2
n
Vy phng trnh cho c mt nghim ring l: xn = n. cos
.
2
Trng hp 4:
m
X
f (n) =
fk (n).
k=1

Khi ta tm nghim ring ca (1) di dng:


xn =

m
X

xnk

k=1

trong xnk l nghim ring ca phng trnh: (1) vi V P = fk (n) v c tm


theo mt trong cc trng hp trn.
V d 2.18. Tm mt nghim ring ca phng trnh sai phn sau:
xn+2 = 5xn+1 6xn + 2n + n 1.
Gii. Phng trnh c trng:
2 5 + 6 = 0 = 2 := 1 hoc = 3 := 2.
f (n) = 2n + n 1 := f1 (n) + f2 (n) vi f1 (n) = 2n ; f2 (n) = n 1.
i vi phng trnh: xn+2 = 5xn+1 6xn + 2n (1.8) do 1 = 2 = nn ta chn
nghim ring: xn1 = an.2n . Thay vo (1.8) ri chia c hai v cho 2n ta c:
2a = 1 a =

1
1
xn1 = .n.2n = n.2n1 .
2
2

i vi phng trnh: xn+2 = 5xn+1 6xn + n 1 (1.9) ta chn nghim ring:


xn1 = cn + d. Thay vo (1.9) ri so snh cc h s hai v ta c:
c=

1
1
1
1
; d = xn2 = .n + .
2
4
2
4

Vy phng trnh cho c mt nghim ring l:


1
1
1
1
xn = xn1 + xn2 = .n.2n + .n + = (1 + 2n n.2n+1 ).
2
2
4
4
Bi tp Tm cc nghim ring ca cc phng trnh sai phn sau:

2.5. Phng trnh sai phn tuyn tnh cp 3

1.

yn+2 5yn+1 + 6yn = 0

2.

8yn+2 6yn+1 + yn = 2n

3.

yn+2 3yn+1 + 2yn = 5n + 2n3 + 3n + 1

4.

yn+2 yn+1 + 2yn = n2

5.

yn+2 + yn = sin

6.

4yn+2 + 4yn+1 + yn = 2

7.

yn+2 2yn+1 + yn = 5 + 3n

8.

yn+2 2yn+1 + yn = 2n (n 1)

9.

yn+2 3yn+1 + 2yn = 3n

10.

2.5
2.5.1

55

n
2

8yn+2 6yn+1 + yn = 5 sin

n
2

Phng trnh sai phn tuyn tnh cp 3


nh ngha

Cho a, b, c, d, , , l cc hng s R ; a 6= 0 ; d 6= 0 cn f (n) l mt hm


s bin s n. Phng trnh:
(
u 1 = ; u2 = ; u3 =
aun+3 + bun+2 + cun+1 + dun = f (n) quad(1)
c gi l phng trnh sai phn tuyn tnh cp (bc) ba.

2.5.2

Phng php gii

Phng trnh sai phn tuyn tnh cp ba lun gii c. Nghim tng qut
ca n c dng:
un = u
n + un
trong , u
n l nghim tng qut ca phng trnh sai phn tuyn tnh thun
nht, cn un l mt nghim ring no ca phng trnh cho. Cch tm u
n
Xt phng trnh c trng:
a3 + b2 + c + d = 0 (2)

56

2.5. Phng trnh sai phn tuyn tnh cp 3

+) Nu (3) c ba nghim thc phn bit: 1 6= 2 6= 3 6= 1 th:


u
n = C1 n1 + C2 n2 + C3 n3
+) Nu (3) c mt nghim thc bi 2 v mt nghim n: 1 = 2 6= 3 th:
u
n = (C1 + C2 n)n1 + C3 n3
+) Nu (3) c mt nghim thc bi 3: 1 = 2 = 3 := 0 th:
u
n = (C1 + C2 n + C3 n2 )n0
+) Nu (3) c mt nghim thc 1 v hai nghim phc lin hp:
2;3 = r(cos i sin ) th
u
n = C1 n1 + rn (C2 cos n + C3 sin n)
Trn y ta k hiu C1 ; C2; C3 l cc hng s m s c xc nh bng cch
thay u
n vo cc iu kin bin v gii h phng trnh thu c. Cch tm un
Trng hp 1. Nu f (n) = Pm (n) l a thc bc m i vi n th: +) Khi
(3) khng c nghim = 1 th ta chn: un = Qm (n) trong , Qm (n) l a thc
bc m i vi n. +) Khi (3) c nghim n = 1 th ta chn: un = nQm (n)
trong , Qm (n) l a thc bc m i vi n. +) Khi (3) c nghim bi hai = 1
th ta chn: un = n2 Qm (n) trong , Qm (n) l a thc bc m i vi n. +) Khi
(3) c nghim bi ba = 1 th ta chn: un = n3 Qm (n) trong , Qm (n) l a
thc bc m i vi n.
Trng hp 2. Nu f (n) = A.n ( A ; l cc hng s cho trc) th: +)
Khi khng l nghim ca (3) th ta chn: un = B.n vi B l hng s c xc
nh bng cch thay un vo phng trnh cho. +) Khi l nghim n ca
(3) th ta chn: un = B.n.n . +) Khi l nghim bi hai ca (3) th ta chn:
un = B.n2 .n . +) Khi l nghim bi ba (3) th ta chn: un = B.n3 .n .

2.5.3

V d

V d 2.19. Gii phng trnh sai phn:

x1 = 0 ; x 2 = 1 ; x 3 = 3
xn = 7xn1 11xn2 + 5xn3

Gii. Phng trnh c trng:


3 72 + 11 5 = 0 : c ba nghim: 1 = 2 = 1 ; 3 = 5.
Vy phng trnh c nghim tng qut
xn = (C1 + C2 n).1n + C3 .5n = C1 + C2 n + C3 .5n .

57

2.5. Phng trnh sai phn tuyn tnh cp 3

Thay vo iu kin bin ta c h phng trnh:

13

C1 =

C
+
C
+
5C
=
0

2
3
16
1

3
C2 =
C1 + 2C2 + 25C3 = 1

C1 + 3C2 + 125C3 = 3

C3 = 1
80

Vy phng trnh cho c nghim:


1 n1
+ 12n 13).
(5
16
(
x0 = 1 ; x 1 = 2 ; x 2 = 3
V d 2.20. Gii phng trnh sai phn:
xn+3 3xn+2 + 3xn+1 xn = 1
xn =

Gii. Phng trnh c trng:


3 32 + 3 1 = 0 : c nghim bi ba: 1 = 2 = 3 = 1.
Vy phng trnh thun nht c nghim tng qut
x
n = (C1 + C2 n + C3 n2 ).1n = C1 + C2 n + C3 n2 .
Do f (n) = 1 = 1.1n nn ta chn nghim ring xn = B.n3 .1n = B.n3 . Thay vo
phng trnh cho ri so snh cc h s ta c
B=

1
1
xn = n3 .
6
6

Vy nghim tng qut ca phng trnh cho l:


1
xn = xn + x
n = C 1 + C 2 n + C 3 n2 + n 3 .
6
Thay vo cc iu kin bin v gii h phng trnh thu c ta c:
4
1
C1 = 1 ; C2 = ; C3 = . Vy phng trnh cho c nghim l
3
2
4
1
1
xn = 1 + n n 2 + n 3 .
3
2
6

58

2.5. Phng trnh sai phn tuyn tnh cp 3

2.5.4

Phng trnh sai phn tuyn tnh cp k

nh ngha Phng trnh


a0 yn+k + a1 yn+k1 + + ak yn = f (n) quad(1)

c gi l phng trnh sai phn tuyn tnh cp k. Cch gii A. Gii phng trnh sai phn tuyn
1o ) Gii phng trnh c trng
a0 k + a1 k1 + + ak1 + ak = 0 (2) tm k.
2o ) Tm nghim tng qut ca phng trnh thun nht tng ng.
Nu (2) c k nghim thc khc nhau l 1, 2, , k th nghim tng
qut l
yn = c1n1 + c2n2 + + ck nk (3).
trong c1 , c1, , ck l cc hng s tu .
Nu (2) c nghim thc j bi s th nghim tng qut l:
yn =

s1
X
i=1

k

X
cj+i ni nj +
ci ni .
i=1;i6=j

Nu phng trnh c trng (2) c nghim phc n j = r(cos + i. sin )


th j = r(cos i. sin ) cng l nghim ca (2). t j+1 = j . thu
c cng thc nghim tng qut, trong cng thc (3) ta thay b phn
cj nj + cj+1 nj+1
bi b phn tng ng:
cj rn cos n + cj+1 rn sin n.
Nu phng trnh c trng (2) c nghim phc bi s
j = j+1 = = j+s1 = r(cos + i. sin )
th (2) cng c nghim phc bi s lin hp vi j l j m ta t l
j+s = j+s+1 = = j+2s1 = r(cos i. sin ).
Trong trng hp ny, thu c cng thc nghim tng qut, trong cng
thc (3) ta thay b phn
cj nj + cj+1 nj+1 + + cj+2s1 nj+2s1

59

2.5. Phng trnh sai phn tuyn tnh cp 3

bi b phn tng ng
s1
X
i=0

s1


X
cj+i ni rn cos n +
cj+s+i ni rn sin n.
i=0

B. Tm nghim ring ca phng trnh sai phn tuyn tnh


khng thun nht. Vic tm nghim ring ca phng trnh sai phn
tuyn tnh khng thun nht cp k lm tng t nh tm nghim ring ca
phng trnh sai phn tuyn tnh khng thun nht cp hai v cp ba.
C. Tm nghim tng qut ca phng trnh sai phn tuyn tnh cp k.
Nghim tng qut c dng
yn = yn + yn ,
trong : +) yn l nghim ca phng trnh sai phn tuyn tnh cp k. +) yn l
nghim ca phng trnh thun nht tng ng. +) yn l mt nghim ring ca
phng trnh khng thun nht.

Chng 3

Xc nh s hng tng qut


ca mt dy s
Vic tnh gii hn ca mt dy s c cho bi cng thc truy hi thng
phi qua giai on chng minh s tn ti gii hn ca dy cho v sau s
dng h thc lim xn+1 = lim xn i vi dy hi t bt k. iu thng
n

c thc hin bng cch s dng nguyn l Weierstrass ( iu kin dy


hi t ) hoc nguyn l hi t Bolzano - Cauchy. Qu trnh gp khng t kh
khn. Mt trong nhng phng thc khc phc kh khn l chuyn t cch
cho dy bng cng thc truy hi sang cho dy bng phng php gii tch, tc
l xc nh dy bng cng thc s hng tng qut ca n. Bi ton xc nh s
hng tng qut ca mt dy s c cho bi h thc truy hi l bi ton thng
gp trong chng trnh ph thng. Bi ton c pht biu nh sau. Xc nh
s hng tng qut ca dy s (xn ) c cho bi h thc truy hi.
(
x1 = 1 ; x2 = 2 ; ; xk = k ()
quad(I)
f (xn+k ; xn+k1 ; ; xn+1 ; xn; n) = 0 (1)
trong 1 ; 2; ; k l cc s R, cho trc, cn f l mt biu thc cha
k + 2 bin, cho trc. Thc cht bi ton ang xt l bi ton xc nh hm s
xn = x(n) tho mn phng trnh sai phn (I) vi cc iu kin bin (). Do ,
i khi ta cng gi bi ton xc nh dy s c cho bi h thc truy hi (I) l
bi ton gii phng trnh sai phn (I).

60

61

3.1. Tm s hng tng qut ca dy (dng a thc) khi bit cc s hng u tin

3.1

Tm s hng tng qut ca dy (dng a thc)


khi bit cc s hng u tin

V d 3.1. Cho dy s:
1; 1; 1; 1; 5; 11; 19; 29; 41; 55; .
Hy tm quy lut biu din ca dy s v tm s tip theo.
Gii. Lp bng mt s sai phn ban u:
y=
y
2 y

-1
-2

-1
0

1
2

5
4

11
6

19
8

29
10

41
12

14
2

Ta thy sai phn cp hai khng i nn dy s l dy cc gi tr ca a thc bc


hai:
y = an2 + bn + c (a 6= 0)
trong n l s th t ca cc s trong dy s. Cho n = 0 ; 1 ; 2 (nh s cc
s bt u t 0) ta nhn c h phng trnh:

c
=
1

a = 1
b = 3 .
a + b + c = 1

4a + 2b + c = 1
c=1
Vy dy s tun theo quy lut sau:
yn = n2 3n + 1
S hng u tin l y0 = 1, s hng tip theo s hng 55 s ng vi n = 10 nn
s l:
y10 = 102 3.10 + 1 = 71.
V d 3.2. Cho dy s:
5; 3; 11; 43; 99; 185; 307; 471; .
Hy tm quy lut biu din ca dy s v tm hai s hng tip theo.
Gii. Lp bng mt s sai phn ban u:

55

3.1. Tm s hng tng qut ca dy (dng a thc) khi bit cc s hng u tin

y=
y
2 y
3 y

-5

-3
2

11
14

12

43
32

18
6

99
56

24
6

185
86

30
6

307
122

36
6

62

471
164

42
6

Ta thy sai phn cp ba khng i nn dy s l dy cc gi tr ca a thc bc


ba:
y = an3 + bn2 + cn + d (a 6= 0)
trong n l s th t ca cc s trong dy s. Cho n = 0 ; 1 ; 2 ; 3 (nh s
th t cc s hng bt u t 0) ta nhn c h phng trnh:

d
=
5
a=1

a + b + c + d = 3
b = 3

8a + 4b + 2c + d = 11
c = 2

27a + 9b + 3c + d = 43
d = 5
Vy dy s tun theo quy lut sau:
yn = n3 + 3n2 2n 5.
S hng u tin l y0 = 5, hai s hng tip theo s hng 471 s ng vi n = 8; 9
nn s l:
y8 = 83 + 3.82 2.8 5 = 683 ; y9 = 93 + 3.92 2.9 5 = 949.
Ch : 1) Quy lut tm c trn l khng duy nht v hin nhin, cc s hng
cho cng tho mn, chng hn quy lut:
yn = n3 +3n2 2n5+P (n).(n+5)(n+3)(n11)(n43)(n99)(n185)(n307)(n471)
trong P (x) l mt a thc bt k. Vy thc cht trn y ta mi ch tm c
mt quy lut m dy cc s cho tho mn m khng tm c tt c cc quy
lut m dy cc s cho tho mn. 2) Nh rng 2 (ax2 + bx + c) = Const,
nhng nu 2y = Const th cha chc l (khng th suy ra c) y = ax2 +bx+c.
Bi tp tng t
Bi ton 3.1. 1 Vi mi dy s sau y hy: a) Tm mt quy lut biu din ca
dy s. b) Vit hai s hng tip theo ca mi dy s theo quy lut va tm c
:
1. : 1; 2; 2; 1; 7; 16; 28; 43; 61; .

3.2. Cng thc truy hi l mt biu thc tuyn tnh

63

2. : 1; 6; 17; 34; 57; 86; 121; .


3. : 2; 3; 7; 14; 24; 37; .
4. : 3; 5; 10; 18; 29; .
5. : 5; 1; 5; 14; 28; 47; 71; 100; 134; 173; 217; .
Bi ton 3.2. 2 Tm quy lut ca cc dy s sau:
1.

2; 2; 8; 26; 62; 122; 212; 338 ; .

2.

1; 6; 17; 34; 57; 86; 121; 162; 209; 262 ; .

3.

5; 3; 11; 43; 99; 185; 307; 471; 683; 949 ; .

Bi ton 3.3. 3 Tm cng thc s hng tng qut ca dy s khi bit cc s


hng u tin.
1.

{8; 14; 20; 26; 32 ; }.

2.

{0, 5 ; 1, 5 ; 4, 5 ; 13, 5 ; 40, 5 ; }.

3.

3 4 5 6
{2; ; ; ; ; }.
2 3 4 5

4.

{1; 3; 1; 3 ; }.

5.

{5; 7; 11; 19; 35 ; }.

6.

{1; 2; 6; 24; 120 ; }.

7.

1
4
3
6
{2 ; ; ; ; ; }.
2
3
4
5

8.

{0, 3 ; 0, 33 ; 0, 333 ; }.

9.

1 1 3 1 5
{ ; ; ; ;
; }.
2 2 8 4 32

3.2

Cng thc truy hi l mt biu thc tuyn tnh

Trng hp h thc truy hi cho l h thc tuyn tnh.


a0 xn+k + a1 xn+k1 + + ak xn = f (n).
vi a0 ; a1; ; ak (a0 6= 0 ; ak 6= 0) l cc hng s th bi ton c th c xem
nh mt phng trnh sai phn tuyn tnh v c gii nh trong chng trc.
Tuy nhin, cng c th gii bng cc phng php khc.

3.2. Cng thc truy hi l mt biu thc tuyn tnh

3.2.1

64

V d

V d 3.3. Tm s hng tng qut ca dy s (xn ) c cho bi h thc truy


hi.
(
x0 = 99
.
xn+1 = xn 2n 1 (1)
Gii. Coi (1) l phng trnh sai phn tuyn tnh cp 1. Do f (n) = 2n 1 l
a thc bc nht, = 1 nn ta chn xn = n(an + b). Thay vo (1) c.
(n + 1)[a(n + 1) + b] = n(an + b) 2n 1 a = 1 ; b = 0 xn = n2 .
Cn x
n = C.1n = C xn = C n2 , m x0 = 99 C 02 = 99 C = 99. Vy
phng trnh (1) c nghim. xn = 99 n2 .
Gii (Cch 2). T h thc cho ta c.
x0 = 99
x1 = x0 1
x2 = x1 3

xn1 = xn2 (2n 3)
xn = xn1 (2n 1)
Cng tng v cc ng thc trn, ta c
xn = 99 [1 + 3 + 5 + + (2n 1)] = 99 n2 .
Vy cng thc s hng tng qut ca dy s cn tm l xn = 99 n2 .
V d 3.4. Tm s hng tng qut ca dy s (xn ) c cho bi h thc truy
hi.
(
x0 = 8
.
xn+1 = 2xn + 3n (2)
Gii. Do = 2 6= 3 = nn ta chn xn = d.3n . Thay vo phng trnh (2)
c d = 1 xn = 3n . Cn x
n = C.2n . Vy xn = C.2n + 3n . Thay vo iu kin
bin c C = 7.
Tr li: phng trnh cho c nghim. xn = 7.2n + 3n .
Gii (Cch khc). t yn = xn 3n , ta c
(
(
y0 = 8 1 = 7
y0 = 7

n
n
n
yn+1 + 3.3 = 2(yn + 3 ) + 3
yn+1 = 2yn

3.2. Cng thc truy hi l mt biu thc tuyn tnh

65

T c (yn ) l cp s nhn yn = 7.2n xn = 7.2n + 3n l cng thc s hng


tng qut cn tm .
V d 3.5. Tm tt c cc dy s (an ) tho mn an+1 = 2n 3an v (an ) l mt
dy s tng.
Gii. Xt phng trnh sai phn. an+1 = 2n 3an (3).
t an = un .2n . Thay vo (3) c.
3
1
un+1 .2n+1 = 3.un .2n + 2n un+1 = un + . (3.1)
2
2
Phng trnh ny c nghim tng qut l.
3
1
1
un = C.( )n + an = C.(3)n + .2n .
2
5
5
Ta c.
(an ) an+1 > an
2
1
3C.(3)n + .2n > C.(3)n + .2n vi mi n N
5
5
1 n
n
4C.(3) < .2 vi mi n N. ()
5
1
3
+) Vi C > 0 th ()
> ( )n vi mi n N. Ta khng chn c C v
20C
2
3 n
khi n chn th ( ) +.
2
1
3
+) Vi C < 0 th ()
< ( )n vi mi n N. Ta cng khng chn c
20C
2
3 n
C v khi n l th ( ) .
2
1
+) Vi C = 0 th an = .2n l dy s tng.
5
1
Vy dy s cn tm l. an = .2n .
5
V d 3.6. Cho a ; q ; d l cc s R, cho trc. Hy xc nh s hng tng
qut ca dy s (un ) c cho bi cng thc truy hi.
(
u1 = a
. (4)
un+1 = qun + d (n 1)
Hy xt tt c cc trng hp c th xy ra i vi cc tham s a ; q ; d.
(Dy s c cho bi cng thc trn cn c gi l cp s nhn - cng)

3.2. Cng thc truy hi l mt biu thc tuyn tnh

66

Gii.
+) Nu q = 0 th (4) xc nh dy s c cng thc s hng tng qut
u1 = a ; un = d, n N, n 2.
+) Nu q = 1 th (4) xc nh mt cp s cng c cng thc s hng tng qut
un = a + (n 1)d.
+) Nu d = 0 th (4) xc nh mt cp s nhn c cng thc s hng tng qut
un = a q n1 .
+) Ta xt trng hp d 6= 0 ; q 6= 0 ; q 6= 1. t un := vn + (vi chn sau)
ta c
(
v1 = a
(4)
. (4.1)
vn+1 + = q(vn + ) + d (n 1)
d
ta c (4.1) l h thc truy hi xc nh mt cp s nhn vi
1q
cng bi q v do

d  n1
d
vn = v1 q n1 un = a
+
q
.
1q
1q

Chn =

V d 3.7. Cho a ; b ; p ; q l cc s R, cho trc. Hy xc nh s hng


tng qut ca dy s (un ) c cho bi cng thc truy hi.
(
u0 = a ; u1 = b
.
un+1 = (p + q)un pqun1 , (n 1) (5)
Hy xt tt c cc trng hp c th xy ra i vi cc tham s a ; b ; p ; q.
Gii. t vn = un pun1 ta c c
v1 = u1 pu0 = a pb ; vn+1 = qvn vn = v1 q n1 ().
Ap dng lin tip () ta c.
u1 pu0 = v1
u2 pu1 = v1q
u3 pu2 = v1q 2

un pun1 = v1q n1 .

67

3.2. Cng thc truy hi l mt biu thc tuyn tnh

T cc dng thc trn d dng nhn c kt qu sau.


n
n
n1 q n1
p q b pq p
a nu p 6= q
un =
pq
pq
n1
np
b (n 1)pn a nu p = q

V d 3.8. Cho a ; b ; p ; q ; r l cc s R, cho trc, pr 6= 0. Bit rng


phng trnh
pt2 + qt + r = 0 ()
c hai nghim thc t = t1 ; t = t2 . Hy xc nh s hng tng qut ca dy s
(un ) c cho bi cng thc truy hi.
(
u1 = a ; u2 = b
. (6)
pun+2 + qun+1 + run = 0 (n 1)
HDG. Chia hai v ca phng trnh cho p ri s dng nh l Vieete, a v v
d 5.
V d 3.9. Xc nh s hng tng qut ca dy s c cho bi h thc truy hi:
(
x0 = 101
.
xn+1 = 7.xn + 7n+1 (1.4)
Gii. Coi (1.4) l phng trnh sai phn tuyn tnh (cp 1), khng thun nht,
vi h s hng s. Do = 7 = nn ta chn xn = d.n.7n . Thay vo phng
trnh (1.4) c d = 1 xn = n.7n . Cn x
n = C.7n . Vy xn = C.7n + n.7n .
Thay vo iu kin bin c C = 101.
Tr li. phng trnh cho c nghim. xn = (101 + n).7n .
Gii (Cch khc). t xn = yn .7n . Ta thu c h thc truy hi i vi dy
s (yn ).
(
(
x0
= y0 .70
= 101
y0

n+1
n
n+1
yn+1 = yn + 1
yn+1 .7
= 7.yn .7 + 7
T ta thy (yn ) l dy s cng vi s hng u y0 = 101, cng sai d = 1. Theo
cng thc s hng tng qut ca dy s cng ta c
yn = y0 + n.d yn = 101 + n.
Bi vy
xn = (101 + n).7n
l cng thc s hng tng qut ca dy s cn tm.

68

3.2. Cng thc truy hi l mt biu thc tuyn tnh

V d 3.10. Xc nh cng thc s hng tng qut ca dy s (xn ) c cho bi


h thc sau .
(
x0 = 1 ; x1 = 16
.
xn+2 = 8xn+1 16xn
Gii. Coi h thc cho l phng trnh sai phn tuyn tnh (cp 2) thun
nht, vi h s hng s. Ta c phng trnh c trng.
2 8 + 16 = 0 1 = 2 = 4 (c nghim kp).
Vy. x
n = xn = (A + Bn).4n .
Gii iu kin bin.
(
(
x0 = 1
A=1

x1 = 16
(A + B).4 = 16

(
A=1

B=3

Vy phng trnh cho c nghim. xn = (1 + 3n).4n.


Gii. t xn = yn .4n . Ta thu c h thc truy hi i

y
.4
=
x

0
0

y0
1
y1
y1 .4
= x1

n+2
n+1
n
yn+2
yn+2 .4
= 8.yn .4
16.yn .4

vi dy s (yn ).
=1
=4
= 2yn+1 yn

t tip zn = yn+1 yn (n 0) ta c
(
z0 = 3
zn = 3 n 0
zn+1 = zn n 0
Nh vy, ta c h thc truy hi

y0
y1

yn+1

i vi dy s (yn ) l.
=1
=4
= yn + 3 (n 0)

T ta thy (yn ) l dy s cng vi s hng u y0 = 1, cng sai d = 3. Theo


cng thc s hng tng qut ca dy s cng ta c
yn = y0 + n.d yn = 1 + 3n.
Bi vy
xn = (1 + 3n).4n
l cng thc s hng tng qut ca dy s cn tm.

3.2. Cng thc truy hi l mt biu thc tuyn tnh

Bi tp
1. Tm s hng tng qut ca dy s (un ) c cho bi.
un+1 = 3un 6n + 1 ; u1 = 1.
p s: un = 3n + 1 3n .
2. Tm s hng tng qut ca dy s (un ) c cho bi.
un+1 = un + 2n2 ; u1 = 1.
1
p s: un = (2n3 3n2 + n + 3).
3
3. Tm s hng tng qut ca dy s (un ) c cho bi.
un+1 = 5un 3n ; u0 = 1.
1
p s: un = (5n + 3n ).
2
4. Tm s hng tng qut ca dy s (un ) c cho bi.
un+1 = 2un + 6.2n ; u0 = 1.
p s: un = (3n + 1).2n.
5. Tm s hng tng qut ca dy s (un ) c cho bi.
un+1 = un + 2n.3n ; u0 = 0.
1
p s: un = [(2n 3).3n + 3].
2
6. Tm s hng tng qut ca dy s (un ) c cho bi.
un+1 2un = (n2 + 1).2n ; u0 = 1.


n(2n2 3n + 7)
p s: un =
+ 2 .2n .
6
7. Tm s hng tng qut ca dy s (un ) c cho bi.
un+1 2un = n + 3n ; u0 = 1.
p s. un = 2n + 3n n 1.

69

70

3.3. Cng thc truy hi l mt h biu thc tuyn tnh

3.3

Cng thc truy hi l mt h biu thc tuyn tnh

Xt bi ton sau: Xc nh s hng tng qut ca cc dy s (xn ) ; (yn ) tho


mn h thc truy hi dng.

x1 = a ; y1 = b ()
(I) vi a; b; p; q; r; s l cc hng s R
xn+1 = pxn + qyn (1)

yn+1 = rxn + syn (2)


Phng php gii. Trong (1) thay n bi n + 1 v bin i ta c.
xn+2 = pxn+1 + qyn+1
= pxn+1 + q(rxn + syn )
= pxn+1 + qrxn + s(xn+1 pxn )
xn+2 (p + s)xn+1 + (ps qr)xn = 0
T (1) ta cng c x2 = px1 + qy1 = pa + qb. Vy ta thu c phng trnh sai
phn tuyn tnh cp hai thun nht:
(
x1 = a ; x2 = pa + qb
xn+2 (p + s)xn+1 + (ps qr)xn = 0
M ta bit cch gii chng trc. Gii phng trnh ny ta tm c xn .
Thay vo (1) ta tm c yn .

3.3.1

V d

V d 3.11. Tm xn ; yn tho mn.

x1 = 1 ; y1 = 1
xn+1 = 4xn 2yn (1)

yn+1 = xn + yn quad(2)

Gii. Trong (1) thay n bi n + 1 ta c


xn+2 = 4xn+1 2yn+1
= 4xn+1 2(xn + yn ) = 4xn+1 2xn 2yn
= 4xn+1 2xn + xn+1 4xn = 5xn+1 6xn
xn+2 5xn+1 + 6xn = 0.

71

3.3. Cng thc truy hi l mt h biu thc tuyn tnh

T (1) ta c: x2 = 4x1 2y1 = 4.1 2.1 = 2. Vy ta c phng trnh sai phn


tuyn tnh thun nht:
(
x 1 = 1 ; x2 = 2
.
xn+2 5xn+1 + 6xn = 0
Gii phng trnh ny(ta c xn = 2n1 . Thay xn vo (1) c yn = 2n1 . Vy
xn = 2n1
h cho c nghim.
.
yn = 2n1
V d 3.12. Tm xn ; yn tho mn.

x0 = 2 ; y 0 = 0
4xn+1 = 2xn 3yn (1)

2yn+1 = 2xn + yn quad(2)

Gii. Lp lun tng t v d trn ta c phng trnh sai phn tuyn tnh
thun nht:
(
x0 = 2 ; x1 = 1
.
xn+2 xn+1 + xn = 0
Phng trnh c trng.
2 + 1 = 0 = cos

i sin .
3
3

Vy phng trnh trn c nghim tng qut.


xn = A. cos

n
n
+ B. sin
.
3
3

n
Thay vo iu kin bin c A = 2 ; B = 0 xn = 2 cos
. Thay tip vo (1)
3
4
n
c yn = sin
.
3
3

xn = 2 cos
3
Vy h cho c nghim:
4
n . Bi tp Tm xn , yn tho mn:

y
=
sin
n
3
3

x0
= 2; y0 = 2

1
3
xn+1 = xn yn .
1.
2
4

yn+1 = xn + 1 yn
2

3.4. Cng thc truy hi l biu thc tuyn tnh vi h s bin thin

2.

3.

4.

5.

6.

3.4

x0
xn+1

yn+1

x0
xn+1

yn+1

x0
xn+1

yn+1

x0
xn+1

yn+1

x0

xn+1

yn+1

72

= 0; y0 = 6
= 3xn + yn .
= 5xn yn
= 2; y0 = 1
= 2xn yn .
= xn + 4yn
= 1; y0 = 2
= 2xn 8yn .
= 2xn 6yn
= 1; y0 = 1
= 4xn 2yn + 9n 3 .
= xn + yn + 3n
1
= 1; y0 = 1
2
.
= xn yn
= xn + yn

Cng thc truy hi l biu thc tuyn tnh vi h


s bin thin

L thuyt v phng trnh sai phn tuyn tnh vi cc h s bin thin cho
n nay vn cha hon chnh. Vic gii cc phng trnh sai phn tuyn tnh vi
cc h s bin thin l rt phc tp. Trong phn ny ta s ch xt mt s dng
c bit, n gin ca cc phng trnh sai phn tuyn tnh vi cc h s bin
thin ch yu bng phng php t dy s ph, a v phng trnh sai phn
tuyn tnh.
V d 3.13. Tm un bit rng.
u1 = 0 ; un+1 =

n
(un + 1) vi mi n 1.
n+1

Gii. T gi thit c: (n + 1)un+1 = nun + n. t xn = nun , ta c


x1 = 0 ; xn+1 = xn + n.

3.4. Cng thc truy hi l biu thc tuyn tnh vi h s bin thin

Gii phng trnh ny ta c:


xn =

n(n 1)
n1
. Vy ta c: un =
.
2
2

V d 3.14. Tm un bit rng.


u1 = 0 ; un+1 =

n(n + 1)
(un + 1) vi mi n 1.
(n + 2)(n + 3)

Gii. T gi thit c:
(n + 1)(n + 2)2(n + 3)un+1 = n(n + 1)2(n + 2)un + n(n + 1)2 (n + 2).
t xn = n(n + 1)2(n + 2)un , ta c
x1 = 0 ; xn+1 = xn + n(n + 1)2(n + 2).
Gii phng trnh ny ta c:
xn =

(n 1)n(n + 1)(n + 2)(2n + 1)


.
10

Vy ta c p s:
un =
V d 3.15. Tm:

(n 1)(2n + 1)
.
10(n + 1)

Z
Jn = 2 sinn x dx.
0

Gii. S dng cng thc tch phn tng phn ta c.

Z

Jn = cos x. sinn1 x 2 + 2 (n 1) sinn2 x cos2 x dx
0

Z
= 2 (n 1) sinn2 x(1 sin2 x) dx
0

Z

= (n 1) 2 sinn2 x sinn x dx
0

Jn+2

= (n 1)(Jn2 Jn ).
n+1
=
Jn (1).
n+2

73

3.4. Cng thc truy hi l biu thc tuyn tnh vi h s bin thin

; J1 = 1. T v t (1) ta c
2
+) Khi n chn (n = 2k) th

D thy: J0 =

1
J0
2
3
J4 = J 2
4
5
J6 = J 4
6

2k 1
J2k =
J2k2 .
2k
J2 =

Nhn tng v cc ng thc trn v rt gn ta c.


J2k =

1.3. .(2k 1)
(2k 1)!!
. =
. .
2.4. .(2k) 2
(2k)!! 2

+) Khi n l (n = 2k + 1) th.
2
J 3 = J1
3
4
J5 = J 3
5
6
J7 = J 5
7

2k
J2k+1 =
J2k1 .
2k + 1
Nhn tng v cc ng thc trn v rt gn ta c.
J2k+1 =

2.4. .(2k)
(2k)!!
.1 =
.
1.3. .(2k + 1)
(2k + 1)!!

p s.
J2k =

(2k 1)!!
(2k)!!
. ; J2k+1 =
.
(2k)!! 2
(2k + 1)!!

V d 3.16. Tm xn bit rng.


x1 = a > 0 ; xn+1 = g(n).xkn (1) vi mi n 1,
trong g(n) > 0 vi mi n N ; k R+ .

74

75

3.4. Cng thc truy hi l biu thc tuyn tnh vi h s bin thin

Gii. T gi thit suy ra xn > 0 vi mi n N. Ly logarit Neper hai v ca


(1) ta c
ln xn+1 = ln g(n) + k. ln xn . (2)
t yn = ln xn , khi (2) c dng.
yn+1 kyn = ln g(n). (3)
t tip yn = kn1 un khi (3) c dng.
n1

un+1 un =

X ln g(i)
ln g(n)
un = u1 +
.
n
k
ki
i=1

T gi thit x1 = a > 0 u1 = ln a. Vy.


un = ln a +

n1
X
i=1

ln g(i)
yn = kn1
ki

ln a +

n1
X
i=1

ln g(i)
ki

Cui cng, ta c:
xn = e

k n1 (ln a+

n1 ln g(i)
)
i=1
ki

= exp k

n1

(ln a +

n1
X
i=1

!
ln g(i)
) .
ki

V d 3.17. Tm xn bit rng.


x1 = a > 0 ; xn+1 =

f (n + 1) k
.xn (1) vi mi n 1.
f k (n)

Trong f (n) > 0 vi mi n N cn k N , cho trc.


Gii. T (1) ta c:
xn+1
xk
= k n . (2)
f (n + 1)
f (n)
xn
, khi (2) c dng: vn+1 = vnk . (3). t tip dy s
f (n)
ph: un = ln vn , khi (3) c dng.
t dy ph: vn =

un+1 = kun un = C.kn (Vi C l hng s).


M x1 = a v1 =

a
a
a n1
. Vy ta c.
u1 = ln
= C.k un = ln
k
f (1)
f (1)
f (1)

k n1 ln

vn = e

a

kn1
a
a kn1
f (1) =
. Hay l. xn = f (n)(
.
)
f (1)
f (1)

3.4. Cng thc truy hi l biu thc tuyn tnh vi h s bin thin

76

V d 3.18. Tm s hng tng qut ca dy s (xn ) bit rng x1 = a v


xn+1 = a(n)xn + b(n) quad(6)
trong a(n) ; b(n) l cc hm s i vi n N, a(n) 6= 0, n N.
Gii. t dy s ph
x n = yn .

n1
Y

a(k)

k=0

a
Khi ta c y1 =
v
a(0)
b(n)
(6) yn+1 yn = Qn
:= g(n). quad(6.1)
k=0 a(k)
T ng thc (6.1) ta d dng nhn c
y n = y1 +

n1
X

n1

g(k) =

k=1

X
a
g(k).
+
a(0)
k=1

Vy nn ta c
n1
n1
 a
X
b(k)  Y
xn =
a(k).
+
Qk
a(0)
j=0 a(j)
k=1

k=0

l cng thc s hng tng qut cn tm.


V d 3.19. Tm s hng tng qut ca dy s (xn ) bit rng x1 = a ; x2 = b
v
xn+2 = a(n)xn+1 + b(n)xn + f (n) quad(7)
trong a(n) ; b(n) ; f (n) l cc hm s i vi n N, b(n) 6= 0, n N v tn
ti s p 6= 0, tn ti hm s q(n) 6= 0 n N sao cho:
p + q(n) = a(n) ; p.q(n) = b(n). ()
Gii. S dng iu kin () ta c th vit li (7) di dng
(xn+2 pxn+1 ) q(n)(xn+1 pxn ) = f (n) quad(7.1)
t yn = xn+1 pxn (1). Khi , (7.1) c dng.
yn+1 = q(n)yn + f (n) ; y1 = b pa := .

3.4. Cng thc truy hi l biu thc tuyn tnh vi h s bin thin

77

Theo v d 6, ta tm c
yn =

n1

X f (k)  n1
Y
q(k) := h(n), n > 1.
+
Qk
q(0)
j=0 q(j)
k=1

k=0

Thay yn vo (1), gii phng trnh sai phn tuyn tnh cp 1 thu c ta tm
c cng thc s hng tng qut cn tm l.
xn = a.pn1 + pn .

n1
X
h(k)
k=1

pk+1

, n > 1.

V d 3.20. Tm s hng tng qut ca dy s (xn ) nu bit


x1 = 1 ; x2 = 2 ; xn+2 = (n + 1)(xn+1 + xn ), n 1. (7)
Gii. t xn = n!yn . Khi , (7) c dng
n!yn = (n 1)[(n 1)!xn1 + (n 2)!xn2]
hay l
1
nyn (n 1)yn1 yn2 = 0 (yn yn1 = (yn1 yn2 ), n 2.
n
T c
yn yn1 =

(1)n1
1
1
, y 3 y2 = , y2 y1 = .
n!
6
2

Vit cc ng thc trn ri cng li ta c


yn = y1 +

n1
X
k=2

n1

X (1)k1
(1)k1
=1+
, n 3.
k!
k!
k=2

Vy cng thc s hng tng qut cn tm l




xn = n! 1 +

n1
X
k=2

Bi tp
Tm xn bit rng:

(1)k1 
, n 3.
k!

78

3.5. Cng thc truy hi dng phn tuyn tnh vi h s hng

=0

1.

x1

=0

2.

x1

xn+1

=
=

3.

x1

4.

5.

6.

3.5

xn+1

xn+1

x1
xn+1

x1

xn+1

n
.
(xn + 1)
n+1
.
n(n + 1)
(xn + 1)
(n + 2)(n + 3)

1
2
.
(n + 1)2
n(n + 1)
xn +
n!
=
n(n + 2)
n+2
=a
.
= (n + 1)(xn + 1)
=2
=

.
n
4.3n
xn +
n+2
(n + 1)(n + 2)

33

x1 = 8 ; x 2 =
2
(n + 1)(n + 3)
n(n + 2)
n(n + 3)

xn+2 2
xn+1 3
xn = 4
2
(n + 2)
(n + 1)(n + 3)
n+2

Cng thc truy hi dng phn tuyn tnh vi h


s hng

Trong phn ny ta s tm s hng tng qut ca dy s c cho di dng


cng thc truy hi dng phn tuyn tnh vi h s hng thng qua cc v d c th.
V d 3.21. Tm dy s (xn ) tho mn cc iu kin sau.
xn
x1 = a > 0 ; xn+1 =
vi mi n 1.
xn + 2
Gii. T gi thit suy ra xn > 0 vi mi n N. M.
xn+1 =
t yn =

xn
2
1
=1+
.

xn + 2
xn+1
xn

1
, khi ta c.
xn
yn+1 = 2yn + 1 yn+1 2yn 1 = 0 ; y1 =

1
.
a

3.5. Cng thc truy hi dng phn tuyn tnh vi h s hng

79

Gii phng trnh sai phn ny ta c:


yn =

(a + 1)2n1 a
a
. Hay l. xn =
a
(a + 1)2n1 a

V d 3.22. Tm dy s (xn ) tho mn cc iu kin sau.


x0 = a ; xn+1 =

pxn + q
vi mi n N. (1)
rxn + s

trong a, p, q, r, s R, cho trc.


Gii. Li gii ca v d ny thu c trc tip t B sau.
B 3.1. Nu yn v zn l nghim ca h phng trnh sai phn.
(
yn+1 = pyn + qzn ; y0 = a
. (2)
zn+1 = ryn + szn ; z0 = 1
th xn =

yn
l nghim ca phng trnh.
zn
x0 = a ; xn+1 =

Chng minh. Tht vy, ta c: x0 =

xn+1

pxn + q
.
rxn + s

y0
a
= = a. Ngoi ra.
z0
1

yn
p +q
yn+1
pyn + qzn
pxn + q
z
=
=
= ynn
.
=
zn+1
ryn + szn
rxn + s
r +s
zn

T suy ra pcm. T B trn ta c c cch gii ca phng trnh sai phn


dng phn tuyn tnh (1) bng cch lp v gii h phng trnh (2). T thu
c nghim ca (1) theo B . V d c th xem trong li gii ca v d 3 sau
y.
V d 3.23. Tm dy s (xn ) tho mn cc iu kin sau.
x0 = 0 ; xn+1 =

xn + 1
vi mi n 1.
xn + 1

3.5. Cng thc truy hi dng phn tuyn tnh vi h s hng

80

Gii. Xt h phng trnh

y0 = 0 ; z0 = 1
yn+1 = yn + zn

zn+1 = yn + zn

Gii h ny ta c.

n
n
yn = ( 2)n sin
; zn = ( 2)n cos
.
4
4
T , theo B chng minh trn ta c nghim ca phng trnh cho
l.

n
( 2)n sin
yn
4 = tg n .
xn =
=
n
zn
4
( 2)n cos
4
V d 3.24. Tm cng thc s hng tng qut ca dy s (xn ) c cho bi h
thc truy hi sau
bxn
x1 = a ; xn+1 =
quad(4).
cxn + d
Trong , a, b, c R , d R.
Gii. t yn :=

1
c
d
; := p ; := q ta c
xn b
b
(4) yn+1 = pxn + q ; y1 =

1
.
a

l h thc xc nh dy s nhn - cng. Cng thc s hng tng qut ca dy


s ny c xc nh trong phn trc (xem v d 4 mc 2.1 ca chng ny).
V d 3.25. Xc nh s hng tng qut ca dy s cho bi h thc truy hi
sau.
1
1
x1 = ; xn+1 =
.
2
2 xn
Gii. Ta xc nh mt s s hng u tin.
x1 =

1
2
3
; x2 = ; x3 = .
2
3
4

Ta s chng minh dy s cho c s hng tng qut


xn =

n
quad()
n+1

81

3.6. H thc truy hi phi tuyn

bng phng php quy np. Tht vy, theo trn, () ng ti n = 3. Gi s


() ng ti n, khi
xn+1 =

n+1
1
1
=
n = n + 2.
2 xn
2
n+1

Vy () cng ng ti n + 1 nn theo nguyn l quy np ton hc, () ng vi


mi n N. chnh l iu phi chng minh.

Bi tp
Tm dy s tho mn iu kin sau:
1.

xn+1 =

1 4xn
; x0 = 1.
1 6xn

2.

xn+1 =

2xn 2
; x0 = 1.
3xn 4

3.

xn+1 =

xn + 1
; x0 = 0.
xn + 1

4.

xn+1 =

xn 2
; x0 = 0.
xn + 4

5.

xn+1 =

xn 1
; x0 = 1.
xn + 3

6.

xn+1 =

7.

xn+1

3.6

xn 3
; x0 = 0.
xn + 1
xn
=
; x0 = 0.
2xn + 1

H thc truy hi phi tuyn

Trong phn ny ta xt cc v d gii cc phng trnh sai phn phi tuyn. L


thuyt tng qut gii cc phng trnh dng ny cho n nay cn cha xy dng
c. Trong phn ny ch yu ta s xt cc phng trnh c th c tuyn tnh
ho bng php t hm ph hoc bng phng php quy np ton hc.

3.6. H thc truy hi phi tuyn

3.6.1

82

Quy trnh tuyn tnh ho mt phng trnh sai phn

Tuyn tnh ho mt phng trnh sai phn ngha l a mt phng trnh sai
phn dng phi tuyn v dng tuyn tnh. Gi s dy s (un ) tho mn iu
kin.
(
u1 = 1 ; u2 = 2 uk = k
.
un = f (un1 , un2 , , unk ) vi n; k N ; n > k
Trong f l mt a thc i s bc m hoc l phn thc, hoc l biu thc siu
vit. Gi s hm s f (un1 , un2 , , unk ) c th tuyn tnh ho c, khi
tn ti cc gi tr x1; x2; ; xk sao cho.
un = x1 un1 + x2 un2 + + xk unk (1)
tm x1 ; x2; ; xk trc ht ta xc nh uk+1 ; uk+2 ; ; u2k .
T cng thc lp cho ta c.

uk+1 = f (k ; k1 ; ; 2; 1) := k+1

u
k+2 = f (k+1 ; k ; ; 3 ; 2 ) := k+2
.

quad quad quad

u = f (
2k
2k1 ; 2k2 ; ; k+1 ; k ) := 2k
Thay cc gi tr u1 ; u2; ; uk cho v cc gi tr uk+1 ; uk+2 ; u2k va tm
c trn vo (1) ta c h phng trnh tuyn tnh gm k phng trnh vi
k n x1; x2; ; xk .

uk+1 = x1 k + x2k1 + + xk 1

u
k+2 = x1 k+1 + x2 k + + xk 2
. ()

quad quad quad

u = x
2k
1 2k1 + x2 2k2 + + xk k
Gii h phng trnh ny ta thu c nghim: x1; x2; ; xk . Thay vo (1) ta s
c biu din tuyn tnh cn tm.
un = f (un1 , un2, , unk ) = x1 un1 + x2 un2 + + xk unk
Sau ta chng minh cng thc biu din trn bng phng php quy np ton
hc.
Ch . Nu h (*) v nghim th hm f khng th tuyn tnh ho c.

83

3.6. H thc truy hi phi tuyn

3.6.2

V d

V d 3.26. Cho dy s (an ) tho mn.


a 1 = a2 = 1 ; a n =

a2n1 + 2
() vi mi n 3
an2

Hy tuyn tnh ho, tm s hng tng qut. Chng minh rng an nguyn vi mi
n N .
Gii. Gi s an c biu din tuyn tnh l:
an = an1 + an2 + . (1)
Ta c.
a22 + 2
1+2
=
= 3.
a1
1
a2 + 2
9+2
a4 = 3
=
= 11.
a2
1
a2 + 2
121 + 2
a5 = 4
=
= 41.
a3
3
a3 =

Thay a3 = 3 ; a4 = 11 ; a5 = 41 vo (1) ta thu c h phng trnh.

a2 + a1 + = a3
+ + = 3
= 4

3 + + = 11
= 1
a3 + a2 + = a4

11 + 3 + = 41
=0
a4 + a3 + = a5

Vy ta c:
an = 4an1 an2 (2)
Ta s chng minh dy s (an ) tho mn (*) c biu din tuyn tnh l.
a1 = a2 = 1 ; an = 4an1 an2 vi mi n 3. (3)
Tht vy, vi n = 3 ta c: a3 = 4.a2 a1 = 4.1 1 (2) ng vi n = 3.

84

3.6. H thc truy hi phi tuyn

Gi s (2) ng ti n = k tc l: ak = 4ak1 ak2 (k 3). Ta c.


a2k + 2
(4ak1 ak2 )2 + 2
=
ak1
ak1
2
16ak1 8ak1 ak2 + a2k2 + 2
=
ak1
2
15ak1 4ak1 ak2 + a2k1 4ak1 ak2 + ak1 ak3
=
ak1
2
( Nh rng: ak2 + 2 = ak1 ak3 )

ak+1 =

15a2k1 4ak1 ak2 + ak1 (ak1 4ak2 + ak3 )


ak1
2
15ak1 4ak1 ak2
=
ak1
(Do ak1 4ak2 + ak3 = 0)
=

= 15ak1 4ak2 = 4(ak1 ak2 ) ak1


= 4ak ak1 .
Vy (2) cng ng ti n = k + 1. Theo nguyn l quy np ta c (2) ng vi
mi n N ; n 3.
T (3) ta thy ngay n N : an Z. Ngoi ra, ta chng minh c.

(
a1 = a2 = 1
a 1 = a2 = 1
. ()
()
a2n1 + 2
an =
an = 4an1 an2 (n 3)
(n 3)
an2
tm s hng tng qut ta gii phng trnh (**). C phng trnh c trng:

2 4 + 1 = 0 = 2 + 3 hoc = 2 3.
Do :

an = a.(2 +

3)n + b.(2

3)n . (4)

Thay vo iu kin bin ta tm c


a=

1
5
5
1
(3 ) ; b = (3 + ).
2
2
3
3

Vy ta c s hng tng qut cn tm l.




n
n
1
5
5
an =
(3 )(2 + 3) + (3 + )(2 3) .
2
3
3

85

3.6. H thc truy hi phi tuyn

V d 3.27. Cho dy s (un ) tho mn.


p
u1 = ; un+1 = aun + bu2n + c vi a2 b = 1 ; > 0 ; a > 1 ()
Hy tuyn tnh ho dy s trn.
Gii.
p

un+1 = aun +

bu2n + c un+1 aun =

bu2n + c

(un+1 aun )2 = bu2n + c


u2n+1 + (a2 b)u2n = 2aun+1 un + c
u2n+1 + u2n = 2aun+1 un + c (1)
u2n u2n1 = 2aun un1 + c (2)
Tr tng v (1) v (2) ta c .
u2n+1 u2n1 = 2aun (un+1 un1 )
M un+1 un1 > 0 nn suy ra.
un+1 2aun + un1 = 0
Ni cch khc:
()

u1 = ; u2 = a + b2 + c
un+1 2aun + un1 = 0

Nh vy vic tuyn tnh ho thc hin xong.


V d 3.28. Cho dy s (xn ) tho mn.
x
pn
x1 = ; xn+1 =
vi a2 b = 1 ; > 0 ; a > 1 ()
a + x2n + b
Hy tuyn tnh ho dy s trn.
Gii.
xn+1

xn
a
1
p
=
=
+

2
x
x
a + xn + b
n+1
n

1+

b
(3)
xn

1
. Khi ta c th vit (2) di dng.
xn
p
1
u1 = ; un+1 = aun + bu2n + 1 vi a2 b = 1 ; > 0 ; a > 1

t un =

chnh l phng trnh sai phn m ta tuyn tnh ho trong v d 2 trn.

86

3.6. H thc truy hi phi tuyn

V d 3.29. Cho dy s (un ) tho mn.


u1 = ; u2 = ; un+1 =

a + u2n
vi a ; ; R ()
un1

Hy tuyn tnh ho dy s trn.


Gii.
un+1 =

a + u2n
un+1 un1 = u2n + a (1)
un1
un un2 = u2n1 + a (2)
Tr tng v ca (1) v (2) ta c.
quadun+1 un1 un un2 = u2n u2n1
un+1 un1 + u2n1 = u2n + un un2
un
un1

=
un+1 + un1
un + un2
un
un1
u2

=
= =
= 2
:= k
un+1 + un1
un + un2
u3 + u 1
+ + 2

Do un = k(un+1 + un1 ) hay l.


(
u1 = ; u 2 =
()
kun+1 un + kun1 = 0
Nh vy vic tuyn tnh ho thc hin xong.
V d 3.30. Cho dy s (xn ) tho mn.
x1 = ; x2 = ; xn+1 =

x2n + 2bxn bxn1 + c


vi ; R ; n 2 (1)
b + xn1

Hy tuyn tnh ho dy s trn.


Gii.
(1) xn+1 + b =

x2n + 2bxn bxn1 + c


(xn + b)2 + c
+b=
(2)
b + xn1
xn1 + b

t yn = xn + b ta c phng trnh sai phn.


y1 = + b ; y2 = + b ; yn+1 =

c + yn2
vi c ; ; R ()
yn1

chnh l phng trnh sai phn m ta tuyn tnh ho trong v d 4 trn.

3.6. H thc truy hi phi tuyn

3.6.3

87

Mt s v d khc

V d 3.31. Xc nh s hng tng qut ca dy s {f (n)} (n N) c cho


bi.
p
f (0) = 2 ; f (n + 1) = 3f (n) + 8f 2 (n) + 1 (1) vi mi n 0
p
Gii. T gi thit ta c: f (n + 1) 3f (n) = 8f 2(n) + 1 0 nn.
(f (n + 1) 3f (n))2 = 8f 2 (n) + 1 f 2 (n + 1) + f 2 (n) = 6f (n)f (n + 1) + 1 ()
Thay n bi n 1 ta c:
f 2 (n) + f 2(n 1) = 6f (n 1)f (n) + 1 ()
Tr tng v (*) v (**) ta c
f 2 (n + 1) f 2 (n 1) = 6f (n)(f (n + 1) f (n 1)) ( )
T gi thit ta cn c f (n) > 0 vi mi n (chng minh bng quy np). Ngoi ra.
p
f (n+1) > 3f (n) = 9f (n1)+3 8f 2 (n 1) + 1 > f (n1) f (n+1)f (n1) > 0
nn: ( ) f (n + 1) + f (n 1) = 6f (n). Vy ta c phng trnh sai phn
tuyn tnh.
(

f (0) = 2 ; f (1) = 6 + 33
f (n + 2) 6f (n + 1) + f (n) = 0
Gii phng trnh ny ta c.

(8 + 66)(3 + 8)n (8 66)(3 8)n


f (n) =
+
8
8
D thy f (n) xc nh nh trn tho mn (1).
V d 3.32. Xc nh s hng tng qut ca dy s {f (n)} (n N ) c cho
bi.
f 2 (n) + a
f (1) = ; f (2) = ; f (n + 1) =
() vi mi n 2
f (n 1)
Gii. Khi , (*) c th c vit li di dng.
f (n + 1)f (n 1) = f 2 (n) + a vi mi n 2
Trong ng thc trn thay n bi n 1 ta c .
f (n)f (n 2) = f 2 (n 1) + a vi mi n 3

88

3.6. H thc truy hi phi tuyn

Tr tng v hai ng thc sau ta c.


f (n + 1)f (n 1) f (n)f (n 2) = f 2 (n) f 2 (n 1) vi mi n 3
Hay l.
f (n + 1)f (n 1) + f 2 (n 1) = f (n)f (n 2) + f 2 (n) vi mi n 3
T c.
f (n)
f (n 1)
=
vi mi n 3
f (n + 1) + f (n 1)
f (n) + f (n 2)
t g(n) :=

f (n)
(n N \ {0; 1}) ta c.
f (n + 1) + f (n 1)
g(n) = g(n 1) vi mi n N \ {0; 1}

Do .
g(n) = g(n 1) = = g(2) =

f (2)

= 2
:= k
f (3) + f (1)
+ 2 + a

Ta c phng trnh sai phn tuyn tnh


f (1) = ; f (2) = ; kf (n + 2) f (n + 1) + kf (n) = 0 (n 1)
Gii phng trnh ny ta c biu thc ca f (n) cn tm.
Ch : Cc phng trnh dng.
1) f (n + 2) =

f 2 (n + 1) + 2bf (n + 1) bf (n) + c
, (n N) (1)
f (n) + b
(Trong : f (1) = , f (2) = )

2) f (n + 1) =

f 2 (n)
, (n N, n 2) (2)
(1 + af 2(n))f (n 1)

(Trong : a > 0, f (1) = 6= 0, f (2) = 6= 0)

89

3.6. H thc truy hi phi tuyn

c th a c v dng ca v d trn. Tht vy, ta c.


f 2 (n) + 2bf (n) + b2 bf (n 1) b2 + c
f (n 1) + b
2
[f (n) + b] b[f (n 1) + b] + c
=
f (n 1) + b
2
[f (n) + b] + c
=
b
f (n 1) + b
[f (n) + b]2 + c
vi mi n 1
f (n + 1) + b =
f (n 1) + b
t g(n) = f (n + 1) + b Ta c.
(1) f (n + 1) =

g(n + 1) =

g 2(n) + c
, n N.
g(n 1)

l phng trnh c dng xt v d 2 trn. T cc iu kin ca phng


trnh (2) ta c f (n) 6= 0 vi mi n N , do .
1
1 + af 2(n)
=
.f (n 1)
f (n + 1)
f 2 (n)


1
1
=
+a
2
1
f (n)
f (n 1)
2
g (n) + a
g(n + 1) =
, n N .
g(n 1)
1
1
1
Trong , g(n) =
, n N, g(1) = , g(1) =
f (n)

(2)

l phng trnh c dng xt v d 2.


V d 3.33. Xc nh s hng tng qut ca dy s {f (n)} (n N ) c cho
bi.
9
f (1) = ; f (n + 1) = nf (n) + n.n! vi mi n N
8
Gii. Ta c nghim tng qut ca phng trnh f (n + 1) nf (n) = 0 l.
f(n) = C.1.2. .(n 1) = C.(n 1)!
Ta s tm nghim ring ca phng trnh cho di dng f (n) = C(n).(n1)!.
Thay vo phng trnh cho c:
C(n + 1).n! = nC(n).(n 1)! + n.n! C = C(n + 1) C(n) = n vi mi n N

90

3.6. H thc truy hi phi tuyn

T y d dng c.
C(n) =

1 2
1
(n n) f = (n2 n)(n 1)!
2
2

Do nghim tng qut ca phng trnh cho l.


1
f (n) = C.(n 1)! + (n 1)2(n 1)!
2
1
Thay vo iu kin bin c C = 1. Vy f (n) = (n 1)! + (n 1)2(n 1)!.
2
D thy f (n) xc nh nh trn tho mn bi ra.
V d 3.34. Xc nh s hng tng qut ca dy s {f (n)} (n N) c cho
bi.
f (0) = ; f (n + 1) = 2f 2 (n) 1 vi mi n N
Gii. Ta c. Nu = 1 th.
f (0) = 1
f (1) = 2f 2(0) 1 = 1
f (2) = 2f 2(1) 1 = 1
f (3) = 2f 2(2) 1 = 1
Bng phng php quy np ta chng minh c: n N, f (n) = 1. Nu = 1
th tng t ta cng chng minh c: n N, f (n) = 1. Nu || < 1 th tn ti
sao cho cos = = arccos . Khi , ta c:
f (0) = cos = cos 20
f (1) = 2f 2 (0) 1 = 2 cos2 1 = cos 21
f (2) = 2f 2 (1) 1 = 2 cos2 2 1 = cos 22
f (3) = 2f 2 (2) 1 = 2 cos2 22 1 = cos 23
Bng phng php quy np ta chng minh c: n N, f (n) = cos 2n . Nu
|| > 1 th tn ti sao cho cosh = . Khi , ta c:
f (0) = cosh = cosh 20
f (1) = 2f 2 (0) 1 = 2 cosh2 1 = cosh 21
f (2) = 2f 2 (1) 1 = 2 cosh2 2 1 = cosh 22
f (3) = 2f 2 (2) 1 = 2 cosh2 22 1 = cosh 23

91

3.6. H thc truy hi phi tuyn

Bng phng php quy np ta chng minh c: n N, f (n) = cosh 2n . V


e + e
cosh = nn:
= . Gii phng trnh ny ta c.
2
p
p
e = 2 1 hoc e = + 2 1
Suy ra.


i
p
p
1
1h
1
n
n
2n
f (n) =
(e ) + 2n =
( 2 1)2 + ( + 2 1)2
2
(e )
2
D thy cc dy s xc nh nh trn tho mn phng trnh cho. Vy ta c.

1 khi = 1

1 (n = 0)

khi = 1
1 (n 1)
f (n) =

cos 2n arccos khi || < 1

i
h
p
p

1 ( 2 1)2n + ( + 2 1)2n khi || > 1


2
V d 3.35. Xc nh s hng tng qut ca dy s {f (n)} (n N) c cho
bi.
f (0) = ; f (n + 1) = af 2 (n) + b (n N) vi ab = 2
HDG: t f (n) = bg(n), n N, ta c.
g(0) =

:= ; g(n + 1) = 2g 2(n) 1 (n N).


b

chnh l v d 4.
V d 3.36. Xc nh s hng tng qut ca dy s {f (n)} (n N) c cho
bi.
n
f (1) = ; f (n + 1) = f 2 (n) 2a2 (n N) vi a > 0.
HDG: t f (n) = 2a2
g(1) =

n1

g(n), n N, ta c.

:= ; g(n + 1) = 2g 2(n) 1 (n N ).
2a

chnh l v d 4.
V d 3.37. Xc nh s hng tng qut ca dy s {f (n)} (n N) c cho
bi.
f (n + 1) = af 2 (n) + bf (n) + c (n N) (1) vi a 6= 0 ; c =

b2 2b
.
4a

92

3.6. H thc truy hi phi tuyn

Gii. Khi ta c.


b
b 2 b2 4ac
b
(1) f (n + 1) +
+
= a f (n) +
+
2a
2a
4a
2a

2
2
2
b
b (b 2b) + 2b
= a f (n) +
+
2a
4a

2
b
= a f (n) +
2a
b
g(n + 1) = ag 2(n) (vi g(n) = f (n) + )
2a
2
g(n + 1) = a[g(n)]2 = a[ag 2(n 1)]2 = a3 [g(n 1)]2 =
=
= a2

n 1

[g(1)]2 = a2

n 1

[ +

b 2n
]
2a

b 2n1
]
2a
n1
b n1
b
f (n) = a2 1 [ + ]2
+
2a
2a
g(n) = a2

n1 1

[ +

Bng php quy np ta d dng chng t c f (n) xc nh nh trn tho mn


phng trnh cho. Vy ta c p s.
f (n) = a2

n1 1

[ +

b 2n1
b
+
]
(n N).
2a
2a

V d 3.38. Tm dy s (xn ) tho mn cc iu kin sau.


x1 = a ; xn+1 =

x2n + d
vi mi n 1. (1).
2xn

Gii.
+) Nu d = 0 th ta c ngay.
1
1
xn+1 = xn xn = a.( )n1 .
2
2
+) Xt trng hp d > 0. Gi s un ; vn l mt nghim ca h phng trnh
sai phn.

u1 = a ; v1 = 1
un+1 = u2n + dvn2 , (2)

vn+1 = 2un vn

93

3.6. H thc truy hi phi tuyn

khi , xn =

un
l nghim ca phng trnh (1). Tht vy, ta c:
vn

u1
a
= = a Khng nh ng vi n = 1.
v1
1
un
Gi s khng nh ng ti n, tc l: xn =
l nghim ca (1). Khi .
vn
x1 =

xn+1 =

un+1
=
vn+1

u2n

dvn2

+
2un vn

u2n
+d
x2 + d
v2
= n un = n
.
2xn
2
vn

Vy xn+1 cng l nghim ca (1). Tc l khng nh cng ng ti n + 1. Theo


nguyn l quy np ton hc, khng nh trn ng vi mi n N. Vy, gii
(1) ta i gii (2). Vit li (2) di dng.

u1 = a ; v1 = 1
. (3)
un+1 = u2n + dvn2 ()

dvn+1 = 2 dun vn ()
Cng tng v (*) v (**) ta c.

un+1 + dvn+1 = (un + dvn )2 vi mi n 1.


T c.

un+1 +

dvn+1 = (u1 +

n
n
dv1 )2 = (a + d)2 . (4)

Tr tng v (*) v (**) ta c.

un+1 dvn+1 = (un dvn )2 vi mi n 1.


T c.

un+1

dvn+1 = (u1

n
n
dv1 )2 = (a d)2 . (5)

T (4) v (5) ta c.

ni
n
1h

un+1 =
(a + d)2 + (a d)2
2
n i . (6)
n
1 h

vn+1 = (a + d)2 (a d)2


2 d
un
Do xn =
nn t (6) ta c.
vn

(a + d)2n1 + (a d)2n1
n1
n1 .
xn = d
(a + d)2
(a d)2

94

3.6. H thc truy hi phi tuyn

C th kim tra nghim ny tho mn bng cch th vo (1).


+)Xt trng hp d < 0. t d = q (q > 0). Tng t trn, ta s chng
minh.
Gi s un ; vn l mt nghim ca h phng trnh sai phn.

u1 = a ; v1 = 1
un+1 = u2n qvn2 , (7)

vn+1 = 2un vn
un
l nghim ca phng trnh (1). Tht vy, ta c:
vn
u1
a
x1 =
= = a Khng nh ng vi n = 1.
v1
1
un
Gi s khng nh ng ti n, tc l: xn =
l nghim ca (1). Khi .
vn
khi , xn =

xn+1 =

un+1
vn+1

u2n
q
u2n qvn2
x2n + d
vn2
=
=
=
.
un
2un vn
2xn
2
vn

Vy xn+1 cng l nghim ca (1). Tc l khng nh cng ng ti n + 1. Theo


nguyn l quy np ton hc, khng nh trn ng vi mi n N. Vy, gii
phng trnh (1) ta i gii h (7). Vit li (7) di dng.

u1 = a ; v 1 = 1
. (8)
un+1 = u2n qvn2 ( )

i qvn+1 = 2i qun vn ( )
Trong i l n v o (i2 = 1). Cng tng v (***) v (****) ta c.

un+1 + i qvn+1 = (un + i qvn )2 vi mi n 1.


T c.
n

n
un+1 + i qvn+1 = (u1 + i qv1 )2 = (a + i q)2 . (9)

Tr tng v (***) v (****) ta c.

un+1 i qvn+1 = (un i qvn )2 vi mi n 1.


T c.
n

n
un+1 i qvn+1 = (u1 i qv1)2 = (a i q)2 . (10)

95

3.6. H thc truy hi phi tuyn

T (9) v (10) ta c.

n
1
n

un+1 =
(a + i q)2 + (a i q)2
2
1 
n
n

vn+1 = (a + i q)2 (a i q)2


2i q
Do xn =

. (11)

un
nn t (6) ta c.
vn
n1
n1
+ (a i q)2
(a + i q)2
xn = i q
n1
n1 .
(a + i q)2
(a i q)2

Cng c th kim tra nghim ny tho mn bng cch th vo (1).


V d 3.39. Bit rng dy s (xn ) c dng xn = f (n), trong f (x) l a thc
bc khng qu 2. Hy xc nh cng thc tng qut ca dy s bit ba s hng
u: x1 , x2 , x3 .
Gii. Gi s f (x) = ax2 + bx + c. Theo gi thit ta c

= x1
a + b + c
4a + 2b + c = x2

8a + 4b + c = x3
Gii h ny ta c

x1 2x2 + x3
2
5x1 8x2 + 3x3
=
2
= 3x1 3x2 + x3
=

Vy
x1 2x2 + x3 2 5x1 8x2 + 3x3
n
n + 3x1 3x2 + x3 .
2
2
l hm s cn tm.
Gii. Vit f (n) di dng.
f (n) =

f (n) = a(n 2)(n 3) + b(n 1)(n 3) + c(n 1)(n 2).


Ln lt cho n = 1,

1
x2

x
3

2, 3 ta c
= f (1) = a.(1 2)(1 3) = 2a
= f (2) = b.(2 1)(2 3) = b
= f (3) = c.(3 1)(3 2) = 2c

x1
2
b = x2
x3
c=
2
a=

96

3.6. H thc truy hi phi tuyn

Vy cng thc cn tm c dng


x1
x3
xn = f (n) = (n 2)(n 3) x2(n 1)(n 3) + (n 1)(n 2).
2
2
V d 3.40. Xc nh s hng tng qut ca dy s cho bi h thc truy hi
sau.
xn + 1
x1 = 0 ; xn+1 =
.
n+1
Gii. Vit li iu kin cho di dng.
x1 = 0 quad (1)
2x2 = x1 + 1 quad(2)
3x3 = x2 + 1 quad(3)

(n 1)xn = xn2 + 1 quad(n 1)
nxn = xn1 + 1 quad(n)
Nhn hai v ca ng thc th k trn vi (k 1)!, cng tng v cc ng thc
thu c v rt gn cc s hng ng dng hai v ta c.
n1

n!.xn = 1! + 2! + + (n 1)! xn =

1 X
k!
.
n!
k=1

l cng thc s hng tng qut cn tm.

3.6.4

Bi tp.

1. Xc nh s hng tng qut ca dy s {f (n)} (n N ) c cho bi h


thc truy hi.
n

f (1) = ; f (n + 1) = 2a2 f 2 (n) a(n+1)2

(n N) vi a > 0.

p s.
n1
n2
a

(n N) nu = a

a quadkhi n = 1

nu = a
n1

an2
khi n 2
f (n) = an2n1 cos 2n1 (n N ) nu || < a



 n1 
2n1 

p
p

1 a(n1)2n1 + 2 a2 2
2
2

+ a

(n N ) nu || > a

97

3.6. H thc truy hi phi tuyn

2. Xc nh s hng tng qut ca dy s {f (n)} (n N) c cho bi h thc


truy hi.
f (0) = ; f (n + 1) = af 3 (n) 3f (n) (n N) vi a > 0.
p s.

2
2
a

cos 3 arccos
(n N) nu ||

2
a

a 
3 n 
3 n 
p
p
1
2
2
f (n) =
a a4
+ a+ a4
n

23

(n N) nu || >

3. Xc nh s hng tng qut ca dy s {f (n)} (n N) c cho bi h thc


truy hi.
n

f (0) = ; f (n + 1) = f 3 (n) 3a3 f (n) (n N) vi a > 0.


p s.

2 a cos 3n arccos (n N) nu || 2 a

2 a

3 n 
3 n 
1 
p
p
2
2
4a
+ + 4a
f (n) =
23 n

(n N) nu || >
a

4. Xc nh s hng tng qut ca dy s {f (n)} (n N) c cho bi h thc


truy hi:
2
f (0) = ; f (n + 1) = af 3 (n) + 3f (n) (n N) vi a > 0 ; || > .
a
p s:

1
a
f (n) =

a
2

2 a
4

!3 n

+1

a
+
2

2 a
4

!3 n
(n N )
+1

5. Xc nh s hng tng qut ca dy s {f (n)} (n N) c cho bi h thc


truy hi:

n
f (0) = ; f (n + 1) = f 3 (n) + 3a3 f (n) (n N) vi || > 2 a.
p s:


3 n 
 3n 
p
p
1 
2
2
f (n) = 3n + + 4a
(n N).
+ + 4a
2

98

3.6. H thc truy hi phi tuyn

6. Xc nh s hng tng qut ca dy s {f (n)} (n N ) c cho bi h


thc truy hi:
f (1) = ; f (n + 1) = af 3 (n) + bf 2(n) + cf (n) + d (n N )
vi a > 0 ; c =

b2
b(c 3)
b
; d=
; > .
3a
9a
3a

p s:
f (n) =

b
+
3a

3n1

n1
b
( a)3 1
(n N).
3a

7. Xc nh s hng tng qut ca dy s {f (n)} (n N) c cho bi h thc


truy hi:
f (0) = ; f (n + 1) = af 3 (n) + bf 2(n) + cf (n) + d (n N).
vi a > 0 ; c =

b2 + 9a
2
b3 + 18ab)
b
; > .
; d=
2
3a
27a
a 3a

p s:

3 n 
3 n  b
p
p

1
2
2
f (n) = 3n

+ a+ a+4
a a+4
(n N).
2
a
3a
( Vi = +

b
).
3a

8. Xc nh s hng tng qut ca dy s (xn ) (n N ) c cho bi h thc


truy hi:
n+1
x1 = a ; xn+1 =
(n 1).
xn + 1
p s:
n1

X 1
xn = n! a +
.
k!
k=1

9. Xc nh s hng tng qut ca dy s (xn ) (n N ) c cho bi h thc


truy hi:
1
2
x1 = ; xn+1 =
(n 1).
2
3 xn
p s:
xn =

3.2n1 2
.
3.2n1 1

Chng 4

Phng trnh hm sai phn


bc hai
Trong chng ny, ta gii hai bi ton v phng trnh hm tuyn tnh thun
nht bc hai i vi hm tun hon v phn tun hon cng tnh; phng trnh
hm tuyn tnh thun nht bc hai i vi hm tun hon v phn tun hon
nhn tnh. Ta s gii quyt hai bi ton trn da vo kt qu ca cc bi ton v
phng trnh hm tuyn tnh bc nht c trong ti liu tham kho [3].

4.1

Hm tun hon v phn tun hon cng tnh

nh ngha 4.1. Cho hm s f (x) v tp M (M D(f )) Hm f (x) c gi


l hm tun hon trn M nu tn ti s dng a sao cho

x M ta u c x a M
f (x + a) = f (x), x M
a c gi l chu k ca hm tun hon f (x).
Chu k nh nht (nu c) trong cc chu k ca f (x) c gi l chu k c s
ca hm tun hon f (x).
nh ngha 4.2. Cho hm s f (x) v tp M (M D(f )) Hm f (x) c gi
l hm tun hon trn M nu tn ti s dng a sao cho

x M ta u c x a M
f (x + a) = f (x), x M
a c gi l chu k ca hm tun hon f (x).
99

4.2. Phng trnh hm sai phn bc hai vi hm tun hon v phn tun hon

100

Chu k nh nht (nu c) trong cc chu k ca f (x) c gi l chu k c s


ca hm tun hon f (x).
nh ngha 4.3. f (x) c gi l hm tun hon nhn tnh chu k a (a /
{0, 1, 1}) trn M nu M D(f ) v

x M a1 x M
f (ax) = f (x), x M
nh ngha 4.4. f (x) c gi l hm phn tun hon nhn tnh chu k a
(a
/ {0, 1, 1}) trn M nu M D(f ) v

x M a1 x M
f (ax) = f (x), x M

4.2

Phng trnh hm sai phn bc hai vi hm tun


hon v phn tun hon

Bi ton 4.1. Cho a R\{0}; , R, 6= 0. Tm tt c cc hm f : R R


tho mn iu kin:
f (x + 2a) + f (x + a) + f (x) = 0

(4.1)

Phng trnh c dng (1.2.1) c gi l phng trnh tuyn tnh thun nht bc
hai
Gii. Xt phng trnh
2 + + = 0

(4.2)

(gi l phng trnh c trng ca phng trnh (1.2.1))


C
4 = 2 4
a) Trng hp 4 > 0
Khi phng trnh (4.2) c hai nghim thc 1 6= 2. p dng nh l Viete:
(
1 + 2 =
12 =
thay vo (4.1)
(4.1) f (x + 2a) (1 + 2)f (x + a) + 12 f (x) = 0
f (x + 2a) 1f (x + a) = 2[f (x + a) 1f (x)]

(4.3)

101

4.2. Phng trnh hm sai phn bc hai vi hm tun hon v phn tun hon

t g1 (x) = f (x + a) 1f (x), (1.2.3) tr thnh


g1(x + a) = 2g1(x)
(
x
h(x)
t g1(x) = |2| a .h(x). Khi ta c h(x + a) =
h(x)
c
x
f (x + a) 1f (x) = |2| a h1 (x)

(1.2.3*)
nu 2 > 0
Khi ta
nu 2 < 0
(4.4)

i vai tr 2 cho 1 v bin i tng t ta c


x

f (x + a) 2f (x) = |1| a h2 (x)

(4.5)

Tr (4.4) cho (1.2.5) ta c


x

(2 1)f (x) = |2| a h1 (x) |1| a h2(x)


Vy
f (x) =

i
1 h
x
x
|2| a h1 (x) |1| a h2 (x)
2 1

trong h1 (x) v h2 (x) l hai hm tu tho mn:


(
(
nu 2 > 0
h1 (x)
h2 (x)
h1 (x + a) =
; h2 (x + a) =
h1 (x) nu 2 < 0
h2 (x)
b) Trng hp 4 = 0
Tc l
2 4 = 0 hay =

nu 1 > 0
nu 1 < 0

2
4

Khi phng trnh (1.2.2) c nghim kp 1 = 2 = . Do


2
2
f (x) = 0
4
 

h

f (x + 2a) +
f (x + a) = f (x + a) + ( )f (x)
2
2
2

(4.1) f (x + 2a) + f (x + a) +

(4.6)

t f (x + a) + 2 f (x) = g(x)

b1) Trng hp = 1 hay = 2.


2
Khi (1.2.6) tr thnh
f (x + 2a) f (x + a) = f (x + a) f (x)
f (x + a) f (x) = g(x),

(1.2.6*)

4.2. Phng trnh hm sai phn bc hai vi hm tun hon v phn tun hon

102

vi g(x + a) = g(x)
x
Ta c g(x) = (x+a)x
g(x) = x+a
a
a g(x + a) a g(x), phng trnh (1.2.6*) tr thnh
(x+a)
f (x + a) a g(x + a) = f (x) xa g(x)
t f (x) xa g(x) = h(x) ta c
f (x) = h(x) +

xg(x)
a

trong h(x) l hm tu sao cho h(x + a) = h(x).

b2) Trng hp 0 < 6= 1 hay 2 6= < 0 Bi ton quy v vic gii phng
2
trnh dng

f (x + a) + ( )f (x) = g(x)
2

vi g(x + a) = g(x)
2
Tng t vic gii (1.2.3*) ta c
x
g(x) = ( ) a h1 (x), vi h1 (x + a) = h1 (x)
2
Suy ra

x
f (x + a) ( )f (x) = ( ) a h1 (x)
2
2
f (x + a)
f (x)

( )
= h1(x)
2
x
x
( ) a
( ) a
2
2
f (x + a)
f (x)
h1 (x)

=
x+a
x

( ) a
( ) a

2
2
2
t
f (x)
h1 (x)
= I1 (x);
= h2 (x)
x
2
( 2 ) a
(4.7) I1(x + a) I1 (x) = h2(x), vi h2 (x + a) = h2 (x)

(4.7)

4.2. Phng trnh hm sai phn bc hai vi hm tun hon v phn tun hon

103

Tng t cch gii (1.2.6*) ta c


xh2(x)
a
2xh1(x)
= k1 (x)
a
xh
2xh1 (x) i
f (x) = ( ) a k1(x)
2
a
I1(x) = k1 (x) +

trong k1(x) l hm tu tho mn k1(x + a) = k1(x).


b3) Trng hp 2 < 0 hay > 0.
Bi ton quy v vic gii phng trnh
f (x + a) +

f (x) = g(x)
2

vi g(x + a) = ( 2 )g(x).
Tng t vic gii (1.2.3*) ta c
x
x
a

a
g(x) = h3 (x) =
h3 (x)
2
2

(4.8)

(4.9)

vi h3 (x + a) = h3 (x) (v 2 < 0)
T (4.8) v (4.9) ta c
x

a
h3 (x)
f (x) =
2
2
f (x + a) f (x)
  x +   x = h3 (x)
2 a
a
f (x + a) +

f (x + a)
f (x)
h3 (x)
  x+a +   x =
a

t
f (x)
2h3 (x)
= h4 (x)
  x = I2 (x);

a
2

vi h4 (x) l hm tu tho mn: h4 (x+a) = h4 (x), (v: h4 (x+a) =

2h3(x + a)
=

2h3 (x)
= h4 (x))

Khi ta c
I2(x + a) + I2 (x) = h4(x).

(1.2.9*)

4.2. Phng trnh hm sai phn bc hai vi hm tun hon v phn tun hon

104

Ta c h4 (x) = (ax)+x
h4(x) = xa h4(x + a) xa
a
a h4 (x).
V vy (1.2.9*) tr thnh k2(x) = k2 (x)
vi k2(x) = I2 (x) + xa
a h4 (x).
Suy ra
2(x a)h3(x)
I2(x) = k2 (x)
a
vi k2(x) l hm tu tho mn k2(x + a) = k2(x)
Vy
xh
2(x a)h3(x) i
f (x) = ( ) a k2(x)
2
a
c) Trng hp 4 < 0
Phng trnh (1.2.2) c hai nghim phc lin hp 1p
, 2 C; 1 = 2, do
t 1 = p iq, 2 = p + iq suy ra |0| = |1| = |2| p2 + q 2 , arg2 = =
arg1, tan = pq . Bin i tng t nh trng hp 4 > 0 ta c
g1(x + a) = 2g1(x)

(*)

vi g1(x) = f (x + a) 1f (x).
Nh vy hm: g1 : R C
Ta c
() g1(x + a) = eln 2 g1(x)
g1(x)
x
e a ln 2
e
g1(x + a)
g1(x)
x+a
= x ln
ln

2
ea 2
e a

g1(x + a)

g1(x + a)
g1(x)
= eln 2 . x ln
x
ln

2
ea
ea 2

x+a
a

ln 2

t
g1(x)
x

e a ln 2

= h1 (x)

(4.10)

Ta c
h1 (x + a) = h1 (x)
i vai tr ca 1, 2 cho nhau v bin i tng t nh trn ta c
h2 (x + a) = h2 (x)
Vi
g2(x)
x

e a ln 1

= h2 (x)

(4.11)

4.2. Phng trnh hm sai phn bc hai vi hm tun hon v phn tun hon

105

T (4.10) v (4.11) ta c
(

g1 (x) = e a ln 2 h1 (x)
x
g2 (x) = e a ln 1 h2 (x)

(i)

Ta chng minh h1 (x) = h2 (x)


Tht vy, trc ht ta chng minh g1(x) = g2 (x).
Ta c g1(x) = f (x + a) 1f (x),
Ly x0 bt k, x0 R.
Ta c g1(x0 ) = f (x0 + a) 1 f (x0) = f (x0 +a)1 f (x0 ) = f (x0 +a)2 f (x0) =
g2 (x0).
V x0 bt k nn x R ta c
g1(x) = g2(x)

(4.12)

Tip theo ta chng minh


x

e a ln 2 = e a ln 1
Tht vy
x

e a ln 2 = e a (ln |2 |+iarg2 +2ki)


x

= e a ln |2 | .eiarrg2 a ei2k a
 cos x
x
x 
2kx
2kx 
= e a ln |2 |
+ i sin
cos
+ i sin
a
a
a
a


x
x
2kx
x
2kx 
= e a ln |2 | cos
cos
+ i sin
+ i sin
a
a
a
a



x
x
x
2kx
2kx
= e a ln |2 | cos
i sin
cos
i sin
a
a
a
a
  x 
 x   2kx 
 2kx 
x
ln
|
|
2
= ea
cos
+i sin
cos
+i sin
a
a
a
a
x
x
2kx
ln
|
|
i(
)
i(
)
2
a e
x
= ea
e
x

= e a ln |1 | ei

arg1 x
a

ei

2k 0 x
a

= e a (ln |1 |+iarg1 +2k i) = e a ln 1


y: (arg1 = ; arg2 = ; k = k0)
T (4.12) v (4.13):
h g (x) i
1
e

x
a

ln 1

g2 (x)
x

e a ln 1

h1 (x) = h2 (x)

Theo trn ta c cc hm h1 : R C; h2 : R C

(4.13)

106

4.2. Phng trnh hm sai phn bc hai vi hm tun hon v phn tun hon

Nh vy ta s t: h1 (x) = m(x) + in(x); trong cc hm:


m : R R; n : R R
Theo chng minh trn: h1 (x) = h2 (x) h2 (x) = m(x) in(x)
Quay tr li bi ton ban u ta c
(
x
f (x + a) 1f (x) = e a ln 2 h1 (x)
x
f (x + a) 2f (x) = e a ln 1 h2 (x)

(4.14)

Tr (??) cho (??)


i
x
1 h x ln 2
ea
h1 (x) e a ln 1 h2 (x)
2 1
i
x
1 h x ln 2
=
ea
h1 (x) e a ln 2 h2 (x)
2 1
i
x
1 h x (ln |2 |+iarg2 +2ki)
=
ea
h1 (x) e a (ln |2 |+iarg2 +2ki) h2(x)
2 1

f (x) =

V hm e a ln 2 l hm a tr, ta s chn mt nhnh lin tc bng cch chn


k = 0, nn ta c:
i
x
1 h x (ln |2 |+iarg2)
f (x) =
ea
h1 (x) e a (ln |2|+iarg2 ) h2 (x)
2 1

i
x
1 h x ln |2 | 
x
x
x 
x 
=
ea
cos
+ i sin
h1 (x) e a ln |2 | cos
i sin
h2 (x)
2 1
a
a
a
a
x
i
h
ln
|
|
0
ea
x
x
=
cos
(h1(x) h2 (x)) + i sin
(h1 (x) + h2(x))
2iq
a
a
x
i
e a ln |0| h
x
x
=
2i cos
.n(x) + 2i sin
.m(x)
2iq
a
a
x
i
ln |0 | h
a
e
x
x
=
cos
.n(x) + sin
.m(x)
q
a
a
x h
i
a
|0|
x
x
=
cos
.n(x) + sin
.m(x)
q
a
a
Trong n(x) v m(x) l hai hm s bt k tho mn
n(x + a) = n(x); m(x + a) = m(x)
n : R R; m : R R
1 = p iq
2 = p + iq

|1| = |2| =

p2 + q 2 ; arg2 = = arg1

4.2. Phng trnh hm sai phn bc hai vi hm tun hon v phn tun hon

q
; arg2 =
p

tan =
Kt lun:
+) 4 > 0
f (x) =

i
x
x
1 h
|2| a .h1(x) |1| a .h2 (x)
2 1

h1 (x) v (h2(x) l hai hm tu tho mn:


(
nu 2
h1 (x)
h1 (x + a) =
h1 (x) nu 2
(
nu 1
h2 (x)
h2 (x + a) =
h2 (x) nu 1

>0
<0
>0
<0

+) 4 = 0
Trng hp 1: = 2 f (x) = h(x) +(xg(x)
a
h(x + a) = h(x)
h(x) v g(x) l hai hm tu tho mn:
g(x + a) = g(x)
Trng hp 2: 2 6= < 0


 xa h
2xh1 (x) i
k1(x)
2
a
(
h1 (x + a) = h1 (x)
k1(x) v h1 (x) l hai hm tu tho mn:
k1(x + a) = k1(x)
Trng hp 3: > 0
f (x) =

x h

2(x a)h2(x) i
2
a
(
h2 (x + a) = h2 (x)
k2(x) v h2 (x) l hai hm tu tho mn:
k2(x + a) = k2 (x)
+) 4 < 0
x
i
|0| a h
x
x
f (x) =
cos
.n(x) + sin
.m(x)
q
a
a
f (x) =

k2 (x)

m(x) v n(x) l hai hm tu tho mn


(
m(x + a) = m(x); m : R R
n(x + a) = n(x);
n: RR

107

4.3. Phng trnh vi hm s tun hon, phn tun hon nhn tnh

108

1 = p iq; 2 = p + iq
p
|1| = |2| = |0| = q 2 + q 2 ; arg2 = arg1 =
Cc v d p dng
V d 4.1. Tm tt c cc hm f : R R tho mn iu kin:
f (x + 2) 8f (x + 1) + 15f (x) = 16

(1)

Li gii. t f (x) = g(x) + C; (C: xc nh sau)


(1) g(x + 2) 8g(x + 1) + 15g(x) + 8C = 16.
Chn C = 2
g(x + 2) 8g(x + 1) + 15g(x) = 0.
Xt phng trnh c trng:
2 8 + 15 = 0; 4 = 1 > 0
1 = 3; 2 = 5
p dng cng thc nghim vi 4 > 0 ta c
g(x) =

1 x
1
[5 h1 (x) 3x h2 (x)] f (x) = [5x h1 (x) 3x h2 (x)] + 2
2
2

h1 (x) v h2 (x) l hai hm tu tho mn


(
h1(x + 1) = h1 (x);
h2(x + 1) = h2 (x);

h1 : R R
h2 : R R

V d 4.2. Tm tt c cc hm f : R R tho mn iu kin

4.3

Phng trnh vi hm s tun hon, phn tun


hon nhn tnh

Phng trnh hm l mt chuyn c bn ca chng trnh ton cho


cc trng THPT Chuyn. Cc bi ton v phng trnh hm cng l nhng bi
tp thng gp trong cc k thi hc sinh gii ton cp Quc gia, thi Olympic khu
vc hay Olympic Quc t. Phng trnh hm tuyn tnh bc hai l mt vn
quan trng trong lp phng trnh hm ni chung. Trong chng trnh ton cho
cc trng THPT chuyn,phng trnh hm tuyn tnh bc hai c cp trong
trng hp > 0 ca phng trnh c trng: 2 + + = 0() i vi hm

4.3. Phng trnh vi hm s tun hon, phn tun hon nhn tnh

109

tun hon cng tnh; cc trng hp = 0 v < 0 ca phng trnh () cha


c cp n. Ngoi ra, phng trnh hm tuyn tnh bc hai i vi hm tun
hon nhn tnh cha c cp n c ba trng hp: > 0; = 0v < 0
ca phng trnh (). Hn th na, phng trnh hm tuyn tnh bc hai i vi
hm tun hon cng tnh v nhn tnh cng cha c cp n. Bo co ny
a ra ba bi ca phng trnh hm tuyn tnh bc hai vi v phi l hm s i
vi hm tun hon v phn tun hon nhn tnh.

4.3.1

nh ngha

Cho a R\{0; 1; 1}; , R. Tm tt c cc hm: f : R\{0} R tha


mn iu kin:
f (a2x) + f (ax) + f (x) = g(x).
trong g(x) l hm cho trc.

4.3.2

Mt s bi ton

Bi ton 4.2. Cho h(x) l hm tun hon nhn tnh chu k a trn R(h(ax) =
h(x)); a R\{0; 1; 1}; , R. Tm tt c cc hm: f : R\{0} R tha mn
iu kin:
f (a2 x) + f (ax) + f (x) = h(x).

(4.15)

Li gii:
Xt phng trnh c trng:
2 + + = 0; = 2 4.

(4.16)

a) Trng hp > 0 :
Phng trnh (2.2.40) c hai nghim thc: 1 6= 2. p dng nh l Viete ta
c:
(
1 + 2 =
Thay vo (2.2.39):
1 2 =

(2.2.39) f (a2 x) (1 + 2)f (ax) + 12f (x) = h(x)


f (a2 x) 1f (ax) 2[f (ax) 1 f (x)] = h(x)

(4.17)

4.3. Phng trnh vi hm s tun hon, phn tun hon nhn tnh

110

t g1 (x) = f (ax) 1f (x) ta c:


(2.2.41) g1(ax) 2 g1(x) = h(x)

(4.18)

a1) Trng hp 1:1 = 1 hoc 2 = 1.


Khng mt tnh tng qut ta gi s: 2 = 1. Khi :
(2.2.42) g1(ax) g1(x) = h(x) (theo Bi ton 1.1- chng 1)
ln |x|h(x)
g1 (x) = g(x) +
trong g(x) l hm ty sao cho:
ln |a|
g(ax) = g(x). Hay:
f (ax) 1 f (x) = g(x) +

ln |x|h(x)
.
ln |a|

(4.19)

i vai tr 2 cho 1 v bin i tng t ta c:


g2(ax) 1g2 (x) = h(x)
trong :g2 (x) = f (ax) f (x).
V 2 = 1 v 1 6= 2 1 6= 1. Theo Bi ton 1.3 chng 1 ta c:
g2(x) =

h(x)
+ |x|log|a| |1 |.q(x)
1 1

trong q(x) l hm ty sao cho:


(
q(x)
q(ax) =
q(x)

nu 1 > 0
nu 1 < 0

Hay:
f (ax) f (x) =

h(x)
+ |x|log|a| |1| .q(x)
1 1

(4.20)

Tr (2.2.43) cho (2.2.44) ta c:


f (x) =


ln |x|(1 1) ln |a|
1 
g(x) + |x|log|a| |1 | q(x) .
h(x) +
2
ln |a|(1 1)
1 1

a2) Trng hp 2: 1 6= 1 v 2 6= 1 1 + + 6= 0.
(v nu 1 + + = 0 1 = 1 hoc 2 = 1 iu ny mu thun vi gi thit.)
Ta c:
1++
h(a2 x) + h(ax) + h(x)
h(x) =
h(x) =
1++
1++

111

4.3. Phng trnh vi hm s tun hon, phn tun hon nhn tnh

(2.2.41) f (a2 x)

t g(x) = f (x)

h(a2 x)
h(ax) 
+ f (ax)
+
1++
1++
h(x) 
+ f (x)
=0
1++

(4.21)

h(x)
ta c:
1++
(2.2.45) g(a2x) + g(ax) + g(x) = 0.

Theo bi ton 1.2- chng 1- trng hp > 0 ta c:


g(x) =


1  log|a| |2|
|x|
h1 (x) |x|log|a| |1 | h2 (x)
2 1

trong : 1, 2 l nghim ca phng trnh: 2 + + = 0.


h1 (x), h2(x) l hai hm ty sao cho:
(
(
nu 2 > 0
nu 1 > 0
h1(x)
h2 (x)
h1 (ax) =
; h2 (ax) =
h1 (x) nu 2 < 0
h2 (x) nu 1 < 0
T ta c:
f (x) =


h(x)
1  log|a| |2 |
|x|
h1 (x) |x|log|a| |1| h2 (x)
+
1 + + 2 1

b) Trng hp = 0.
Phng trnh c trng (2.2.40) c nghim: 1 = 2 =

2
=
2
4

2
f (x) = h(x))
4
i

f (a2 x) + f (ax) +
f (ax) + f (x) = h(x)
2
2
2

t g(x) = f (ax) + f (x) th:


2
(2.2.41) f (a2x) + f (ax) +

(2.2.46) g(ax) +

g(x) = h(x)
2

(4.22)

(4.23)

b1) Trng hp 1: = 2 : (2.2.47) g(ax) g(x) = h(x). Theo Bi ton


ln |x|h(x)
1.1- chng 1 ta c: g(x) = k(x) +
trong k(x) l hm ty sao
ln |a|

4.3. Phng trnh vi hm s tun hon, phn tun hon nhn tnh

112

cho:
k(ax) = k(x). Khi :
ln |x|h(x)
ln |a|
ln |x|h(x)
= k(x)
f (ax) f (x)
ln |a|
1  (ln |ax|)2h(ax) (ln |x|)2h(x) (ln |a|)2h(x) 
f (ax) f (x)
= k(x)
ln |a|
2 ln |a|
 (ln |ax|)2h(ax)
f (ax) f (x)

2(ln |a|)2
(ln |x|)2h(x)
ln |ax|h(ax) ln |x|h(x) 

= k(x)
2(ln |a|)2
2 ln |a|
2 ln |a|

f (ax) f (x) = k(x) +

 (ln |ax|)2 ln |a| ln |ax|


h(ax)
2(ln |a|)2
(ln |x|)2 ln |a| ln |x| 

= k(x)
2(ln |a|)2
(ln |ax|)2 ln |a| ln |ax|
f (ax)
h(ax)
2(ln |a|)2


(ln |x|)2 ln |a| ln |x|
f (x)
h(x) = k(x)
2
2(ln |a|)

f (ax) f (x)

(4.24)

(ln |x|)2 ln |a| ln |x|


h(x), khi :
2(ln |a|)2
(2.2.48) p(ax) p(x) = k(x).

t p(x) = f (x)

Theo Bi ton 1.1 chng 1 ta c: p(x) = I(x) +

ln |x|k(x)
, trong I(x) l
ln |a|

hm ty sao cho: I(ax) = I(x).


f (x) = I(x) +

ln |x|k(x) (ln |x|)2 ln |a| ln |x|


h(x).
+
ln |a|
2(ln |a|)2

b2) Trng hp 2: 6= 2; < 0 :


(2.2.47)| g(ax) +

g(x) = h(x).
2

Theo Bi ton 1.3 chng 1 ta c:


g(x) =

h(x)
2h(x)
+ |x|log|a| | 2 | q1 (x) =
+ |x|log|a| ( 2 ) q1 (x)
+1
+2

113

4.3. Phng trnh vi hm s tun hon, phn tun hon nhn tnh

trong q1 (x) l hm ty sao cho: q1 (ax) = q(x) (v < 0). Ta c:

2h(x)
f (x) =
+ |x|log|a| ( 2 )q1 (x)
2
+2

2h(x)
f (ax) + f (x)
= |x|log|a| ( 2 ) q1 (x)
2
+2

f (ax) +

(4.25)

f (ax) +

h(x) i
2 h h(ax)
f (x)
+
df
rac2
= |x|

2
+ 2 2 + 1
2 +1

log|a| (
)
2

q1 (x)
(4.26)

h 4h(ax)

4h(x) i
= |x|log|a| ( 2 ) q1 (x)
+
f (x)
2
( + 2)2
2 ( + 2)2

4h(ax)

4h(x) 
f (ax)
= |x|log|a| ( 2 ) q1 (x)
+
f (x)
2
2
( + 2)
2
( + 2)

f (ax) +

t f1 (x) = f (x)

(4.27)

4h(x)
ta c:
( + 2)2

(2.2.49) f1 (ax) +

f1 (x) = |x|log|a| ( 2 ) q1 (x).


2

p dng kt qu Bi ton 1.2-chng 1 (trng hp = 0, < 0, 6= 2) ta


c:

2 ln |x|q1(x) 
f1 (x) = |x|log|a| ( 2 ) k1(x)
ln |a|
trong : k1(x) l hm ty sao cho: k1(ax) = k1 (x). Do :
f (x) =


4h(x)
2 ln |x|q1(x) 
log|a| (
)
2
k
+
|x|
(x)

1
( + 2)2
ln |a|

b2) Trng hp 3: > 0, khi (2.2.47) g(ax) +


Theo Bi ton 1.3- chng 1 ta c:
g(x) =

g(x) = h(x)
2

h(x)
2h(x)
+ |x|log|a| | 2 | q2 (x) =
+ |x|log|a| 2 q2 (x)
+1
+2

trong q2 (x) l hm ty sao cho: q2 (ax) = q2 (x) (v > 0). T c:


f (ax) +

2h(x)
f (x) =
+ |x|log|a| 2 q2 (x)
2
+2

()

114

4.3. Phng trnh vi hm s tun hon, phn tun hon nhn tnh

Bin i tng t nh trng hp 2 trn ta c:


() f (ax)

4h(ax)

4h(x) 
= |x|log|a| 2 q2 (x)
+
f (x)
2
2
( + 2)
2
( + 2)

(4.28)

4h(x)
ta c:
( + 2)2

t f2 (x) = f (x)

(2.2.50) f2 (ax) +

f2 (x) = |x|log|a| 2 q2 (x)


2

p dng kt qu Bi ton 1.2- chng 1 (trng hp = 0, > 0) ta c:


f2 (x) = |x|log|a|

k2(x)

2 ln | xa |q2 (x) i
ln |a|

trong k2(x) l hm ty sao cho: k2(ax) = k2 (x). Vy:


f (x) =

h
2 ln | xa |q2(x) i
4h(x)
log|a|
2 k (x)
+
|x|
2
( + 2)2
ln |a|

c) Trng hp < 0 1 + + 6= 0. (Chng minh hon ton tng t nh


Bi ton 2.1.) ta c:
h(x) =

1++
h(a2 x) + h(ax) + h(x)
h(x) =
1++
1++

(V: h(a2x) = h(ax) = h(x))




h(a2)
h(ax) 
h(x) 
+ f (ax)
+ f (x)
=0
1++
1++
1++
(4.29)
h(x)
t g(x) = f (x)
ta c:
1++

(2.2.39) f (a2x)

(2.2.51) g(a2x) + g(ax) + g(x) = 0.


Phng trnh c trng: 2 + + = 0, < 0 nn c hai nghim phc lin
hp 1 , 2. Theo Bi ton (1.2)- chng 1 (trng hp < 0) ta c:
ln |x|

i
|2| ln |a| h
ln |x|
ln |x|
g(x) =
cos
n(x) + sin
m(x)
q
ln |a|
ln |a|
trong :
1 = p iq; 2 = p + iq |1| = |2| =

p2 + q 2

4.3. Phng trnh vi hm s tun hon, phn tun hon nhn tnh

115

q
tan = ; arg 2 = arg 1 =
p
m(x), n(x) l hai hm ty tha mn: m(ax) = m(x); n(ax) = n(x). T ta
c:
ln |x|
i
h(x)
|2| ln |a| h
ln |x|
ln |x|
f (x) =
+
cos
n(x) + sin
m(x)
1++
q
ln |a|
ln |a|
Bi ton 4.3. Cho h(x) l hm phn tun hon nhn tnh chu k a trn R
(h(ax) = h(x)); a R\{0; 1; 1}; , R. Xc nh tt c cc hm f :
R\{0} R tha mn iu kin:
f (a2 x) + f (ax) + f (x) = h(x).

(4.30)

Li gii:
Xt phng trnh c trng:
2 + + = 0

(4.31)

c = 2 4.
a) Trng hp > 0 : Phng trnh (2.2.53) c hai nghim thc 1 6= 2 . p
dng nh l Viete ta c:
(
1 + 2 =
thay vo (2.2.52) ta c:
12 =
(2.2.52) f (a2 x) (1 + 2)f (ax) + 12f (x) = h(x)
f (a2 x) 1f (ax) 2[f (ax) 1 f (x)] = h(x)

(4.32)

t g1(x) = f (ax) 1f (x), khi :


(2.2.54) g1(ax) 2g1 (x) = h(x).

(4.33)

a1) Trng hp 1: 1 6= 1 v 2 = 1.
(2.2.52) f (a2 x) (1 + 1)f (ax) + 1f (x) = h(x).
(2.2.55) g1(ax) g1(x) = h(x). Theo Bi ton 1.2 - chng 1 th: g1(x) =
h(x)
k(x)
. trong k(x) l hm ty sao cho: k(ax) = k(x).
2
Hay
h(x)
f (ax) 1f (x) = k(x)
.
(4.34)
2

4.3. Phng trnh vi hm s tun hon, phn tun hon nhn tnh

116

i vai tr 2 cho 1 v bin i tng t ta c:


g2 (ax) 1g2(x) = h(x) trong : g2(x) = f (ax) f (x).
V 1 6= 1 nn theo Bi ton 1.4(ii)- chng 1, ta c:
g2(x) = |x|log|a| |1 | q(x) +

h(x)
h(x)
= |x|log|a| |1| q(x)
1 1
1 + 1

trong q(x) l hm ty sao cho:


(
q(x)
q(ax) =
q(x)

nu 1 > 0
nu 1 < 0

Khi :
f (ax) f (x) = |x|log|a| |1 | q(x)

h(x)
1 + 1

(4.35)

Tr (2.2.56) cho (2.2.57) ta c:


f (x) =


h(x)
1 
k(x) |x|log|a| |1 | q(x)
+
2(1 + 1) 1 1

a2) Trng hp 2: 1 = 1 v 2 = 1
(2.2.52) f (a2x) f (x) = h(x).
(2.2.55) g1(ax) g1(x) = h(x). Theo Bi ton 1.2- chng 1 ta c:
h(x)
g1 (x) = k(x)
, trong k(x) l hm ty tha mn: k(ax) = k(x).
2
Vy:
h(x)
f (ax) + f (x) = k(x)
(4.36)
2
i vai tr ca 1 v 2 v bin i tng t ta c:
g2 (ax) + g2(x) = h(x) trong g2 (x) = f (ax) f (x).
Theo Bi ton 1.4(i)- chng 1 ta c: g2 (x) = q(x)
ty sao cho: q(ax) = q(x).
Hay:
f (ax) f (x) = q(x)
Tr (2.2.58) cho (2.2.59) c: f (x) =

ln | xa |h(x)
vi q(x) l hm
ln |a|

ln | xa |h(x)
ln |a|

ln | ax3 |
k(x) q(x)
+
h(x).
2
4 ln |a|

a3) Trng hp 3: 1 6= 1; 2 = 1.
(2.2.52) f (a2 x) (1 1)f (ax) 1f (x) = h(x).

(4.37)

4.3. Phng trnh vi hm s tun hon, phn tun hon nhn tnh

117

(2.2.55) g1(ax) + g1(x) = h(x), theo Bi ton 1.4(i)- chng 1 ta c:


ln | xa |h(x)
g1 (x) = q1 (x)
, trong q1 (x) l hm ty sao cho:q1 (ax) = q1 (x).
ln |a|
Vy:
ln | xa |h(x)
f (ax) f (x) = q1 (x)
(4.38)
ln |a|
i vai tr 1 cho 2 v bin i tng t ta c:
g2 (ax) 1g2(x) = h(x), trong g2 (x) = f (ax) + f (x).
V 1 6= 1 nn theo Bi ton 1.4(ii)- chng 1 c:
g2 (x) = |x|log|a| |1 | q2 (x)
trong q2 (x) l hm ty sao cho:
(
q2 (x)
q2 (ax) =
q2 (x)

h(x)
1 + 1

nu 1 > 0
nu 1 < 0

hay:
f (ax) + f (x) = |x|log|a| |1| q2 (x)

h(x)
.
1 + 1

(4.39)

Tr (2.2.61) cho (2.2.60) ta c:


x1+1
|
a1+2 h(x) + 1 |x|log|a| |1 | q (x) q (x).
f (x) =
2
1
ln |a|(1 + 1)2
1 + 1
ln |

a4) Trng hp 4: 1 6= 1, 2 6= 1 1 + + 6= 0.
1++
h(a2 x) + h(ax) + h(x)
Ta c: h(x) =
h(x) =
1++
1++
2
(V: h(a x) = h(ax) = h(x).)

h(a2 x)
h(ax) 
+ f (ax)
+
1++
1++

h(x) 
+ f (x)
=0
1++

(2.2.52) f (a2x)

(4.40)

h(x)
ta c: (2.2.62) g(a2x) + g(ax) + g(x) = 0.
1++
Phng trnh c trng: 2 + + = 0 ta c: > 0 nn c hai nghim thc
t g(x) = f (x)

118

4.3. Phng trnh vi hm s tun hon, phn tun hon nhn tnh

phn bit 1, 2.
p dng kt qu Bi ton 1.2- chng 1 (trng hp > 0) ta c:
i
1 h log|a| |2|
g(x) =
|x|
h1 (x) |x|log|a| |1 | h2 (x)
2 1
trong h1 (x), h2(x) l hai hm ty tha mn:
(
(
h1 (x)
h2(x)
nu 2 > 0
h1 (ax) =
, h2 (ax) =
h1 (x) nu 2 < 0
h2 (x)

nu 1 > 0
nu 1 < 0

Vy:
f (x) =

i
h(x)
1 h log|a| |2 |
|x|
h1 (x) |x|log|a| |1| h2 (x)
+
1 + + 2 1

b) Trng hp = 0
Phng trnh (2.2.53) c nghim: 1 = 2 =

2
=
.
2
4

2
(2.2.52) f (a2 x) + f (ax) +
f (x) = h(x)
4

f (a2 x) + f (ax) + [f (ax) + f (x)] = h(x)


2
2
2

t g(x) = f (ax) + f (x) th:


2

(2.2.63) g(ax) + g(x) = h(x).


2

(4.41)

(4.42)

b1) Trng hp 1: = 2, khi : (2.2.64) g(ax) g(x) = h(x).


h(x)
Theo Bi ton 1.2- chng 1 ta c: g(x) = k(x)
trong k(x) l hm
2
ty tha mn: k(ax) = k(x). Ta c:
h(x)
h(x)
f (ax) f (x) +
= k(x)
2
2
1 h(x) h(ax)
f (ax) f (x) + [

] = k(x)
2 2
2

f (ax) f (x) = k(x)

f (ax)

h(ax)
h(x)
[f (x)
] = k(x)
4
4

(4.43)

h(x)
, khi : (2.2.65) p(ax) p(x) = k(x). Theo Bi
4

ton 1.1 - chng 1 ta c:


t p(x) = f (x)

119

4.3. Phng trnh vi hm s tun hon, phn tun hon nhn tnh

p(x) = q(x) +

ln |x|k(x)
trong q(x) l hm ty tha mn: q(ax) = q(x). Vy:
ln |a|
f (x) = q(x) +

h(x) ln |x|k(x)
+
.
4
ln |a|

b2) Trng hp 2: = 2.
Khi : (2.2.64) g(ax) + g(x) = h(x). Theo Bi ton 1.4(i)- chng 1 ta c:
ln | xa |h(x)
g(x) = g1(x)
vi g1(x) l hm ty : g1(ax) = g1 (x). Ta c:
ln |a|
ln | xa |h(x)
ln |a|
x
ln | a |h(x)
f (ax) + f (x) +
= g1 (x)
ln |a|
ln | xa |h(x)
f (ax) + f (x) +
h(x) = g1 (x)
ln |a|
1 (ln |ax|)2h(ax) + (ln |x|)2h(x) + (ln |a|)2h(x)
f (ax) + f (x) +
ln |a|
2 ln |a|
x
 ln |x|h(ax) ln | a |h(x) 

= g1 (x)
ln |a|
ln |a|
(ln |ax|)2h(ax) (ln |x|)2h(x)
f (ax) + f (x)

2(ln |a|)2
2(ln |a|)2
h(x) ln |x|h(ax) ln | xa |h(x)

+
+
= g1(x).
2
ln |a|
ln |a|
(ln |ax|)2h(ax) (ln |x|)2h(x) ln |x|h(ax) ln | xa |h(x)
f (ax) + f (x)

+
+

2(ln |a|)2
2(ln |a|)2
ln |a|
ln |a|
1  ln |x|h(ax) ln | xa |h(x) 

= g1(x)
2
ln |a|
ln |a|
3 ln |a| ln |x| (ln |ax|)2
f (ax) +
h(ax)+
2(ln |a|)2

f (ax) + f (x) = g1(x)

+ f (x) +

3 ln |a| ln | xa | (ln |x|)2


h(x) = g1(x).
2(ln |a|)2

(4.44)

3 ln |a| ln | xa | (ln |x|)2


h(x) ta c:
2(ln |a|)2
(2.2.66) f1 (ax) + f1 (x) = g1 (x). Theo Bi ton 1.4(ii)- chng 1 ta c:

t f1 (x) = f (x) +

120

4.3. Phng trnh vi hm s tun hon, phn tun hon nhn tnh

f1 (x) = g2 (x)

ln | xa |g1(x)
, g1(x) tha mn: g2(ax) = g2 (x). Vy:
ln |a|

f (x) = g2 (x)

ln | xa |g1(x) 3 ln |a| ln | xa | (ln |x|)2

h(x).
ln |a|
2(ln |a|)2

b3) Trng hp 3: < 0, 6= 2 6= 1.


2

g(ax) + g(x) = h(x). Theo Bi ton 1.4(ii)- chng 1 ta c:


2

h(x)
+ |x|log|a| | 2 |q1 (x)
1

2h(x)
=
+ |x|log|a| ( 2 ) q1 (x)
2

g(x) =

trong q1 (x) l hm ty tha mn: q1 (ax) = q1 (x).(V: < 0). Nh vy:

2h(x)
f (x) =
+ |x|log|a| ( 2 ) q1 (x)
2
2

2h(x)
f (ax) + f (x)
= |x|log|a| ( 2 ) q1 (x)
2
2

2  h(ax) h(x) 
f (ax) + f (x)
+
= |x|log|a| ( 2 ) q1 (x)
2
2 2 1 2 2 1

f (ax) +

f (ax)

4h(x)

4h(x) 
= |x|log|a| ( 2 ) q1 (x)
+ f (x)
2
2
( 2)
2
( 2)

(4.45)

4h(x)

, (2.2.67) g1 (ax) + g1(x) = |x|log|a| ( 2 ) q1 (x).


( 2)2
2
Theo Bi ton 1.2- chng 1 (trng hp = 0; < 0; 6= 2) ta c:

t g1 (x) = f (x)

2 ln |x|q1(x) 
g1(x) = |x|log|a| ( 2 ) q1 (x) p1(x)
ln |a|
trong p1 (x) l hm ty : p1 (ax) = p1(x). Vy:
f (x) =


4h(x)
2 ln |x|q1(x) 
+ |x|log|a| ( 2 ) p1(x)
.
2
( 2)
ln |a|

b4) Trng hp 4: > 0; 6= 2 6= 1 Khi :


2

g(ax) + g(x) = h(x). Theo Bi ton 1.4(ii)- chng 1:


2
g(x) =

h(x)
2h(x)
+ |x|log|a| | 2 | q2 (x) =
+ |x|log|a| 2 q2 (x)
1
2

4.3. Phng trnh vi hm s tun hon, phn tun hon nhn tnh

121

vi q2 (x) l hm ty tha mn: q2 (ax) = q2 (x) (V: > 0). Do :


f (ax) +

2h(x)
f (x) =
+ |x|log|a| | 2 | q2 (x)
2
2

()

Bin i tng t nh trng hp 3 ta c:


() f (ax)
t g2 (x) = f (x)

4h(ax)

4h(x) 
= |x|log|a| 2 q2 (x)
+
f (x)
2
2
( 2)
2
( 2)

(4.46)

4h(x)
ta c:
( 2)2

(2.2.68) g2(x) +

g2(x) = |x|log|a| 2 q2 (x).


2

Theo bi ton 1.2- chng 1(trng hp = 0; > 0; 6= 2):


g2(x) = |x|log|a|


2 ln | xa |q2 (x) 
p2 (x)
,
ln |a|

trong p2 (x) l hm ty tha mn: p2 (ax) = p2(x). T ta c:


f (x) =


2 ln | xa |q2 (x) 
4h(x)
log|a|
2 p (x)
+
|x|
.
2
( 2)2
ln |a|

c) Trng hp: < 0 D dng chng minh c 1 + 6= 0. Ta c:


h(x) =

1+
h(a2 x) + h(ax) + h(x)
h(x) =
1+
1+

(V: h(a2x) = h(ax) = h(x))


(2.2.52) f (a2 x)

t g(x) = f (x)


h(a2)
h(ax) 
+ f (ax)
+
1+
1+

h(x) 
+ f (x)
=0
1+

(4.47)

h(x)
ta c:
1+
(2.2.69) g(a2x) + g(ax) + g(x) = 0.

Phng trnh c trng: 2 + + = 0, < 0 nn c hai nghim phc lin


hp 1 , 2. Theo Bi ton (1.2)- chng 1 (trng hp < 0) ta c:
ln |x|

i
|o | ln |a| h
ln |x|
ln |x|
g(x) =
cos
n(x) + sin
m(x)
q
ln |a|
ln |a|

4.3. Phng trnh vi hm s tun hon, phn tun hon nhn tnh

trong :
1 = p iq; 2 = p + iq |o| = |1| = |2| =
q
tan = ; arg 2 = arg 1 =
p

122

p2 + q 2

m(x), n(x) l hai hm ty tha mn: m(ax) = m(x); n(ax) = n(x). T ta


c:
ln |x|
i
h(x)
|o | ln |a| h
ln |x|
ln |x|
f (x) =
+
cos
n(x) + sin
m(x) .
1+
q
ln |a|
ln |a|
Nhn xt 2.2 Sau khi th li hai bi ton trn ta nhn thy: Trong biu thc
nghim ca tt c cc trng hp, phn biu thc c cha h(x) l nghim ring
ca phng trnh: f (a2 x) + f (ax) + f (x) = h(x).
Bi ton 4.4. Cho g(x) l hm tun hon nhn tnh chu k a, (g(ax) = g(x)); h(x)
l hm phn tun hon nhn tnh chu k a, (h(ax) = h(x)); a R\{0, 1, 1}; ,
R. Xc nh tt c cc hm: f : R\{0} R tha mn iu kin:
f (a2 x) + f (ax) + f (x) = g(x) + h(x).

(4.48)

Li gii:
Xt phng trnh c trng:
2 + + = 0, = 2 4.

(4.49)

a) Trng hp > 0: Phng trnh c trng c hai nghim thc: 1 6= 2.


a1) Trng hp 1: 1 6= 1; 2 = 1.
p dng nh l Viete:
(
1 + 1 =
1 =
(2.2.70) f (a2 x) (1 + 1)f (ax) + 1f (x) = g(x) + h(x)

(4.50)

Xt phng trnh:
f1 (a2 x) (1 + 1)f1 (ax) + 1f1 (x) = g(x).

(4.51)

p dng kt qu ca Bi ton 2.4 (trng hp a1) v nhn xt 2.2 ta c biu


ln |x|(1 1) ln |a|
thc:
g(x) l nghim ring ca (2.2.73). Thay vo ta c:
ln |a|(1 1)2
ln |a2 x|(1 1 ) ln |a|
ln |ax|(1 1) ln |a|
g(a2x) (1 + 1)
g(ax)+
2
ln |a|(1 1)
ln |a|(1 1)2
(4.52)
ln |x|(1 1 ) ln |a|
+ 1
g(x) = g(x)
ln |a|(1 1)2

123

4.3. Phng trnh vi hm s tun hon, phn tun hon nhn tnh

Thay (2.2.74) vo (2.2.72):


ln |a2x|(1 1) ln |a|
g(a2x)
ln |a|(1 1)2


ln |ax|(1 1) ln |a|
(1 + 1) f (ax)
g(ax) +
2
ln |a|(1 1)


ln |x|(1 1) ln |a|
+ 1 f (x)
g(x)
= g(x)
ln |a|(1 1 )2

(2.2.72) f (a2 x)

t f2 (x) = f (x)

(4.53)

ln |x|(1 1) ln |a|
g(x) (i) ta c:
ln |a|(1 1)2

(2.2.75) f (a2 x) (1 + 1)f2(ax) + 1f2 (x) = h(x).


p dng kt qu Bi ton 2.5 (trng hp a1) ta c:
f2 (x) =


h(x)
1 
k(x) |x|log|a| |1| q(x)
+
2(1 + 1) 1 1

trong k(x); q(x) l hai hm ty tha mn:


(
q(x)
k(ax) = k(x); q(ax) =
q(x)

(ii)

nu 1 > 0
nu 1 < 0

T (i) v (ii) ta c:
f (x) =


h(x)
1 
ln |x|(1 1) ln |a|
k(x) |x|log|a| |1 | q(x) .
g(x) +
+
2
2(1 + 1)
ln |a|(1 1)
1 1

a2) Trng hp 2: 1 = 1; 2 = 1.
Bin i tng t nh trng hp a1) ta c:
2

ln | ax3 |
ln | xa |
k(x) q(x)
f (x) =
g(x) +
h(x) +
4
4 ln |a|
ln a
2
trong k(x), q(x) l hai hm ty tha mn: k(ax) = k(x); q(ax) = q(x).
a3) Trng hp 3: 1 6= 1; 2 = 1.
Bin i tng t nh trng hp a1) ta c:
1 +1

f (x) =

ln | xa1 +2 |h(x)
ln |a|(1 + 1)2

i
h
g(x)
+ |x|log|a| |1 | q2 (x) q1 (x)
2(1 1 )

4.3. Phng trnh vi hm s tun hon, phn tun hon nhn tnh

vi q1 (x), q2(x) l hai hm ty tha mn:


(
q2 (x)
q1 (ax) = q1 (x); q2 (ax) =
q2 (x)

124

nu 1 > 0
nu 1 < 0

a4) Trng hp 4: 1 6= 1; 2 6= 1.
Bin i tng t nh trng hp a1) ta c:
f (x) =


1  log|a| |2 |
h(x)
g(x)
+
|x|
h1 (x) |x|log|a| |1 | h2 (x)
+
1 + 1 + + 2 1

trong h1 (x), h2(x) l hai hm ty tha mn:


(
(
nu 2 > 0
h1 (x)
h2 (x)
h1 (x) =
h2 (x) =
h1 (x) nu 2 < 0
h2 (x)

nu 1 > 0
nu 1 < 0

b) Trng hp = 0
b1) Trng hp 1: = 2
Bin i tng t nh trng hp a1) ta c:
f (x) =

h(x) (ln |x|)2 ln |a| ln |x|


ln |x|k(x)
h(x) + q(x) +
+
2
4
2(ln |a|)
ln |a|

Vi k(x), q(x) l hai hm ty tha mn: k(ax) = k(x), q(ax) = q(x).


b2) Trng hp 2: = 2
Bin i tng t nh trng hp a1) ta c:
f (x) =

3 ln |a| ln | xa | (ln |x|)2


ln | xa |g1(x)
g(x)
h(x)
+
(x)

+
g
2
2(ln |a|)2
4
ln |a|

vi g1(x), g2(x) l hai hm ty tha mn: g1 (ax) = g1(x); g2(ax) = g2 (x).


b3) Trng hp 3: < 0; 6= 2
Bin i tng t nh trng hp a1) ta c:

4h(x)
4g(x)
2 ln |x|q1(x) 
+
+ |x|log|a| ( 2 ) p1 (x)
,
2
2
( 2)
( + 2)
ln |a|

trong : p1(x), q1(x) l hai hm ty : p1 (ax) = p1(x); q1(ax) = q1 (x)


b4) Trng hp 4: > 0; 6= 2
Bin i tng t nh trng hp a1) ta c:

2 ln | xa |q2 (x) 
4h(x)
4g(x)
log|a|
2 p (x)
+
+
|x|
2
( 2)2 ( + 2)2
ln |a|

4.3. Phng trnh vi hm s tun hon, phn tun hon nhn tnh

125

trong : p2(x), q2(x) l hai hm ty : p2 (ax) = p2 (x); q2(ax) = q2 (x)


c) Trng hp < 0 Bin i tng t nh trng hp a1) ta c:
ln |x|

i
h(x)
g(x)
|o| ln |a| h
ln |x|
ln |x|
f (x) =
+
+
cos
n(x) + sin
m(x)
1+ 1++
q
ln |a|
ln |a|
trong :
2
1 = p iq; 2 = p + iq
pl nghim phng trnh: + + = 0.
2
2
|o | = |1| = |2| = p + q
q
tan = ; arg 2 = arg 1 = ; m(x), n(x) l hai hm ty tha mn: m(ax) =
p
m(x); n(ax) = n(x).

4.3.3

Mt s v d p dng

V d 4.3. Cho g(x) l hm tun hon nhn tnh chu k 3, (g(3x) = g(x)); h(x)
l hm phn tun hon nhn tnh chu k 3, (h(3x) = h(x)). Xc nh tt c cc
hm f : R R sao cho:
f (9x) 7f (3x) + 10f (x) = 5g(x) + 21h(x).
Li gii:
Xt phng trnh c trng: 2 7 + 10 = 0 1 = 2; 2 = 5.
p dng Bi ton 2.6 (trng hp a4) ta c:
f (x) =


h(x) g(x) 1  log3 5
h1 x |x|log3 2 h2x
+
+ |x|
18
4
3

trong h1 , h2 l hai hm ty : h1 (3x) = h1 (x); h2(3x) = h2(x)


1
1
V d 4.4. Cho g(x) l hm tun hon nhn tnh chu k , (g( x) =
3
3
1
1
g(x)); h(x) l hm phn tun hon nhn tnh chu k , (h( x) = h(x)). Xc
3
3
nh tt c cc hm: f : R\{0} R sao cho:
1
1
f ( x) + 2f ( x) + f (x) = 4g(x) 7h(x) + 13.
9
3
Li gii:
t g1 (x) = 4g(x) + 13 g1( 13 x) = g1(x).
Xt phng trnh c trng: 2 + 2 + 1 = 0, = 0 1 = 2 = 1 hn na
= 2, p dng bi ton 2.6 trng hp b2) ta c:
f (x) =

4g(x) + 13 ln 9 ln |3x| (ln |x|)2


ln |3x|g1(x)
7h(x) +

+ g2 (x),
4
2(ln 3)2
ln 3

g1 (x), g2(x) l cc hm ty tha mn: g1 ( 13 x) = g1(x); g2( 13 x) = g2 (x).

4.3. Phng trnh vi hm s tun hon, phn tun hon nhn tnh

126

V d 4.5. Cho hm g(x) l hm tun hon nhn tnh chu k e, (g(ex) =


g(x)); h(x) l hm phn tun hon nhn tnh chu k e, (h(ex) = h(x)). Xc
nh tt c cc hm: f : R\{0} R sao cho:

f (e2x) 2 3f (ex) + 4f (x) = h(x) 3g(x).


Li gii:

Xt phng trnh c trng: 2 2 3 + 4 = 0, = 1 < 0 phng trnh c cc


nghim:

1 = 3 i; 2 = 3 + i; r = |1| = |2| = 2; q = 1; cos =


= .
2
6
p dng Bi ton 2.6 (phn c) ) ta c:
f (x) =



h(x)
ln |x|
ln |x|
3g(x)

+ 2ln |x| cos
n(x) + sin
m(x) ,
6
6
5+2 3 52 3

trong m(x), n(x) l hai hm tha mn: m(ex) = m(x); n(ex) = n(x).

Ti liu tham kho


[1] Nguyn Thy Thanh (1990), L thuyt hm bin phc mt bin, NXB i
hc v Trung hc Chuyn nghip.
[2] Nguyn Vn Mu (2004, 2006), a thc i s v phn thc hu t, NXB
Gio dc.
[3] Nguyn Vn Mu (2003), Phng trnh hm, NXB Gio dc.
[4] L nh Thnh v cc tc gi khc, Phng trnh sai phn v mt s ng
dng, NXB Gio dc.
[5] Nguyn Trng Tun, Bi ton hm s qua cc k thi Olimpic, NXB Gio
dc.
[6] Kuczma Marek (1968), Functional equations in a single variable, PWN- Polish scientific publishers.

127

Chng 5

Dy s sinh bi hm s
5.1

Hm s chuyn i php tnh s hc v i s

Trong mc ny, ta kho st mt s tnh cht c bn ca mt s dng hm s


thng qua cc h thc hm n gin. Ta cng kho st mt s dng hm bo ton
v chuyn i cc tnh cht c bn ca php tnh i s nh giao hon, phn b
v kt hp.
Bi ton 1. Xc nh cc hm s f (x) xc nh v lin tc trn R tho mn
iu kin
f (x + y) = f (x) + f (y) + f (x)f (y), x, y R.
(1)
Gii.
t f (x) = g(x) 1, ta thu c
g(x + y) 1 = g(x) 1 + g(y) 1 + [g(x) 1][g(y) 1], x, y R
hay
g(x + y) = g(x)g(y), x, y R.

(2)

Do f (x) lin tc trn R nn g(x) cng l hm lin tc trn R. Suy ra (2) c


nghim g(x) = eax, a R v (1) c nghim
f (x) = eax 1, a R.
Bi ton 2. Cho hm s F (u, v) (u, v R). Gi s phng trnh hm:
f (x + y) = F [f (x), f (y)], x, y R

(3)

c nghim f (x) xc nh v lin tc trn R. Chng minh rng F (u, v) l hm i


xng (F (u, v) = F (v, u) v c tnh kt hp
F [F (u, v), w] = F [u, F (v, w)], u, v, w =f.
128

(4)

5.1. Hm s chuyn i php tnh s hc v i s

129

Gii.
Nhn xt rng tnh i xng ca F (u, v) c suy trc tip t (3). Mt khc,
theo (3), ta c
f (x + y + z) = f [(x + y) + z] = F {F [f (x), f (y)], f (z)}, x, y, z R

(5)

v
f (x + y + z) = f [x + (y + z)] = f [(y + z) + x] = F {F [f (y), f (z)], f (x)}
= F {f (x), F [f (y), f (z)]}, x, y, z R.

(6)

T (5) v (6) suy ra (4):


F [F (u, v), w] = F [u, F (v, w)], u, v, w =f.
Bi ton 3. Gi s phng trnh hm:
f (x + y) = F [f (x), f (y)], x, y R
vi hm s F (u, v) (u, v R) l mt a thc (khc hng), c nghim f (x) xc
nh v lin tc (khc hng) trn R. Chng minh rng F (u, v) c dng
F (u, v) = auv + bu + bv + c.

(7)

Gii.
Gi s F (u, v) l a thc bc m theo u v bc n theo v. Khi , do F (u, v)
i xng nn m = n. Theo (4) th
F [F (u, v), w] = F [u, F (v, w)], u, v, w =f
nn v tri l mt a thc bc n theo w cn v phi l a thc bc n2 theo w.
Suy ra n2 = n hay n = 1. Vy F (u, v) c dng
F (u, v) = auv + b1u + b2v + c.
Do F (u, v) l a thc i xng nn b1 = b2 v
F (u, v) = auv + bu + bv + c.
Nhn xt rng, vi F (u, v) = auv + bu + bv + c v F (u, v) tho mn iu kin
(4) th
ac = b2 b.
Vy vi a 6= 0 th
ac = b2 b c =

b2 b
, a 6= 0.
a

(8)

130

5.1. Hm s chuyn i php tnh s hc v i s

By gi, ta chuyn sang xt cc dng c bit ca (7).


Bi ton 4. Cho a thc F (u, v) = bu + bv + c, b 6= 0. Xc nh cc hm s f (x)
xc nh v lin tc trn R tho mn iu kin
f (x + y) = F [f (x), f (y)], x, y R
tc l
f (x + y) = bf (x) + bf (y) + c, x, y R.

(9)

Gii.
Nhn xt rng, nu b 6= 1 th t (9) vi y = 0, ta c ngay f (x) = const. Khi
1
1
b = v c = 0 th mi hm hng u tho mn (8). Khi b = v c 6= 0 th (9) v
2
2
1
c
nghim. Cc trng hp khc (b 6= 1, b 6= th nghim ca (9) l f (x) =
.
2
1 2b
Xt trng hp b = 1. Khi (9) c dng
f (x + y) = f (x) + f (y) + c, x, y R
v phng trnh hm ny c nghim f (x) = x c.
b2 b
, a 6= 0. Xc nh cc
a
hm s f (x) xc nh v lin tc trn R tho mn iu kin

Bi ton 5. Cho a thc F (u, v) = auv + bu + bv +

f (x + y) = F [f (x), f (y)], x, y R
tc l
f (x + y) = af (x)f (y) + bf (x) + bf (y) +

b2 b
, x, y R.
a

(10)

Gii.
Nhn xt rng, nu t
f (x) =

h(x) b
a

th th t (10) ta nhn c
h(x + y) = h(x)h(y), x, y R
v phng trnh hm ny c nghim h(x) = ex . Suy ra nghim ca (10) c dng
f (x) =

ex b
.
a

Bi ton 6. Gi s f (x) l nghim ca phng trnh hm:


f (ax + by + c) = Af (x) + Bf (y) + C (abAB 6= 0), x, y R

(11)

5.1. Hm s chuyn i php tnh s hc v i s

131

Chng minh rng hm s g(x) = f (x) f (0) tho mn phng trnh Cauchy
g(x + y) = g(x) + g(y), x, y R.
Gii.
u
vc
u
c
vc
Ln lt t x = , y =
; x = , y = ; x = 0, y =
; x = 0,
a
b
a
b
b
c
y = vo (11), ta thu c cc ng thc
b
u
 v c
f (u + v) = Af
+ Bf
+ C,
a
b
u
 c
f (u) = Af
+ Bf
+ C,
a
b
v c
f (v) = Af (0) + Bf
+ C,
b
 c
f (0) = Af (0) + Bf
+ C.
b
Suy ra
f (u + v) = f (u) + f (v) f (0).
T y suy ra iu phi chng minh.
Bi ton 7. Gi s hm s f (x) lin tc trn R l nghim ca phng trnh
hm:
f (ax + by + c) = Af (x) + Bf (y) + C (abAB 6= 0), x, y R.

(11)

Chng minh rng khi A = a, B = b.


Gii.
Tht vy, nghim ca
g(x + y) = g(x) + g(y), x, y R
trong lp cc hm lin tc l hm tuyn tnh g(x) = x. Do vy, nghim f (x) c
dng f (x) = x + . Th vo (11), ta thu c A = a, B = b v
c C = (a + b 1).

(12)

Bi ton 8. Gii v bin lun phng trnh hm sau trong lp cc hm s f (x)


lin tc trn R:
f (ax + by + c) = Af (x) + Bf (y) + C (abAB 6= 0), x, y R.
Gii.

(11)

5.1. Hm s chuyn i php tnh s hc v i s

132

Theo Bi ton 7, th iu kin cn phng trnh hm (11) c nghim l


a = A, b = B.
Gi s iu kin ny c tho mn. Theo (12), ta chia cc trng hp ring
kho st.
Xt cc trng hp sau:
Trng hp b + a = 1, c = 0.
Khi , (11) c dng
f (ax + (1 a)y) = af (x) + (1 a)f (y) (abAB 6= 0), x, y R.

(13)

Ta thu c (13) thuc lp hm chuyn tip cc i lng trung bnh cng. V


vy (13) c nghim f (x) = x + , , R.
Trng hp b + a = 1, c 6= 0.
Khi , (11) c dng
f (ax + (1 a)y + c) = af (x) + (1 a)f (y) + C (abAB 6= 0), x, y R. (13)
t f (x) =

C
x + h(x). Ta thu c (13) di dng
c
h(ax + (1 a)y + c) = ah(x) + (1 a)h(y), x, y R.

(14)

D kim tra, phng trnh (14) ch c nghim hng tu (xem (12)) v v vy,
C
(13) c nghim f (x) = x + , R.
c
Trng hp b + a 6= 1. Theo Bi ton 6 th nghim ca (13) c dng f (x) =
x + . Theo (12) th c C = (a + b 1). Vy nu cho R gi tr tu th
c C
=
.
a+b1
Ch
Nu khng i hi nghim ca (11) l hm s lin tc trn R th cc ng
thc a = A, b = B v (12) c th khng tho mn. Tuy nhin, ta vn c cc tnh
cht i s sau y.
Bi ton 9. Gi s phng trnh hm
f (ax + y) = Af (x) + f (y) (aA 6= 0), x, y R

(15)

c nghim khc hng. Chng minh rng nu a (hoc A) l s i s vi a thc


ti tiu Pa (t)(tng ng PA (t)) th A (tng ng a) l s i s v
Pa (t) PA (t).

(16)

133

5.1. Hm s chuyn i php tnh s hc v i s

Gii.
Ta thy f (0) = 0 nn f (ax) = af (x) v bng quy np ton hc, d dng chng
minh
f (ak x) = Ak f (x), k N.
(17)
Gi s
Pa (t) = tn +

n1
X

ri ti , r0, . . . , rn1 Q.

i=0

Khi , theo (17) th


n1
n1
h
i
X
X
f (an +
rai )x = f (an x) +
ri f (ai x)
i=0

i=0

An +

n1
X


i
f (x).
rA

i=0

V f (x) khc hng nn


An +

n1
X

i
rA
=0

(18)

i=0

v v vy A l s i s. Suy ra Pa (t) l c ca PA (t) v do PA (t) l a thc


ti tiu nn c (16).
Ngc li, nu A l s i s tho mn (18) th thc hin quy trnh ngc li,
ta thu c
n1
X
an +
rai = 0
(19)
i=0

v t suy ra (16).
Bi ton 10. Gi s phng trnh hm
f (ax + y) = Af (x) + f (y) (aA 6= 0, a Q), x, y R

(20)

c nghim khc hng. Chng minh rng khi a = A.


Gii.
Tht vy, theo Bi ton 9 th Pa (t) l a thc bc nht v v vy PA (t) cng
l a thc bc nht (vi h s bc cao nht u bng 1) nn a = A.
Bi ton 11. Gii phng trnh hm sau trong lp cc hm s f (x) lin tc trn
R:
f (x + y) = axy f (x)f (y) (a > 0), x, y R.
(21)
Gii.

134

5.1. Hm s chuyn i php tnh s hc v i s

D thy f (1) > 0. Nu f (1) = 0 th t (21) ta c ngay f (x) 0. Xt trng


hp f (1) > 0. Bng quy np, d dng kim chng h thc
(n2 n)x2
2

f (nx) = a

[f (x)]n, n N.

Vy vi x = 1 th
f (n) = a
Vi x =

n2 n
2

[f (1)]n, n N.

m
, ta thu c
n
f (m) = a

2
(n2 n)( m
n)
2

v
f (m) = a
Suy ra
f

m
n

m2 m
2

f(

m in
) , m, n N.
n

[f (1)]m, m N .

h 1
im
1 m 2
n
= a 2 ( n ) a 2 f (1) .

(22)

Do f (1) > 0 nn c th vit


1
c = + loga f (1).
2
T (21) suy ra
1

f (x) = a 2 x

2 +cx

, x Q+ .

(23)

Do f (x) lin tc nn (16) tho mn vi mi x R . Vi x < 0, ta t x = y


v do f (0) = 1 nn t gi thit (21) ta nhn c
2

1 = ax f (x)a(x

2 /2)cx

hay
1

f (x) = a 2 x

2 +cx

, x R.

Nhn xt.
Bng cch t
f (x) = ax

2 /2

g(x)

ta a (15) v dng quen bit


g(x + y) = g(x)g(y), x R.
Bi ton 12. Xc nh cc hm s f xc nh v lin tc trn R tho mn iu
kin
f (x + y) + f (z) = f (x) + f (y + z), x, y, z R.
(1)

5.2. V cc dy s xc nh bi dy cc phng trnh

135

Gii.
t f (0) = a th vi z = 0 trong (1) ta thu c
f (x + y) + a = f (x) + f (y), x, y R.

(2)

t f (x) = g(x) + a. T (2) ta nhn c


g(x + y) = g(x) + g(y), x, y R.

(3)

Phng trnh (3) c nghim g(x) = x, R.


Suy ra phng trnh (1) c nghim
f (x) = x + , , R.
Th li, ta thy hm f (x) = x + tho mn iu kin bi ra.
Bi ton 13. Xc nh cc hm s f xc nh v lin tc trn R tho mn iu
kin
f (x + y)f (z) = f (x)[f (y) + f (z)], x, y, z R.
(4)
Gii.
Thay y = z = 0 trong (4), ta thu c f (0)f (x) = 0. Vy f (0) = 0. Vi z = 0
th
f (x + y)f (0) = f (x)[f (y) + f (0)], x, y R
hay
f (x)f (y) = 0, x, y R.
Suy ra f (x) 0.

5.2

V cc dy s xc nh bi dy cc phng trnh

Trong ton hc, c rt nhiu trng hp ta khng xc nh c gi tr c th


i tng m chng ta ang xt (v d s, hm s) nhng vn c th thc hin
cc php ton trn cc i tng . V d ta c th khng bit gi tr cc nghim
ca mt phng trnh, nhng vn bit c tng ca chng:
V d 5.1. Tm tng cc nghim ca phng trnh cos5 x5 cos3 x+3 cos x1 = 0
trn on [0, 2].
i khi ta cn tnh tch phn ca mt hm m ta khng c biu thc tng
minh:
3
V d 5.2. Chng minh rng vi mi t 0, phng
R 7 trnh 2x + tx 8 = 0 lun
c 1 nghim dng duy nht, k hiu l x(t). Tnh 0 [x(t)] dt.

136

5.2. V cc dy s xc nh bi dy cc phng trnh

Trong bi vit nh ny, chng ta s cp n mt tnh hung cn bn khc,


l kho st nhng dy s xc nh bi dy cc phng trnh.
Bi ton 5.1. Cho dy cc hm s fn (x) xc nh bi cng thc tng minh
hoc truy hi tho mn iu kin: cc phng trnh fn (x) = 0 c nghim duy
nht xn D. Cn kho st cc tnh cht ca xn nh kho st s hi t, tm gii
hn...
Chng ta bt u t mt bi ton thi tuyn sinh vo khoa Ton trng i
hc c lp Matxcva nm 2000.
Bi ton 5.2. K hiu xn l nghim ca phng trnh
1
1
1
+
++
=0
x x1
xn
thuc khong (0, 1)
a) Chng minh dy {xn } hi t;
b) Hy tm gii hn .
Bnh lun: Dy xn c xc nh duy nht v hm s
fn (x) =

1
1
1
+
+ +
x x1
xn

lin tc v n iu trn (0, 1). Tuy nhin, ta khng th xc nh c gi tr c


th ca xn . Rt may mn, chng minh tnh hi t ca xn , ta khng cn n
iu . Ch cn chng minh tnh n iu v b chn l . Vi tnh b chn,
mi th u n v 0 < xn < 1. Vi tnh n iu, ta ch mt cht n mi lin
h gia fn (x) v fn+1 (x) trong
fn+1 (x) = fn (x) + fn+1 (x) = fn (x) +

1
.
xn1

y chnh l cha kho chng minh tnh n iu ca xn .


Li gii. R rng xn c xc nh mt cch duy nht, 0 < xn < 1. Ta c
fn+1 (xn ) = fn (xn ) + 1/(xn n 1) = 1/(xn n 1) < 0,
trong khi fn+1 (0+ ) > 0. Theo tnh cht ca hm lin tc, trn khong (0, xn)
c t nht mt nghim ca fn+1 (x). Nghim chnh l xn+1 . Nh th ta
chng minh c xn+1 < xn , tc l dy s {xn } n iu gim. Do dy ny b
chn di bi 0 nn dy s cho c gii hn.

5.2. V cc dy s xc nh bi dy cc phng trnh

137

Ta s chng minh gii hn ni trn bng 0. chng minh iu ny, ta cn


n kt qu quen thuc sau:
1+

1 1
1
+ + + > ln n
2 3
n


1 1
(C th chng minh d dng bng cch s dng nh gi ln 1 +
< ).
n
n
Tht vy, gi s lim xn = a > 0. Khi , do dy s gim nn ta c xn a
vi mi n.
1 1
1
Do 1 + + + + khi n nn tn ti N sao cho vi mi n N
2 3
n
1 1
1 1
ta c 1 + + + + > .
2 3
n a
Khi vi n N ta c
0=

1
1
1
1 1
1
1
1
1
+
+
< + = 0,
++
<
+
+ +
xn xn 1
xn n xn 1 2
xn a a

mu thun. Vy ta phi c lim xn = 0.


Bi ton 5.3. Cho n l mt s nguyn dng (n > 1). Chng minh rng phng
trnh xn = x + 1 c mt nghim dng duy nht, k hiu l xn . Chng minh rng
xn dn v 1 khi n dn n v cng v tm
lim n(xn ) 1).

Li gii. R rng xn > 1. t fn (x) = xn x 1. Khi fn+1 (1) = 1 < 0 v


fn+1 (xn ) = xnn+1 xn 1 > xnn xn 1 = fn (xn ) = 0.
T ta suy ra 1 < xn+1 < xn . Suy ra dy {xn } c gii hn hu hn a. Ta
chng minh a = 1. Tht vy, gi s a > 1. Khi xn a vi mi n v ta tm
c n ln sao cho: xnn an > 3 v xn + 1 < 3, mu thun vi fn (xn ) = 0.
gii phn cui ca bi ton, ta t xn = 1 + yn vi lim yn = 0. Thay vo
phng trnh fn (xn ) = 0, ta c (1 + yn )n = 2 + yn . Ly logarith hai v, ta c
n ln(1 + yn ) = ln(2 + yn )
T suy ra
lim n ln(1 + yn ) = ln 2
Nhng lim ln(1 + yn )/yn = 1 nn t y ta suy ra lim nyn = ln 2, tc l
lim n(xn 1) = ln 2.

5.2. V cc dy s xc nh bi dy cc phng trnh

138

Bi ton 5.4 (VMO 2007). Cho s thc a > 2 v fn (x) = a10xn+10 + xn + +


x + 1.
a) Chng minh rng vi mi s nguyn dng n, phng trnh fn (x) = a lun
c ng mt nghim dng duy nht.
b) Gi nghim l xn , chng minh rng dy {xn } c gii hn hu hn khi
n dn n v cng.
Li gii. Kt qu ca cu a) l hin nhin v hm fn (x) tng trn (0, +). D
dng nhn thy 0 < xn < 1. Ta s chng minh dy xn tng, tc l xn+1 > xn .
Tng t nh nhng li gii trn, ta xt
fn+1 (xn ) = a10xn+11
+ xn+1
+ xnn + + x + 1 = xn fn (xn ) + 1 = axn + 1
n
n
V ta c fn+1 (1) = a10 + n + 1 > a nn ta ch cn chng minh axn + 1 < a l
a1
s suy ra xn < xn+1 < 1. Nh vy, cn chng minh xn <
. Tht vy, nu
a
a1
xn
th
a
 a 1n+1
 a 1n+10 1
a
fn (xn ) a10
+
=
a
a1
1
a

 a 1 n
n
10 a 1
= (a 1)
+a (a 1)
>a
a
a
(do a 1 > 1). Vy dy s tng {xn } tng v b chn bi 1 nn hi t.
Nhn xt 5.1. Mt ln na mi lin h fn+1 (x) = xfn (x) + 1 li gip chng ta
tm c mi quan h gia xn v xn+1 . T li gii trn, ta c th chng minh
a1
a1
c rng lim xn =
. Tht vy, t c =
< 1, theo tnh ton trn th
a
a
fn (c) fn (xn ) = kcn (vi k = (a 1)((a 1)9 1) > 0)
Theo nh l Lagrange th
fn (c) fn (xn ) = f 0 ()(c xn ) vi thuc (xn , c)
Nhng f 0 () = (n + 10)a10 n+9 + n n1 + + 1 > 1 nn t y suy ra
kcn > c xn
T ta c
c kcn < xn < c
V c ngha lm lim xn = c.

5.2. V cc dy s xc nh bi dy cc phng trnh

139

Bi ton 4. (VMO 2002) Cho n l mt s nguyn dng. Chng minh rng


phng trnh
1
1
1
1
+
++ 2
=
x 1 4x 1
n x1 2
c mt nghim duy nht xn > 1. Chng minh rng khi n dn n v cng, xn
dn n 4.
Bnh lun: Vic chng minh phng trnh c nghim duy nht xn > 1 l
1
hin nhin. Mi lin h fn+1 (x) = fn (x) +
cho thy xn l dy
((n + 1)2x 1)
1
1
1
1
s tng ( y fn (x) =
+
+ + 2
). bi cho sn
x 1 4x 1
n x1 2
gii hn ca xn l 4 lm cho bi ton tr nn d hn nhiu. Tng t nh
cch chng minh lim xn = c nhn xt trn, ta s dng nh l Lagrange
nh gi khong cch gia xn v 4. lm iu ny, ta cn tnh fn (4), vi
1
1
1
1
fn (x) =
+
+ + 2
. Rt may mn, bi tnh fn (4) ny
x 1 4x 1
n x1 2
lin quan n 1 dng tng quen thuc.
Li gii: t fn (x) nh trn v gi xn l nghim > 1 duy nht ca phng trnh
fn (x) = 0. Ta c
1
1
1
1
1
1
1
1
+
+ + 2
=
+
++

4 1 16 1
4n 1 2 1.3 3.5
(2n 1)(2n + 1) 2


1 1 1 1 1
1
1
1
1
=
+ + +

=
2 1 3 3 5
2n 1 2n 2
4n

fn (4) =

p dng nh l Lagrange, ta c
1
= |fn (xn ) f (4)| = |f 0(c)||xn 4|
4n
vi c thuc (xn , 4)
Nhng do
|f 0 (c)| =

1
1
1
+
+ >
2
2
(c 1)
(4c 1)
9

9
, suy ra lim xn = 4.
4n
Trong v d trn (v trong phn nhn xt bi ton 3) chng ta s dng
nh l Lagrange nh gi hiu s gia xn v gi tr gii hn. v d cui
cng ca bi vit ny, ta tip tc nu ra ng dng dng nh l ny trong mt
tnh hung phc tp hn.
Nn t y |xn 4| <

140

5.2. V cc dy s xc nh bi dy cc phng trnh

Bi ton 5. Cho n l mt s nguyn dng > 1. Chng minh rng phng


trnh xn = x2 + x + 1 c mt nghim dng duy nht, k hiu l xn . Hy tm s
thc a sao cho gii hn limn na (xn xn+1 ) tn ti, hu hn v khc 0.
Bnh lun. D thy gi tr a, nu tn ti, l duy nht. Tng t nh bi
ln(3)
ton 2, c th chng minh c rng xn 1 +
. T c d on l a = 2.
n
nh l Lagrange s gip chng ta nh gi hiu xn xn+1 v chng minh d
on ny.
Li gii. t
Pn (x) = xn x2 x 1.
Ta c
Pn+1 (x) = xn+1 x2 x 1 = xn+1 xn + Pn (x) = xn (x 1) + Pn (x).
T
Pn+1 (xn ) = xnn (xn 1) + Pn (xn ) = (x2n + xn + 1)(xn 1) = x3n 1.
p dng nh l Lagrange, ta c
0

(x2n + xn + 1)(xn 1) = Pn+1 (xn ) Pn+1 (xn+1 ) = (xn xn+1 )Pn+1 (c)
0

vi c thuc (xn+1 , xn), Pn+1 (x) = (n + 1)xn 2x 1.


T

(n + 1) xn+1 + 1 +

1 

xn+1

0
0
2xn+1 1 = Pn+1
(xn+1 ) < Pn+1
(c) <
0
< Pn+1
(xn ) = (n + 1)(x2n + xn + 1) 2xn 1.

T y, vi lu lim xn = 1, ta suy ra
lim

0
Pn+1
(c)
=3
n

Tip tc s dng lim n(xn 1) = 3, ta suy ra


0
lim nPn+1
(c)(xn xn+1 ) = lim n(x2n + xn + 1)(xn 1) = 3 ln(3)

P 0 (c)
lim n2 (xn xn+1 ) n+1
= 3 ln(3)
n
n
P 0 (c)
lim n2 (xn xn+1 ) lim n+1
= 3 ln(3)
n
n
n
lim n2 (xn xn+1 )3 = 3 ln(3)
n

lim n2 (xn xn+1 ) = ln(3)

141

5.3. nh l v ba mnh tng ng

Vy vi c = 2 th gii hn cho tn ti, hu hn v khc 0. D thy vi


c > 2 th gii hn cho bng v cng v vi c < 2 th gii hn cho bng 0.
Vy c = 2 l p s duy nht ca bi ton.
Qua cc v d trn, chng ta thy cng c c bn kho st cc dy s cho
bi dy cc phng trnh l cc nh l c bn ca gii tch (v hm lin tc, hm
n iu, nh l v s hi t ca dy s n iu v b chn, nh l Lagrange)
v mi lin h mang tnh truy hi gia cc phng trnh. Hy vng rng vic phn
tch cc tnh hung 5 v d trn y s gip chng ta c mt cch nhn tng
qut cho cc bi ton dng ny.

5.3

nh l v ba mnh tng ng

nh l 5.1 (V ba mnh tng ng). . Cho dy s {ck } vi 0 < ck < 1,


k = 1, 2, 3, . . .. Xt cc dy s
Xn =

n
Y

(1 + ci ); Yn =

i=1

n
Y

(1 ci ).

i=1

Khi ba khng nh sau l tng ng


(i) lim Xn = +,
n+

(ii)

lim Yn = 0,
P

n
(iii) lim
ci = +.
n+

n+

i=1

Chng minh.
Xt khng nh (i) (iii).
n
P
Gi s
ci < M, vi 0 < M < +. Khi
i=1
n
Y
i=1

1 X n 
M n
(1 + ci ) < 1 +
ci < 1 +
< eM ,
n
n


i=1

v l v rng lim Xn = +. Do lim


n+

n+

P
n


ci = +.

i=1

Xt khng nh (iii) (i). iu ny l hin nhin ng v rng


n
Y
i=1

(1 + ci ) >

n
X
i=1

ci .

142

5.4. Mt s bi ton v c lng tng v tch

Xt khng nh (ii) (iii). Nhn xt rng, ng vi b n s bt k a1, a2, . . . , an


vi 0 < ai < 1, th
n
n
X
Y
ai > 1
(1 ai ).
i=1

i=1

D dng kim tra tnh ng n ca bt ng thc ny bng qui np.


n
n
Q
Q
Do lim
(1ci ) = 0 nn ng vi mi m lun tn ti n sao cho
(1ci ) <
n+ i=1

i=1

1
. T ta c
2

n
X

ci > 1

i=1

Suy ra

+
P

n
Y

(1 ci ) >

i=1

1
.
2

ci = +.

i=1

Xt khng nh (i) (ii). Ta c


1>

n
Y

(1 c2i ) =

i=1

Nhng v lim

n
Q

n+ i=1

n
Y

(1 + ci)

i=1

(1 + ci) = + nn lim

n
Y

(1 ci).

i=1
n
Q

n+ i=1

(1 ci) = 0 (theo nguyn l kp).

Do lim Yn = 0.
n+


By gi ta chuyn sang phn p dng nh l trn gii quyt mt s bi
ton.

5.4

Mt s bi ton v c lng tng v tch

Bi ton 5.5. Cho dy s thc tng {un } c tnh cht


lim un = +.

n+

Chng minh rng lun tn ti k N sao cho


u 1 u2
uk
+
++
< k 2007
u2 u3
uk+1
(ta gi s u1 > 0).

143

5.4. Mt s bi ton v c lng tng v tch

Gii.
Ta s dng bin i tng ng sau
u

X
ui 
u2
uk 
k
> 2007
1
> 2007.
+
++
u2 u3
uk+1
ui+1
k

i=1

ui
Do {un } l dy tng nn 0 < 1
< 1.
ui+1
ui
t ci = 1
. Suy ra 0 < ci < 1. Mt khc, ta c
ui+1
n
Y

(1 ci) =

i=1

n
Y
ui
u1
=
ui+1
un+1
i=1

tin dn ti 0 khi n +. Vy nn

n
P

ci = + (T 2) 3). Do k N

i=1
k 
X

ui  X
=
ci > 2007.
ui+1
k

i=1

i=1

Bi ton 5.6. Cho dy s {an } dng c tnh cht lim an = +. Chng minh
n+

rng lun tn ti k N sao cho


k
X
i=1

Gii.
t ci =

ai
2007
> 263
.
a1 + a 2 + + a i

ai
V ai > 0 nn 0 < ci < 1 v
a1 + a 2 + + a i
1 ci =

a1 + a2 + + ai1
, vi i 2.
a1 + a 2 + + a i

Suy ra
n
Y

(1 ci ) =

i=2

tin dn ti 0 khi n +.
V ai > 0 v lim an = +, nn
n+

iu phi chng minh.

a1
a1 + a 2 + + a n
n
P
i=1

ci = + hay k N

n
P
i=1

ci > 263

2007

144

5.5. Bi tp

Bi ton 5.7. Xt dy tt c cc s nguyn t {pn }, 2 = p1 < p2 < p3 < .


Chng minh rng
+
X
1
= +.
p
n=1 n
Gii.
Bi ton 5.8. Xt dy s {an } xc nh bi cng thc
1
2n 1
a1 = , an+1 =
an , n = 1, 2, . . ..
2
2n + 2
Chng minh rng
+
X

an = 1.

n=1

Gii.
Bi ton 5.9. Cho dy s {an } dng, tng v khng b chn. Chng minh rng
+ h
X

arccos

n=1

 a i2
n
= +.
an+1

Gii.

5.5

Bi tp

Bi 5.1. Cho dy s {an } dng, tng v khng b chn. Chng minh rng
+
X
n=1

arccos

 a 
n
= +.
an+1

Chng 6

Mt s lp hm chuyn i cc
cp s
Trong chng ny s m t mt s lp hm s chuyn i cc cp s.

6.1

Cp s cng, cp s nhn v cp s iu ho

Trong chng trnh ton bc trung hc, cc bi ton v cp s cng v


cp s nhn c cp kh y . c bit, trong cc sch gio khoa v
sch bi dng, nng cao c mt s lng rt ln cc bi ton v tnh tng, xc
nh s hng tng qut, iu kin mt dy lp thnh mt cp s,... . V vy,
trong mc ny chng ta ch yu tp trung kho st mt s c trng c lin quan
trc tip n dy s l cc cp s cng, cp s nhn v mt vi dng cp s m
rng.
nh ngha 6.1. (i) Dy s {un } (hoc (un )) tho mn iu kin
u1 u0 = u2 u1 = = un+1 un =
c gi l mt cp s cng.
(ii) Khi dy s {un } lp thnh mt cp s cng th hiu d = u1 u0 c gi
l cng sai ca cp s cho.
Nhn xt rng khi cho mt dy hu hn s {u0 , u1, . . ., us } tho mn iu
kin
u1 u0 = u2 u1 = = us us1
th ta cng ni rng dy hu hn cho lp thnh mt cp s cng vi cng sai
d = u 1 u0 .
145

146

6.2. Dy s tun hon

nh ngha 6.2. (i) Dy s {un } (un 6= 0 vi mi n N) tho mn iu kin


u1
u2
un+1
=
= =
=
u0
u1
un
c gi l mt cp s nhn.
u1
(ii) Khi dy s {un } lp thnh mt cp s nhn th thng q =
c gi
u0
l cng bi ca cp s cho.
Nhn xt rng khi cho mt dy hu hn s khc 0 : {u0 , u1, . . . , us} tho mn
iu kin
u1
u2
us
=
= =
u0
u1
us1
th ta cng ni rng dy hu hn cho lp thnh mt cp s nhn vi cng bi
u1
p=
.
u0
Ta lun c mi lin h gia cp s cng v cp s nhn sau y.
Bi ton 6.1. (i) Nu dy s {un } l mt cp s cng th dy s {vn } vi
vn = aun , n N, a > 0
s lp thnh mt cp s nhn.
(ii) Ngc li, nu dy s {un } l mt cp s nhn vi cc s hng dng th
dy s {vn } vi
vn = loga un , n, N, 0 < a 6= 1
s lp thnh mt cp s cng.
Gii Chng minh c suy ra trc tip t nh ngha 6.1 v 6.2

nh ngha 6.3. Dy s {un } (un 6= 0 vi mi n N) tho mn iu kin


un =

2un1 un+1
, n N
un1 + un+1

c gi l cp s iu ho.

6.2

Dy s tun hon

Tng t nh i vi hm s thng thng, ta c th coi dy s {xn } nh


mt hm f (n) = xn xc nh trn tp N v nhn gi tr trong R. Ta ch quan
tm n hai loi dy tun hon c bn l tun hon cng tnh v tun hon nhn
tnh.

147

6.2. Dy s tun hon

nh ngha 6.4. Dy s {un } c gi l mt dy tun hon (cng tnh) nu


tn ti s nguyn dng l sao cho
un+l = un , n N.

(1)

S nguyn dng l nh nht dy {un } tho mn (1) c gi l chu k c s


ca dy.
Trong thc hnh, chng minh mt dy cho l tun hon, khng nht
thit phi xc nh chu k c s ca n.
Nhn xt 6.1. Dy tun hon chu k 1 khi v ch khi dy l mt dy hng.
Tng t, ta cng c nh ngha v dy tun hon nhn tnh.
nh ngha 6.5. Dy s {un } c gi l mt dy tun hon nhn tnh nu tn
ti s nguyn dng s (s > 1) sao cho
usn = un , n N.

(2)

S nguyn dng s nh nht dy {un } tho mn (2) c gi l chu k c s


ca dy.
Bi ton 6.2. Chng minh rng dy {un } tun hon (cng tnh) chu k 2 khi
v ch khi dy c dng
un =

1
[ + + ( )(1)n+1], , R.
2

(3)

Gii Gi s u0 = , u1 = v un+2 = un , n N. Khi ta thy ngay (bng


quy np ton hc) dy {un } c dng (3). Ngc li, mi dy xc nh theo (3) l
mt dy tun hon chu k 2.

Bi ton 6.3. Chng minh rng dy {un } tun hon nhn tnh chu k 2 khi v
ch khi dy c dng
(
tu vi nl,
un =
u2k+1 vi n = 2m (2k + 1), m N, k N.
Gii Chng minh c suy trc tip t h thc truy hi.
Bi ton 6.4. Chng minh rng dy {un } tun hon chu k 3 khi v ch khi dy
c dng

3
1
2n
2n
un = [ + + +( +2)] cos
+
( ) sin
, , , R. (4)
3
3
2
3

148

6.2. Dy s tun hon

Gii. Gi s u0 = , u1 = , u2 = v un+3 = un , n N. Khi , ta thy


ngay (bng quy np ton hc) dy {un } c dng (4).
Ngc li, mi dy xc nh theo (4) l mt dy tun hon chu k 3 :
, , , , , , . . . .
Bi ton 6.5. Cho k Q \ Z. Chng minh rng dy s {un } xc nh theo cng
thc
u0 = 1, u1 = 1, un+1 = kun un1 , n N
khng l mt dy tun hon.
Gii. Khi |k| > 2 th
|un+1 | > |k||un| |un1 | > 2|un | |un1 |.
Nu lun lun xy ra |un | < |un1 | vi mi n N th ta c ngay iu phi chng
minh. Nu xy ra |um | > |um1 | > 0 th suy ra
|um | < |um+1 | <
v do dy {un } khng l mt dy s tun hon.
p
Xt |k| 6 2 vi k = , (p, q) = 1, 2 6 q Z , p Z. Bng quy np theo n ta
q
thu c
pj
uj = j1 , pj Z, (pj , q) = 1, j {1, . . ., n}.
q
T suy ra

p
pn+1
un+1 = un un1 = n ,
q
q

trong
pn+1 = ppn q 2pn1 Z
v (pn+1 , q) = 1. Do q > 2 nn un 6= um khi n 6= m v dy {un } khng l dy s
tun hon.

Bi ton 6.6. Xc nh cc gi tr ca k Q dy s {un } xc nh theo cng
thc
u0 = 1, u1 = 1, un+1 = kun un1 , n N
l mt dy s tun hon.

149

6.2. Dy s tun hon

Gii.
p
Theo kt qu ca Bi ton 4, khi |k| > 2 v |k| 6 2, k =
vi (p, q) = 1,
q
2 6 q Z th dy {un } khng l dy s tun hon.
Xt |k| 6 2 v k Z.
Vi k = 2 th {un } l mt cp s cng vi cng sai bng 2 nn hin nhin
dy {un } khng l dy tun hon.
Vi k = 1 th {un } l dy tun hon chu k 6 :
u2 = 2, u3 = 1, u4 = 1, u5 = 2, u6 = 1, u7 = 1, . . . .
Vi k = 0 th {un } l dy tun hon chu k 4 :
u0 = 1, u1 = 1, u2 = 1, u3 = 1, u4 = 1, u5 = 1, . . . .
Vi k = 1 th {un } l dy tun hon chu k 3 :
u0 = 1, u1 = 1, u2 = 0, u3 = 1, u4 = 1, . . . .
Vi k = 2 th {un } l dy tun hon chu k 2 :
u0 = 1, u1 = 1, u2 = 1, u3 = 1, u4 = 1, . . . .

nh ngha 6.6. a) Dy s {un } c gi l mt dy phn tun hon (cng
tnh) nu tn ti s nguyn dng l sao cho
un+l = un , n N.

(5)

S nguyn dng l nh nht dy {un } tho mn (5) c gi l chu k c s


ca dy.
b) Dy s {vn } c gi l mt dy phn tun hon nhn tnh nu tn ti s
nguyn dng s (s > 1) sao cho
vsn = vn , n N.

(6)

S nguyn dng s (s > 1) nh nht dy {vn } tho mn (6) c gi l chu


k c s ca dy.
Nhn xt 6.2. a) Dy phn tun hon vi chu k l l mt dy tun hon chu
k 2l.
b) Dy phn tun hon nhn tnh chu k s l mt dy tun hon nhn tnh chu
k 2s.

150

6.2. Dy s tun hon

Bi ton 6.7. Chng minh rng mi dy {un } phn tun hon chu k r u c
dng
1
(7)
un = (vn vn+r ) vi vn+2r = vn .
2
Gii. Gi s un+r = un , n N. Khi , ta thy ngay rng dy {un } tun
hon chu k 2r v
1
un = (un un+r ),
2
tc l c dng (7).
Ngc li, kim tra trc tip, ta thy mi dy xc nh theo (7) u l dy
phn tun hon chu k r.

Bi ton 6.8. Cho f (x) l mt a thc vi deg f = k 1, f (x) Z ng vi
mi x Z. K hiu r(k) = min{2s | s N, 2s > k}. Chng minh rng dy s
{(1)f (k)} (k = 1, 2, . . .) l dy tun hon vi chu k r(k).
Gii. Ta c k!f (x) Z[x]. Biu din f (x) di dng
 
 
x
x
f (x) = a0 + a1
+ + ak
,
1
k
trong

 
x
x(x 1) (x k + 1)
=
.
k
k!

Ta cn chng minh f (x + r(k)) f (x) chia ht cho 2 vi mi x Z.


Nhn xt rng

  
x + 2s
x
Mi =

i
i
chia ht cho 2 vi mi i N , 2s i, x Z. Tht vy, ta c
Mi =


1 s
(2 + x)(2s + x 1) . . .(2s + x i + 1) x(x 1) . . . (x i + 1) .
i!

T s hin nhin chia ht cho 2s . Mt khc, s m ca 2 trong khai trin ca i!


l

X
i X
i
<
= i 2s ,
j
2
2j
j=1

j=1

nn Mi chia ht cho 2 vi mi i N, i 2s , x Z. T suy ra



  
x + r(k)
x
Ti =

i
i

151

6.2. Dy s tun hon

chia ht cho 2 vi mi i Z, i k, x Z. Do aj Z nn
f (x + r(k)) f (x) =

k
X

aj Tj

j=0

chia ht cho 2, iu phi chng minh.

Bi ton 6.9. Xc nh dy s {un } tho mn iu kin


u2n+1 = 3un , n N.

(8)

Gii. t n + 1 = m, m = 1, 2, . . .. Khi c th vit (8) di dng


u2m1 = 3um1 , m N
hay
v2m = 3vm , m N

(9)

vm = um1 , m N .

(10)

vi
T (9) ta c v0 = 0. t vm = mlog2 3ym , m N . Khi (9) c dng
y2m = ym , m N .
Vy {ym } l mt dy tun hon nhn tnh chu k 2. Khi theo Bi ton 2 ta c
(
tu vi nl,
yn =
y2k+1 vi nc dng 2m (2k + 1), m N , k N.
T suy ra
um = vm+1 = mlog2 3 ym+1 ,
vi

(
tu
yn =
y2k+1

vi nl,
vi nc dng 2m (2k + 1), m N , k N.


Bi ton 6.10. Xc nh dy {un } tho mn iu kin


u2n+1 = 3un + 4, n N.

(11)

6.3. Hm s chuyn i cp s cng

152

Gii. t n + 1 = m, m = 1, 2, . . .. Khi c th vit (11) di dng


u2m1 = 3um1 + 4, m N
hay
v2m = 3vm + 4, m N

(12)

vi vm = um1 .
t vm = 1 + xm . Khi (12) c dng
x2m = 3xm , m N.

(13)

t xm = mlog2 3 ym , m N . Khi (13) c dng


y2m = ym , m N .
Vy {ym } l mt dy phn tun hon nhn tnh chu k 2.
Khi , theo Bi ton 2, ta c

vi nl,
tu
yn = y2k+1 vi nc dng 22m+1 (2k + 1), m, k N,

y2k+1
vi nc dng 22m (2k + 1), m N, k N.
T suy ra
um = vm+1 = 1 + (m + 1)log2 3 ym+1 ,
vi

6.3

tu
yn = y2k+1

y2k+1

vi nl,
vi nc dng 22m+1 (2k + 1), m, k N,
vi nc dng 22m (2k + 1), m N, k N.

Hm s chuyn i cp s cng

Bi ton 6.11. Nu dy s {un } l mt cp s cng th dy s {vn }


vi vn = aun + b, n N s lp thnh mt cp s cng.
Gii. Gi s {un } l cp s cng vi cng sai bng d.
Xt dy s {vn } vi vn = aun + b, n N.
Ta c v0 = au0 + b, v1 = au1 + b . . .vn = aun + b, vn+1 = a(n + 1) + b.
Khi
v1 v0 = v2 v1 = v3 v2 = vn+1 vn = ad
Vy dy {vn } l cp s nhn vi cng sai bng ad

Vn t ra l ta i tm tt c cc hm s c tnh cht chuyn cp s cng
thnh cp s cng. Xt b sau.

153

6.3. Hm s chuyn i cp s cng

B 6.1. Cho cp s cng {an } v hm s f : R R+ tho mn iu kin


f

x + y
2

f (x) + f (y)
, x, y > 0.
2

Khi dy {f (an )} l mt cp s cng.


Chng minh. T gi thit, ta c cc h thc
a1 a0 = = an an1 = an+1 an = . . .
Suy ra
2an = an1 + an+1 , n N
Khi
f (an ) = f (

an1 + an+1
f (an1 ) + f (an+1 )
)=
.
2
2

T ta c {f (an )} l mt cp s cng.

Bi ton 6.12. Tm hm s f (x) xc nh v lin tc trn R tho mn iu


kin:
 x + y  f (x) + f (y)
f
=
, x, y R
2
2
Gii. t f (x) f (0) = g(x), ta c g(x) lin tc trn R, vi g(0) = 0 v
g

x + y 
2

g(x) + g(y)
, x, y R
2

Ln lt cho y = 0 v x = 0, th
x
g(x)
g( ) =
2
2
v

y
g(y)
g( ) =
, x, y R.
2
2

Do
x + y 
x
y 
g
=g
+g
, x, y R
2
2
2
Vy
g(x + y) = g(x) + g(y), x, y R
Vg(x) lin tc trnR,nn phng trnh trn l phng trnh Cauchy v do
g(x) = ax.Suy ra f (x) = ax + b, (a, b R).

6.4. Hm s chuyn i cp s cng vo cp s nhn

6.4

154

Hm s chuyn i cp s cng vo cp s nhn

Bi ton 6.13. Nu dy s {un } l mt cp s cng th dy s {vn }


vi vn = aun , n N, a > 0 s lp thnh mt cp s nhn.
Gii. Gi s {un } l cp s cng vi cng sai bng d.
Xt dy s {vn } vi vn = aun , n N, a > 0.
Ta c v0 = au0 , v1 = au1 . . . vn = an , vn+1 = an+1 .
Khi
vn+1
v1
v2
v3
=
=
=
= ad
v0
v1
v2
vn
Vy dy{vn } l cp s nhn vi cng bi bng ad
B 6.2. Cho cp s cng {an } v hm s f : R R+ tho mn iu kin
x + y p
f
= f (x)f (y), x, y > 0.
2
Khi dy {f (an )} l mt cp s nhn.
Chng minh. T gi thit, ta c cc h thc
a1 a0 = = an an1 = an+1 an = . . .
Suy ra
2an = an1 + an+1 , n N
Khi
f (an ) = f (

p
an1 + an+1
) = f (an1 )f (an+1 ).
2

T ta c {f (an )} l mt cp s nhn.

Nh vy ta c hai hm s trn chuyn cp s cng thnh cp s nhn, vn


t ra l ta i tm tt c cc hm s c tnh cht chuyn mt cp s cng bt k
thnh mt cp s nhn.Trc ht ta xt bi ton sau.
Bi ton 6.14. Tm hm f (x) xc nh v lin tc trn R tho mn iu kin
p
x+y
f(
) = f (x)f (y), x, y R
2
Gii. Theo iu kin bi ton ta suy raf (x) > 0, x R.
Nu tn ti x0 f (x0 ) = 0 th
p
x0 + y
f(
) = f (x0)f (y) = 0, y R,
2
tc l f (x) 0
Xt trng hp f (x) > 0, x R. Khi ta c

6.5. Hm s chuyn i cp s nhn vo cp s cng

ln f (
hay

155

x+y
ln f (x) + ln f (y)
)=
, x, y R
2
2
x + y 

g(x) + g(y)
=
, x, y R
2
2
trong g(x) = ln f (x). Theo kt qu Bi ton 4 th g(x) = ax + b.
Vy f (x) = eax+b , a, b R tu .
g

6.5

Hm s chuyn i cp s nhn vo cp s cng

Bi ton 6.15. Nu dy s {un } l mt cp s nhn vi cc s hng dng th


dy s {vn }
vi vn = loguan , n N, 0 < a 6= 1 s lp thnh mt cp s cng.
Gii. Gi s {un } l cp s nhn vi cng bi bng q. Xt dy s {vn } vivn =
loguan , n N, 0 < a 6= 1.
Ta c
v0 = logua0 , v1 = logua1 , v2 = logua2 . . . , vn = loguan
Khi
v1 v0 = v2 v1 = v3 v2 = vn vn1 = logda
Vy{vn } l cp s cng vi cng sai bng logda

B 6.3. Cho cp s nhn {an } vi an > 0n N v hm s f (x) tho mn


iu kin

f (x) + f (y
f ( xy) =
, x, y > 0.
2
Khi dy {f (an )} l mt cp s cng.
Chng minh. T gi thit, ta c cc h thc
a0
an1
an
= =
=
= ...
a1
an
an+1
Suy ra
a2n = an1 an+1 , n N
Khi

an1 + an+1

f (an ) = f ( an1 an+1 ) = f (


).
2
T ta c {f (an )} l mt cp s cng.

Vn t ra ta i tm tt c cc hm s chuyn i mt cp s nhn bt k
thnh mt cp s cng. Trc ht ta xt bi ton sau.

6.6. Hm s chuyn i cp s nhn vo cp s iu ho

156

Bi ton 6.16. Tm hm s f (x) xc nh v lin tc trn R tho mn iu kin


f (x) + f (y)

f ( xy) =
, x, y R+
2
Gii V x > 0, y > o nn c th t x = eu , y = ev v f (eu ) = g(u).
Khi g(u) lin tc tn R v c dng
g

u + v 
2

g(u) + g(v)
, u, v R.
2

Theo kt qu ca bi ton 8.52 thg(u) = au + b.


Vy ta c kt qu f (x) = a ln x + b, a, b R tu .
Theo b 6.3 trn ta c hm s f (x) = a ln x + b, a, b R chuyn i mi
cp s nhn thnh cp s cng.

6.6

Hm s chuyn i cp s nhn vo cp s iu
ho

Ta xt bi ton sau.
Bi ton 6.17. Cho cp s nhn {an } vi an > 0, n N v cho hm s
f : R+ R+ tho mn iu kin

2f (x)f (y)
f ( xy) =
, x, y R+
f (x) + f (y)
Chng minh rng dy {f (an )} l mt cp s iu ho.
Gii. T gi thit, ta c cc h thc
a1
an
an+1
= =
=
= ...
a0
an1
an
Suy ra
a2n = an1 an+1 , n N
Khi

2f (an1 )f (an+1 )

f (an ) = f ( an1 an+1 ) =


).
f (an1 ) + f (an+1 )

T ta c {f (an )} l mt cp s iu ho.

By gi ta i tm tt c cc hm s c tnh cht chuyn mt cp s cng bt


k thnh cp s iu ho thng qua vic tm tt c cc hm s c tnh cht sau.

6.6. Hm s chuyn i cp s nhn vo cp s iu ho

157

Bi ton 6.18. Tm hm f (x) xc nh v lin tc trn R+ tho mn iu kin


2f (x)f (y)

f ( xy) =
, x, y R+
f (x) + f (y)
Gii. Ta c

2f (x)f (y)
f ( xy) =
, x, y R+
f (x) + f (y)

Suy ra

f ( xy) =

2
, x, y R+
1
1
+
f (x) f (y)

Hay
1
1
+
1
f (x) f (y)
=
, x, y R+

f ( xy)
2
t g(x) =

1
. Khi ta c
f (x)

g(x) + g(y)
g( xy) =
, x, y R+
2

Theo kt qu ca Bi ton 6.16 th g(x) = a ln x + b. f (x) lin tc trong R+


th g(x) 6= 0 vi mi x R+ . iu tng ng vi a = 0, b 6= 0
Vy
f (x) b R{0} tu .


Chng 7

Mt s lp hm chuyn i cc
cp s trong tp ri rc
Trong chng ny s m t mt s lp hm s chuyn i cc cp s
trong tp hp Z, N.

7.1

Hm s chuyn i cp s cng thnh cp s cng

Trc ht ta xt bi ton sau.


Bi ton 7.1. Tm cc hm s f (x) xc nh trn Z tho mn tnh cht
f (x + y) = f (x) + f (y), x, y Z
Gii. Trc ht ta kho st hm s f (x) trong tp hp N
Ti x = 0, y = 0, ta c f (0) = 0
Ti x = 1, y = 1, ta c f (2) = 2f (1) t f (1) = a ta c f (2) = 2a
Ti x = 2, y = 1, ta c f (3) = f (2) + f (1) f (3) = 3f (1) hay f (3) = 3a
Bng php qui np ta chng minh c f (n) = nf (1) hay f (n) = na, N
Vix, y Z
Thay x = y ta c f (0) = f (x) + f (x) f (x) = f (x). Khi ta c
hm f (x) l hm l.
Xt n Z, n < 0 n > 0, khi theo chng minh phn trn ta c
f (n) = na
m f (n) = f (n) f (n) = na
Vy hm s cn tm l f (x) = axx Z.
Bi ton 7.2. Tm hm s f (x) xc nh trn Z tho mn iu kin:
 x + y  f (x) + f (y)
f
=
, x, y Z, x + y = 2k, k Z
2
2
158

7.1. Hm s chuyn i cp s cng thnh cp s cng

159

Gii. t f (0) = b, f (x) = b + g(x) th g(0) = 0, thay vo cng thc trn ta c


b + g(x) + b + g(y)
x+y
)=
2
2
g(x) + g(y)
x+y
g(
)=
2
2

b + g(

Ln lt chn x = 2k, y = 0 , hoc x = 0, y = 2k ta c


g(x + y)
g(x) + g(y)
=
2
2
g(x + y) = g(x) + g(y)x, y Z
Theo kt qu Bi ton 7.1 ta c g(x) = ax, x Z
Vy f (x) = ax + b
Bi ton 7.3. Chng minh rng iu kin cn v dy s {an } lp thnh
mt cp s cng l dy cho phi tho mn h thc
2am+n = a2m + a2n , m, n N.

(7.1)

Gii.
iu kin cn.
Gi s dy {an } l mt cp s cng vi cng sai bng d.
Khi
an = ao + (n 1)d, n N .
Vy nn
a2n + a2m = 2ao + (2m + 2n 2)d
V
2am+n = 2 [ao + (m + n 1)d]
T ta c ngay cng thc 7.1
iu kin .
Gi s dy {an } tho mn iu kin 7.1. Ta chng minh dy {an } l mt cp
s cng vi cng sai bng d = a1 ao
Thay m = 0 vo cng thc 7.1 ta c
2an = ao + a2n
Thay n = 0 vo cng thc 7.1 ta c
2am = ao + a2m

7.1. Hm s chuyn i cp s cng thnh cp s cng

160

Thay kt qu trn vo cng thc 7.1 ta thu c


2am+n = 2am + 2an 2ao
am+n = am + an ao
Thay m = 1 vo cng thc 7.2 ,ta c
an+1 = an + d, d = a1 ao
Vy dy {an } l mt cp s cng.

(7.2)

B 7.1. iu kin cn v mt hm s chuyn mi cp s cng nguyn


dng thnh cp s cng l hm chuyn tp cc s t nhin thnh cp s cng.
Chng minh iu kin cn.
Nu hm f chuyn mi cp s cng thnh cp s cng th hin nhin hm f
chuyn tp cc s t nhin thnh mt cp s cng v tp cc s t nhin l cp
s cng vi cng sai nh nht l 1.
iu kin .
Hm f chuyn tp cc s t nhin thnh cp s cng,tc l dy {f (n)} l cp
s cng n N.Dy {an } l cp s cng nguyn dng, vi cng bi l d N ta
phi chng minh dy {f (an )} l cp s cng.
V dy {f (n)} l cp s cng nn theo cng thc 7.2 ta c
f (m + n) = f (m) + f (n) f (o), m, n N
Dy {an } l cp s cng nguyn dng, vi cng bi l d N suy ra an+1 =
an + d
Khi
f (an+1 ) = f (an + d) = f (an ) + f (d) f (0)
hay f (an+1 ) f (an + d) = f (d) f (0) khng i.
Vy dy {f (an )} l cp s cng vi cng sai l f (d) f (0)

Bi ton 7.4. Xc nh cc hm s f : Z R+ chuyn mi cp s cng
{an }, an Z thnh cp s cng.
Gii. gii bi ton ny theo B 7.1 ta ch cn xc nh cc hm s chuyn
dy s t nhin thnh cp s cng.Hm f chuyn dy s t nhin thnh cp s
cng th ta c:
f (m + n) = f (m) + f (n) f (o), m, n N

7.2. Hm s chuyn i cp s nhn thnh cp s nhn

161

f (m + n) f (0) = f (m) f (0) + f (n) f (0), m, n N.


t g(n) = f (n) f (0) ta c
g(m + n) = g(m) + g(n)
Khi theo bi ton 7.1 ta c g(x) = ax, x N trong a = g(1)
Do f (x) = g(x) + f (0) .t f (0) = b th f (x) = ax + bx N
Kt hp Bi ton 7.2 ta c:
Hm s chuyn i mi cp s cng thnh cp s cng trong tp cc s nguyn
l f (x) = ax + b, x Z.

Bi ton 7.5. Xc nh hm s f chuyn cp s cng nguyn dng {an } cho
trc thnh cp s cng {bn} cho trc.
Gii. Ta xt hai trng hp sau:
(i) Nu {an } N, theo kt qu Bi ton 7.4 ta c f (x) = ax + b, x
N, a, b R.
(ii) Nu {an } N, ta c hm s f : N R c xc nh nh sau
(
bn nu n {an }
f (n) =
(7.3)
cn nu n 6 {an }
trong cn tu trong R. chuyn cp s cng nguyn dng {an } cho trc thnh
cp s cng {bn } cho trc.

7.2

Hm s chuyn i cp s nhn thnh cp s nhn

Trn c s cc bi ton trn ta tm cc hm s chuyn cc cp s khc


trn tp hp s nguyn.Trc ht ta i tm nhng dy s thc hin php chuyn
tip mt i lng trung bnh ca cp phn t tng ng ca dy s.Cc bi ton
ny lin quan cht ch n vic chuyn tip cc cp s, n s phng on cc
cp s tng qut.
Bi ton 7.6. Xc nh dy s {un }, sao cho
u(

p
m+n
m+n
) = u(m)u(n), m, n,
N .
2
2

Gii. Ta c
u(n) = u(

p
n+n
) = u(n)u(n) =
2

[u(n)]2 =| u(n) |

(7.4)

7.2. Hm s chuyn i cp s nhn thnh cp s nhn

162

t u(1) = , u(2) = ( 0, o).


a) Nu = 0 th
u(n) = u(

p
1 + 2n 1
) = u(1)u(2n 1) = 0, n N
2

Vy u(n) 0 l nghim duy nht ca phng trnh (7.4)


b) Nu > 0 v = 0 th
u(n) = u(

p
2 + 2n 2
) = u(2)u(2n 2) = 0, n 2
2

Suy ra
(

u(n) =
0

nu n = 1
nu n 2

l nghim ca phng trnh (7.4)


c)Xt trng hp > 0 v > 0. Gi s tn ti no 3 sao cho u(no ) = 0
Th th
u(no 1) = u(

p
n o + no 2
) = u(no )u(no 2) = 0.
2

Chn no = 3 th u(no 1) = u(2) = 0, hay = 0, mu thun .


Do , ta c th gi thit rng u(n) > 0, vi mi n N. Khi
u(2) = u(

p
3+1
) = u(3)u(1) = 0.
2

Suy ra
u(3) =
Mt khc
u(3) = u(

u2(2)
2
=
.
u(1)

p
4+2
) = u(4)u(2).
2

Suy ra
2 2
)
3
u(4) =
=
= 2
u(2)

u2(3)

Bng phng php quy np ton hc, ta chng minh c rng


u(n) =

n1
, n 3.
n2

7.2. Hm s chuyn i cp s nhn thnh cp s nhn

Ta c

163

n1
2 n
).( )
=
(
n2

= ab,
= ab2(a > 0, b > 0)

Suy ra
2

= a, = b.

Vy nghim ca phng trnh (7.4) l


(
nu n = 1
u(n) =
0 nu n 2 ( 0)
hoc u(n) = abn (a > 0, b > 0).
Bi ton 7.7. Xc nh dy s {un }, sao cho
u(

m+n
2u(m)u(n)
m+n
)=
, m, n,
N
2
u(m) + u(n)
2

Gii.
u(
u(

(7.5)

m+n
2u(m)u(n)
)=
2
u(m) + u(n)

m+n
)=
2

2
1
1
+
u(m) u(n)

1
, th phng trnh cho tng ng vi
u(n)
v(

m+n
v(m) + v(n)
)=
.
2
2

Theo Bi ton 7.2, ta c v(n) = an + b vi a, b 0, a + b > 0. Vy nghim ca


phng trnh (7.5) l
u(n) =

1
, a, b 0, a + b > 0.
an + b

7.2. Hm s chuyn i cp s nhn thnh cp s nhn

164

B 7.2. Chng minh rng iu kin cn v dy cc s dng {an } lp


thnh mt cp s nhn l dy cho phi tho mn h thc
a2m+n = a2m a2n , x, y N

(7.6)

Chng minh. t ln an = bn vi mi n N. Khi an = ebn v (7.6) c dng


e2 bm + n = eb2m + b2n, m, n N
Hay
2bm+n = b2m + b2n

(7.7)

Theo bi ton 7.3 th (7.7) l iu kin cn v dy s {bn } lp thnh


mt cp s cng vi cng sai d = b1 b0.
T theo bi ton 6.13 suy ra iu phi chng minh.
Nhn xt:T cng thc (7.6) ta c
Xt m = 0 ta c a2n = a0 a2n Xt n = 0 ta c a2m = a0 a2m
a 2 a2
Suy ra a2ma2n = n 2m
a0
2 2
a
a
Do a2m+n = n 2m
a0
a n am
Nn am+n =

a0
Bi ton 7.8. Xc nh dy cc s dng {xn } tho mn iu kin
xmn = xm xn , m, n N
Gii. Ta c x1.n = x1xn . Suy ra x1 = 1. Gi s n = p l s nguyn t. Khi
s
1 2
bng qui np ta chng minh c xpk = (xp)k v nu n = p
1 p2 . . . ps th:
xn = (xp1 )1 (xp2 )2 . . . (xps )s
xp c th nhn gi tr tu khi p l mt s nguyn t.
T ta c kt lun:
xp c th nhn gi tr tu khi p l mt s nguyn t v
xn = (xp1 )1 (xp2 )2 . . . (xps )s
s
1 2
khi n = p
1 p2 . . . p s
By gi ta xt tip bi ton sau:

Bi ton 7.9. Xc nh hm s f tho mn tnh cht f (mn) = f (m)f (n) trong


m, n N.

7.2. Hm s chuyn i cp s nhn thnh cp s nhn

165

Gii. Ta c f (1.n) = f (1)f (n). Suy ra f (1) = 1. Gi s n = p l s nguyn t.


s
1 2
Khi bng qui np ta chng minh c f (pk ) = f (p)k v nu n = p
1 p2 . . . ps th:
f (n) = f (p1)1 f (p2 )2 . . . f (ps )s
f (p) c th nhn gi tr tu khi p l mt s nguyn t.
T ta c kt lun:
f (p) c th nhn gi tr tu khi p l mt s nguyn t v
f (n) = f (p1)1 f (p2 )2 . . . f (ps )s
s
1 2
khi n = p
1 p2 . . . p s

Bi ton 7.10. Chng minh hm s f : N R xc nh nh sau vi n =


s
1
2
s
1 2
p
trong
1 p2 . . . ps , pi l cc s nguyn t th f (n) = f (p1 ) f (p2 ) . . . f (ps )
f (pi ) tu , i = 1, 2, 3, . . . chuyn cp s nhn c cng bi nguyn t thnh
cp s nhn.
Gii. Gi s c cp s nhn {n0q k }, k = 0; 1; 2 . . ., n0; q N ta phi chng minh
{f (n0 q k )} cng l cp s nhn. Tht vy theo kt qa bi ton 7.9 trn ta c ngay
iu phi chng minh.
Sau y ta xt bi ton
Bi ton 7.11. Chng minh rng hm s f chuyn mi cp s nhn thnh cp
s nhn khi v ch khi hm s chuyn cp s nhn c cng bi nguyn t thnh
cp s nhn.
Gii. iu kin cn.
Nu hm s f chuyn mi cp s nhn thnh cp s nhn th hin nhin n
chuyn cp s nhn c cng bi nguyn t thnh cp s nhn.
iu kin .
Nu hm s f : N R chuyn cp s nhn c cng bi nguyn t thnh cp
s nhn. Gi s {un } l cp s nhn ta phi chng minh {f (un )} cng l cp s
nhn. Vi un = u0 q n ta xt hai trng hp sau:
Nu q l s nguyn t th bi ton c chng minh.
s

1 2
Nu q khng l s nguyn t th q = p
1 p2 . . . ps trong pi , i N l cc
s nguyn t. Khi ta c
s n
1 2
f (un ) = f (u0 q n ) = f (u0 (p
1 p2 . . . p s ) )

= f (u0(p1)1 .n (p2 )2.n . . . (ps )s.n )


1 .n 2 .n
s .n
= f (u0 )f(p
f(p2 ) . . . f(p
s)
1)

166

7.2. Hm s chuyn i cp s nhn thnh cp s nhn

Theo B 7.6 ta chng minh f 2 (um+n ) = f (u2m )f (u2n )


.(m+n) 2 .(m+n)
.(m+n) 2
Ta c f 2(um+n ) = (f (u0)f(p11)
f(p2 )
. . . f(pss)
)
2 .(m+n) 22 .(m+n)
2 .(m+n)
f(p2 )
. . . f(pss)
1 .2m 2 .2m
1 .2n 2 .2n
s .2m
f (u2m )f (u2n) = f (u0)f(p
f(p2 ) . . . f(p
f (u0)f(p
f(p2 )
s)
1)
1)
21 .(m+n) 22 .(m+n)
2s .(m+n)
2
= f (u0)f(p1 )
f(p2 )
. . . f(ps )

= f 2 (u0)f(p11)

Vy ta c iu phi chng minh.

s .2n
. . . f(p
s)

Chng 8

Mt s bi ton xc nh dy
s trong lp dy tun hon
cng tnh v nhn tnh.
8.1

Mt s bi ton xc nh dy s trong lp dy
tun hon cng tnh

Bi ton 8.1. Xc nh dy {xn } sao cho xn+3 = xn + 1, n = 1; 2; 3; . . .


Gii. t xn =

n
3

+ yn . Khi ta c
yn+3 +

n+3
n
= + yn + 1
3
3

hay yn+3 = yn , n N. Vy nn

y0 = y3 = y6 =
a tu
y1 = y4 = y7 = yn = b tu

c tu
y 2 = y5 = y8 =

vi n = 3k, k N
vi n = 3k + 1, k N
vi n = 3k + 2, k N

Bi ton 8.2. Xc nh dy s {xn } sao cho xn+3 = 2xn


n
Gii. t xn = 2 3 yn . Suy ra
n+3
n
2 3 yn+3 = 2(2 3 yn ) yn+3 = yn yn+3 = yn .

167

8.1. Mt s bi ton xc nh dy s trong lp dy tun hon cng tnh

Vy nn

y0 = y3 = y6 =
y 1 = y4 = y7 =

y 2 = y5 = y8 =

a tu
yn = b tu

c tu

168

vi n = 3k, k N
vi n = 3k + 1, k N
vi n = 3k + 2, k N

Vy xn = 2 3 yn trong

a tu
yn = b tu

c tu

vi n = 3k, k N
vi n = 3k + 1, k N
vi n = 3k + 2, k N

Sau y ta xy dng bi ton tng qut sau.


Bi ton 8.3. Xc nh dy s {un } tho mn iu kin
un+b = un + d
Trong n, b N, d R
Gii. t un =

d
n + vn . Thay vo cng thc un+b = un + d, ta c
b
d
d
(n + b) + vn+b = n + vn + d.
b
b

Suy ra vn+b = vn do vn l dy tun hon cng tnh chu k b


d
Vy un = n + vn vi vn l dy tun hon cng tnh chu k b.
b
Bi ton 8.4. Xc nh dy s {un } tho mn iu kin
un+b = c.un ,
trong n, b N, c R
n

Gii. t un = c b vn ta c
c

n+b
b

vn+b = cc b vn .
n

Suy ra vn+b = vn do vn l dy tun hon cng tnh chu k b Vy un = c b vn


vi vn l dy tun hon cng tnh chu k b.
Bi ton 8.5. Xc nh dy s {un } tho mn iu kin
un+b = cun + d,
trong n, b N, c, d R.

8.1. Mt s bi ton xc nh dy s trong lp dy tun hon cng tnh

169

Gii. Xt trng hp c = 1 theo kt qu bi ton 8.3, ta c


un =

d
n + vn
b

vi vn l dy tun hon cng tnh chu k b.


d
Xt trng hp c 6= 1. t un = vn +
. Khi ta c
1c
vn+b +

d
d
= c(vn +
) + d, n N
1c
1c

hay
vn+b = cvn .
n

Theo kt qu bi ton 8.4, ta c vn = | c | b xn , trong


(
vi c > 0
xn
xn+b =
xn
vi c < 0
Vy nn

f (x) =

n
d
+ c b xn , vi xn tu sao cho xn+b = xn , vi c > 0
1c

n
d
+ | c | b xn , vi xn tu sao cho xn+b = xn , vi c < 0
1c

Kt lun:
- Nu c = 1 th un =

d
n + vn vi vn l dy tun hon cng tnh chu k b.
b

- Nu c 6= 1 th

n
d

+ c b xn , vi xn tu sao cho xn+b = xn , vi c > 0

1c
f (x) =

n
d

+ | c | b xn , vi xn tu sao cho xn+b = xn vi c < 0


1c
Bi ton 8.6.

8.2. Hm s xc nh trn tp cc s nguyn

8.2
8.2.1

170

Hm s xc nh trn tp cc s nguyn
Hm s chuyn i cc php tnh s hc

Khi xt lp phng trnh hm vi cp ch s t do dng i xng quen bit, ta


thng s dng php th ch s bng cc bin mi a phng trnh hm
cho v mt dng phng trnh hm mi bit cch gii. Tuy nhin, trong cc
trng hp s dng php th ch s tng qut, nghim nhn c ca phng trnh
mi, nhn chung khng tho mn iu kin bi ra. V vy, nghim ca phng
trnh mi cn c th li thng qua cc d liu ca bi ra. Ta xt mt s v d
minh ho.
Bi ton 8.7. Xc nh hm s f : Z R tho mn cc iu kin
f (m + n) = f (m) + f (n) + mn (m, n Z).

(1)

Gii. T phng trnh (1) ta nhn c


f (n + 1) = f (1) + f (n) + n, n Z,
hay
f (n + 1) f (n) = a + n, a = f (1), n Z.

(2)

Phng trnh f (n + 1) f (n) = a + n l mt phng trnh sai phn tuyn tnh


khng thun nht cp 1. Do phng trnh c trng tng ng c nghim = 1,
nn ta c nghim tng qut ca phng trnh thun nht f (n + 1) f (n) = 0 l
f(n) = c
Ta vit
n=

(3)

1
1
1
(n + 1)2 n2 .
2
2
2

Khi , nghim ring ca (2) c dng


f (n) = n(dn + e).
Thay f (n) vo (2) ta c

1
f (n) = a
n.
2

(4)

V f (n) = f(n) + f (n) nn t (3) v (4) ta c nghim ca (2) l



1
1
f (n) = c + n2 + a
n.
2
2

(5)

171

8.2. Hm s xc nh trn tp cc s nguyn

Do f (1) = a, t (5) ta c c = 0.
Thay c = 0 vo (5), ta thu c nghim ca

1
f (n) = n2 + a
2

(2)
1
.
2

(6)

Th li ta thy nghim dng (6) tho mn iu kin ca u bi.


Bi ton 8.8. Tn ti hay khng tn ti mt hm s f : Z R tho mn iu
kin
f (m + n) = f (m) + f (n) + m + n, (m, n Z).
(7)
Gii. Lp li cch gii nh i vi Bi ton 1, t phng trnh (l) ta suy ra
f (n + 1) = f (1) + f (n) + n + 1
hay
f (n + 1) f (n) = a + n vi a = f (1) + 1.

(8)

Phng trnh f (n + 1) f (n) = a + n l mt phng trnh sai phn tuyn tnh


khng thun nht cp 1. Do phng trnh c trng c nghim = 1 nn ta c
nghim tng qut ca phng trnh thun nht f (n + 1) f (n) = 0 l
f(n) = c

(9)

Ta vit

1
1
1
(n + 1)2 n2 .
2
2
2
0
Khi , nghim ring ca (8) c dng xn = n(dn + e). Thay f (n) vo (8) ta
c

1
1
f (n) = n2 + a
n.
(10)
2
2
V f (n) = f(n) + f (n) nn t (9) v (10) ta c nghim ca (8) l
n=


1
1
f (n) = c + n2 + a
n.
2
2
Do f (1) = a 1, t (11) ta c c = 1.
Thay c = 1 vo (11), ta c nghim ca (8)

1
1
f (n) = n2 + a
n + 1.
2
2

(11)

(12)

Th li ta thy nghim dng (12) khng tho mn iu kin ca u bi. Vy


khng tn ti mt hm s f : Z R tho mn iu kin
f (m + n) = f (m). + f (n) + m + n (m, n Z).

8.2. Hm s xc nh trn tp cc s nguyn

172

Bi ton 8.9. Xc nh f : Z R+ tho mn iu kin


f (mn) = f (m)f (n) (m, n Z).
Gii.
Ta c x1.n = f (1)f (n). Suy ra f (1) = 1. Gi s n = p l mt s nguyn t.
s
1
Khi f (pk ) = (f (p))k (quy np) v nu n = p
1 . . . ps th
f (n) = (f (p1))1 (f (ps))s .
Vy f (p) c th nhn gi tr tu khi p l mt s nguyn t.
Kt lun:
f (p) c th nhn gi tr tu khi p l mt s nguyn t v
f (n) = (f (p1))1 (f (ps ))s
s
1
khi n = p
1 . . . tps .

Bi ton 8.10. Xc nh dy f : Z R tho mn iu kin


F (m + n) + f (n m) = f (3n) (m, n Z, n m).
Gii.
Cho m = 0, ta c 2f (n) = f (3n). Suy ra f (0) = 0. t m = n ta c
f (2n) = f (3n). Suy ra, mt mt th
f (4n) = f (6n) = f (9n)
v mt khc th
f (4n) + f (2n) = f (9n).
T suy ra
1
1
f (n) = f (3n) = f (2n) = 0
2
2
vi mi n Z.

8.2.2

Hm s chuyn tip cc i lng trung bnh

Trong mc ny, ta i tm nhng hm s thc hin php chuyn tip mt i


lng trung bnh ca cp ch s sang mt i lng trung bnh ca cp phn t
tng ng ca hm s. Cc bi ton ny lin quan cht ch n vic chuyn tip
cc cp s; n s m phng cc cp s tng qut, chng hn, ta c th chuyn
mt cp s cng sang mt cp s nhn, cp s iu ho,...
Di y ta xt mt s bi ton chuyn tip cc i lng trung bnh c bn
trong chng trnh ph thng.
1) Php chuyn cc i lng trung bnh cng

173

8.2. Hm s xc nh trn tp cc s nguyn

Bi ton 8.11. Xc nh hm s u(n), sao cho






m+n
u(m) + u(n)
m+n
u
=
m, n,
Z
2
2
2
Gii. t u(1) = , u(2) = . Ta c


3+1
u(3) + u(1)
u(2) = u
=
.
2
2
Suy ra
u(3) = 2u(2) u(1) = 2 .
Tip tc qu trnh nh vy, ta c


4+2
u(4) + u(2)
u(3) = u
=
.
2
2
Suy ra
u(4) = 2u(3) u(2) = 2(2 ) = 3 2.
Bng phng php quy np, ta thu c
u(n) = (n 1) (n 2), n.
Vy

(
u(n) =
u(1) =

( )n + 2 , n,
, u(2) = .

t = a + b; = 2a + b, th a = v b = 2 .
Do , nghim ca phng trnh l u(n) = an + b; a, b tu .
2) Php chuyn i lng trung bnh cng sang trung bnh iu ho
Bi ton 8.12. Xc nh hm s u(n) Z sao cho




m+n
2u(m)u(n)
m+n
u
=
m, n,
Z .
2
u(m) + u(n)
2
Gii. Ta c




m+n
2u(m)u(n)
m+n
u
=
u
=
2
u(m) + u(n)
2
t

1
u(n)

2
.
1
1
+
u(m) u(n)

= v(n), th phng trnh cho tng ng vi




m+n
v(m) + v(n)
v
=
.
2
2

174

8.2. Hm s xc nh trn tp cc s nguyn

Theo Bi ton 1, v(n) = an + b; a, b 0, a + b > 0.


Vy nghim ca phng trnh l
u(n) =

1
; a, b 0; a + b > 0.
an + b

3) Php chuyn i lng trung bnh cng sang trung buh nhn
Bi ton 8.13. Xc nh hm s u(n) sao cho




p
m+n
m+n
u
= u(m)u(n); m, n,
Z.
2
2
Gii. Ta c
u(n) = u

n+n
2

u(n)u(n) =

p
[u(n)]2 = |u(n)| 0.

t u(1) = , u(2) = ( 0, 0).


a) Nu = 0 th


p
1 + 2n 1
u(n) = u
= u(1)u(2n 1) = 0, n Z.
2
Vy u(n) 0 l nghim duy nht ca phng trnh .
b) Nu > 0 v = 0 th


p
2 + 2n 2
u(n) = u
= u(2)u(2n 2) = 0, n 2.
2
Suy ra
u(n) =

, nu n = 1
0 nu n 2

l nghim ca phng trnh.


c) Xt trng hp > 0 v > 0. Gi s tn ti n0 3 sao cho u(n0 ) = 0.
Th th


p
n 0 + n0 2
u(n0 1) = u
= u(n0 )u(n0 2) = 0.
2
Chn n0 = 3 th u(n0 1) = u(2) = 0, hay = 0, mu thun.
Do , c th gi thit rng u(n) > 0 vi mi n Z. Ta c


p
3+1
u(2) = u
= u(3)u(1).
2

175

8.2. Hm s xc nh trn tp cc s nguyn

Suy ra
u(3) =
Mt khc
u(3) = u

u2(2)
2
=
.
u(1)

4+2
2

Suy ra
2

u(4) =

u (3)
=
u(2)

u(4)u(2).

2

3
.
2

Bng phng php quy np ton hc, ta chng minh c rng


u(n) =
M

n1
, n 3.
n2

n1
=
n2
(

  n

= ab,
(a > 0, b > 0).
= ab2

Suy ra
2

= a,
= b.

Vy nghim ca phng trnh l


(
u(n) =

nu n = 1
( 0)
0 nu n 2

hoc u(n) = a.bn (a > 0, b > 0).


4) Php chuyn i lng trung bnh cng sang trung bnh bc hai
Bi ton 8.14. Xc nh hm s u(n), sao cho

 r 2


m+n
u (m) + u2 (n)
m+n
u
=
m, n,
Z .
2
2
2

176

8.2. Hm s xc nh trn tp cc s nguyn

Gii. Ta c
u(n) = u

n+n
2

u2 (n) + u2 (n) p 2
= u (n) = |u(n)| 0, n Z.
2

t u(1) = 0 ; u(2) = 0. Ta c

 r 2
3+1
u (3) + u2 (1)
u(2) = u
=
.
2
2
Suy ra
u2(3) = 2u2(2) u2 (1) = 2 2 2
p

u(3) = 2 2 2 ( 2).
Tng t
u(3) = u

4+2
2

u2 (4) + u2 (2)
.
2

Suy ra
u2(4) = 2u2 (3) u2 (2) = 2(2 2 2 ) 2 = 3 2 22
hay
u(4) =

3 2 22

r 
3

.
2

Bng quy np ton hc, ta chng minh c h thc


p
u(n) = (n 1) 2 (n 2)2, n 3.
Nhn xt rng, ta lun c
p
p
(n 1) 2 (n 2)2 = ( 2 2 )n + 22 2 .
t

Suy ra

2 = a + b
2 = 2a + b.

a = 2 2
b = 22 2 .

Vy nghim ca phng trnh l u(n) = an + b ; a 0, a + b 0.


Nhn xt 8.1. Trong c bn bi ton nu trn, nu ta thay m bi (n + 1)
v n bi (n 1) th ta c th a chng c v cc phng trnh sai phn quen
bit.

8.2. Hm s xc nh trn tp cc s nguyn

8.2.3

177

Phng trnh trong hm s vi cp bin t do

Trong mc ny, ta i tm nhng hm s thc hin php chuyn tip mt biu


thc i s ca cp ch s sang mt i lng khc ca cp phn t tng ng
ca dy s. Cc bi ton ny lin quan cht ch n vic chuyn tip cc hm
s; n s m phng cc hm s c bit trong s hc, i s,...
Bi ton 8.15. Tm hm f : Z Z tha mn cc iu kin f (1) = a Z v
f (m + n) + f (m n) = 2f (m)f (n), m, n Z.
Gii. Cho m = n = 0 ta c f (0) = 0 hoc f (0) = 1. Nu f (0) = 0 th thay
n = 0 ta c 2f (m) = 0 vi mi m Z. Do vy f (m) 0 v ng vi a = 0.
Nu f (0) = 1, cho m = n = 1 ta thu c f (2) = 2a2 1.
Tip tc thay m = 2; n = 1 vo iu kin bi ra ta c f (3) = 4a3 3a. T
ta c d on f (n) = Tn (a) vi mi n 1.
D on c chng minh d dng bng phng php quy np.
Mt khc, cho m = 0 ta c f (n)+f (n) = 2f (0)f (n) = 2f (n) nn f (n) =
f (n). Vy f (n) l hm chn. Vy ta c

1 khi m = 0,
f (m) =
a khi m = 1,

T|m|(a) khi |m| 1, m Z.


Bi ton 8.16. Tm hm f : Z R tha mn cc iu kin f (0) 6= 0, f (1) =

5
2

v
f (m + n) + f (m n) = f (m)f (n), m, n Z.
Gii. Cho m = n = 0 ta c, do f (0) 6= 0, f (0) = 2. Tip theo, theo quy np
ta c
f (n) = 2n + 2n , n Z.
Th li ta thy hm ny tho mn iu kin bi ra.
Bi ton 8.17. Tm hm f : Z [0, +) tha mn cc iu kin f (1) = 1 v
f (m + n) + f (m n) =

1
[f (2m) + f (2n)], m, n Z, m n.
2

Gii. Cho m = n = 0 ta c f (0) = 0. Cho m = 1, n = 0 th


1
f (1) + f (1) = [f (2) + f (0)].
2
Suy ra f (2) = 4.

8.2. Hm s xc nh trn tp cc s nguyn

178

Chng minh bng quy np ta c f (n) = n2 .


Tht vy, do f (k) + f (k) = 12 [f (2k) + f (0)] nn c ngay f (2k) = 4k2 .
Cng vy, do f (k + 1) + f (k 1) = 12 [f (2k) + f (2)] nn ta c
1
f (k + 1) = f (2k) + 2 f (k 1) = (k + 1)2.
2
Bi ton 8.18. Tm cc a thc hai bin P (m, n) (m, n Z) tho mn iu
kin
a) P (am, an) = a2 P (m, n) vi mi m, n, a Z,
b) P (b + c, a) + P (c + a, b) + P (a + b, c) = 0 vi mi a, b, c Z,
c) P (1, 0) = 1.
Gii.
Trong b) t b = 1 a; c = 0 ta c
P (1 a, a) = 1 P (a, 1 a).

(1)

Li t c = 1 a b v kt hp vi a) ta c
P (a + b, 1 a b) = P (a, 1 a) + P (b, 1 b) + 2.

(2)

t f (m) = P (m, 1 m) + 2. Khi f (1) = P (1, 0) + 2 = 3 v (5) tr thnh


f (m + n) = f (m) + f (n). l phng trnh dy chuyn i php cng
(
f (m + n) = f (m) + f (n),
(3)
f (1) = 3.
Phng trnh (3) c nghim duy nht f (n) = 3n. Vy nn
P (n, 1 n) = 3n 2.

(4)

Bng phng php quy np ta s thu c


P (a, b) = (a + b)2 3


a
2 = (a + b)(a 2b), a, b Z.
a+b

Tm li P (m, n) = (m + n)2 (m 2n).


Bi ton 8.19. Cho a thc Chebyshev Tn (x) = cos(n arccos x). Chng minh
rng vi m, n Z; n m v x R th Tn (x) l nghim ca phng trnh dy
sau
Tn+m (x) + Tnm (x) = 2Tn (x)Tm(x).

8.2. Hm s xc nh trn tp cc s nguyn

179

Gii. S ng nh ngha Tn (x) v phng php quy np hoc s dng cc cng


thc
cos(n + m)x + cos(n m)x = 2 cos nx cos mx
v
cosh(n + m)x + cosh(n m)x = 2 cosh(nx) cosh(mx),
ta c ngay iu phi chng minh.
Bi ton 8.20. Tm hm f Z Z tha mn cc iu kin
N Z : N < f (n) < N n Z,
f (m + n) + f (m n) = 2f (m)f (n) m, n Z.

(1)

Gii. Cho m = n = 0 ta c f (0) {0, 1}. Gi s f (0) = 0. Cho n = 0 trong


(1) ta c 2f (m) = 2f (m)f (0) = 0 v f 0.
Gi s f (0) = 1. Cho m = 0 trong (1) ta thu c f (n) = f (n) vi mi
n Z. Vy ch cn xt n Z. Cho n = 1 trong (1), ta c
f (m + 1) = 2f (m)f (1) f (m 1)
v thu c cng thc truy hi theo f (1). Nu |f (1)| 2 th t gi thit ta c
f (2n) = 2[f (n)]2 1
tng v khng gii ni, tri vi gi thit. Vy f (1) {1, 0, 1}.
Vi f (1) = 1 th f (n) = (1)n (quy np).
Vi f (1) = 1 th f (n) 1.
Vi f (1) = 0 ta c dy tun hon (quy np)
f (4m) = 1, f (4m + 1) = 0, f (4m + 2) = 1, f (4m + 3) = 0.
Suy ra f (2) = 4. Chng minh bng quy np ta c f (n) = n2 . Tht vy, do
f (k) + f (k) = (1/2)[f (2k) + f (0)] nn c ngay f (2k) = 4k2 . Cng vy, do f (k +
1) + f (k 1) = (1/2)[f (2k) + f (2)] nn ta c
1
f (k + 1) = f (2k) + 2 f (k 1) = (k + 1)2.
2

180

8.2. Hm s xc nh trn tp cc s nguyn

8.2.4

Mt s dng ton lin quan n dy truy hi

Bi ton 8.21. K hiu


un =

sinn xdx, n Z.

Xc nh hm s f : Z R theo cng thc


f (n) = (n + 1)un g(n + 1), n Z.
Gii. S dng cng thc tch phn tng phn, ta thu c
n1

un = cos x sin


x 02 +

(n 1) sinn2 x cos2 xdx

(n 1) sinn2 x(1 sin2 x)dx

= (n 1)(un1 un ).
T y suy ra
g(n + 2) =

n+1
un , n Z.
n+2

T (1) ta nhn c
f (n + 1) = (n + 2)g(n + 1)g(n + 2)
= (n + 2)g(n + 1)

n+1
un
n+2

= (n + 1)g(n + 1)un = f (n).


Vy nn
f (n) = f (0) =

.
2

Bi ton 8.22. K hiu


un =

cosn x cos nxdx, n Z.


0

Xc nh hm s f : Z R theo cng thc


f (n) = 2n un , n Z.

(1)

181

8.2. Hm s xc nh trn tp cc s nguyn

Gii. t cosn x = u v cos nxdx = dv th theo cng thc tch phn tng phn,
ta thu c
Z

1
un = cosn x sin x 0 +
cosn1 x sin x sin nxdx
n
0
Z
1
=
cosn1 x[cos(n 1)x + cos(n + 1)x]dx
2 0
Z
1
1
1
= un1 un +
cosn1 x sin x + sin nxdx
2
2
2 0
1
1
1
un1 un + un .
2
2
2

=
Vy nn
un =

1
1
1
1
un1 = un2 = = n1 u1 = n1 .
2
4
2
2
2

Bi ton 8.23. Xc nh hm s {un } c tnh theo cng thc


un =

tan2n xdx

Gii. Ta vit un di dng sau


un =

tan2n xdx

tan2n2 x[(tan2 x + 1) 1]dx

tan2n2 xd tan x un1

tan2n1 x 4
un1
2n 1 0
1
=
un1 .
2n 1

Do vy
1
,
2n 1
1
=
,
2n 3

un + un1 =
un1 + un2

182

8.2. Hm s xc nh trn tp cc s nguyn

u1 + u 0 =
Suy ra
un = (1)

1
.
21

n
X
(1)k i
+
.
4
2k 1
k=1

Bi ton 8.24. Xc nh hm s f : Z R c tnh theo cng thc


Z1

f (n) =

xn 1 xdx, n Z.

Gii. t xn = u,

1 xdx = dv th
f (n) =

Z1

xn 1 xdx

3  1
2
2n
= xn (1 x) 2 0 +
3
3

Z1

xn1 1 x(1 x)dx

2n
=
3

Z1

(xn1 xn ) 1 xdx

2n
=
3

Z1

(x

n1

2n
1 xdx
3

Z1

xn 1 xdx

2n
2n
xn1
f (n).
3
3
Vy nn
f (n) =
V f (0) =

2n
xn1 .
2n 3

2
nn ta c ngay
3
f (n) =

Z1

(2n)!!
xn 1 xdx = 2
, n Z.
(2n + 3)!!

Bi ton 8.25. Xc nh hm f : Z Z tho mn cc iu kin


f (0) = 1, f (f (n) = f (f (n + 2) + 2) = n n Z.

8.2. Hm s xc nh trn tp cc s nguyn

183

Gii. Nhn xt rng f l nh x 1-1


f (m) = f (n) f (f (m)) = f (f (n)) m = n.
Vy nn
f (n) = f (n + 2) + 2 n Z.
Suy ra
f (n + 2) = f (n) 2, f (0) = 1, f (1) = f (f (0)) = 0.
Vy nn
f (2) = f (0) 2 = 1,
f (3) = f (1) 2 = 2,
f (n) = (n 1).
Tng t
f (1) = f (1) + 2 = 2,
f (2) = f (0) + 2 = 3,
f (3) = f (1) + 2 = 5,
f (n) = (n 1).
Bi ton 8.26. Cho gc vi 0 < < . Xc nh cp s a, b sao cho dy hm
{Pn (x)} c tnh theo cng thc
Pn (x) = xn sin x sin(n) + sin(n 1)
lun lun chia ht cho f (x) = x2 + ax + b.
Gii. Vi n = 3 th
P3 (x) = x3 sin x sin(3) + sin 2 = sin (x + 2 cos )(x2 2x cos + 1).
.
T suy ra vi f (x) = x2 + 2x cos + 1 th P3(x)..f (x). Vi n 3 th
Pn+1 (x) = xPn (x) + (x2 2x cos + 1) sin n.
Suy ra f (x) = x2 + 2x cos + 1.

184

8.3. Hm s xc nh trn tp cc s hu t

Bi tp
Bi 1. Xc nh hm s f : Z R nu bit: f (1) = a, f (m + n) = f (n) + f (m);
Bi 2. Xc nh hm s f : Z R nu bit: f (1) = a, f (m n) = f (n) + f (m)
(m, n, m n Z).
 
Bi 3. Xc nh hm s f : Z R nu bit f m
= f (n)+f (m) (m, n, m
n
n Z).
 
Bi 4. Xc nh hm s f : Z R tho mn iu kin f m
= f (m) f (n)
n
m
(m, n, n Z).
Bi 5. Xc nh hm s f : Z R tho mn iu kin f (m + n) = f (m)f (n)
(m, n Z).
Bi 6. Xc nh hm s f : Z R nu bit f (m + n) + f (m n) = 12 (f (2m) +
f (2n) , (m, n, m n Z).
Bi 7. Xc nh hm s f : Z R+ tho mn iu kin f (m + n) = ff(m)
(n)
(m, n Z).
 p

Bi 8. Xc nh hm s f : Z R+ tho mn iu kin f m+n
= f (m)f (n)
2
(m, n, m+n
2 Z).
Bi 9 . Xc nh hm s f : Z R+ tho mn iu kin x m+n =
2

(m, n, m+n
2 Z).

Bi 10 . Xc nh hm s f : Z R+ tho mn iu kin f
q
f (m)2 +f (n)2
(m, n, m+n
2
2 Z).

8.3

2f (m)f (n)
f (m)+f (n)

m+n
2

Hm s xc nh trn tp cc s hu t

Bi ton 8.27. Xc nh hm s f : Q R tho mn cc iu kin


f ( + ) = f () + f () + (, Q).

(1)

Gii. T phng trnh (1) ta nhn c


f ( + 1) = f (1) + f () +
hay
f ( + 1) f () = a + , a = f (1).

(2)

Phng trnh f ( + 1) f () = a + l mt phng trnh sai phn tuyn tnh


khng thun nht cp 1. Do phng trnh c trng c nghim = 1 nn ta c
nghim tng qut ca phng trnh thun nht f ( + 1) f () = 0 l
f() = c

(3)

8.3. Hm s xc nh trn tp cc s hu t

Ta vit
=

185

1
1
1
( + 1)2 2 .
2
2
2

Khi , nghim ring ca (2) c dng


f () = (d + e).
Thay f () vo (2) ta c

1
f () = a
.
2

(4)

V f () = f() + f () nn t (3) v (4) ta c nghim ca (2) l



1
1
f () = c + 2 + a
n.
2
2

(5)

Do f (1) = a, t (5) ta c c = 0.
Thay c = 0 vo (5), ta thu c nghim ca (2)

1
1
f () = 2 + a
.
2
2

(6)

Th li ta thy nghim dng (6) tho mn iu kin ca u bi.


Bi ton 8.28. Tn ti hay khng tn ti mt hm s f : Q R tho mn
iu kin
f ( + n) = f () + f () + + n, (, Q).
(7)
Gii. Lp li cch gii nh i vi Bi ton trn, t phng trnh (l) ta suy ra
f ( + 1) = f (1) + f () + + 1
hay
f ( + 1) f () = a + vi a = f (1) + 1.

(8)

Phng trnh f ( + 1) f () = a + l mt phng trnh sai phn tuyn tnh


khng thun nht cp 1. Do phng trnh c trng c nghim = 1 nn ta c
nghim tng qut ca phng trnh thun nht f ( + 1) f () = 0 l
f() = c
Ta vit
=

1
1
1
( + 1)2 2 .
2
2
2

(9)

8.3. Hm s xc nh trn tp cc s hu t

186

Khi , nghim ring ca (8) c dng x0n = n(d + e). Thay f () vo (8) ta
c

1
1
f () = 2 + a
.
(10)
2
2
V f () = f() + f () nn t (9) v (10) ta c nghim ca (8) l

1
1
f () = c + 2 + a
.
2
2

(11)

Do f (1) = a 1, t (11) ta c c = 1.
Thay c = 1 vo (11), ta c nghim ca (8)

1
1
f () = 2 + a
+ 1.
2
2

(12)

Th li ta thy nghim dng (12) khng tho mn iu kin ca u bi. Vy


khng tn ti mt hm s f : Q R tho mn iu kin
f ( + ) = f (). + f () + + (, Q).
Bi ton 8.29. Xc nh f : Q R+ tho mn iu kin
f () = f ()f () (, Q).
Gii. Ta c x1. = f (1)f (). Suy ra f (1) = 1. Gi s = p l mt s nguyn
s
1
t. Khi f (pk ) = (f (p))k (quy np) v nu = p
1 . . . ps th
f () = (f (p1))1 (f (ps))s .
Vy f (p) c th nhn gi tr tu khi p l mt s nguyn t.
Kt lun:
f (p) c th nhn gi tr tu khi p l mt s nguyn t v
f () = (f (p1))1 (f (ps ))s
s
1
khi = p
1 . . . tps .

Bi ton 8.30. Xc nh dy f : Q R tho mn iu kin


F ( + ) + f ( ) = f (3) (, Q, ).
Gii. Cho = 0, ta c 2f () = f (3). Suy ra f (0) = 0. t = ta c
f (2) = f (3). Suy ra, mt mt th
f (4) = f (6) = f (9)

8.3. Hm s xc nh trn tp cc s hu t

187

v mt khc th
f (4) + f (2) = f (9).
T suy ra
1
1
f () = f (3) = f (2) = 0
2
2
vi mi Q.
Tip theo, ta i tm nhng hm s thc hin php chuyn tip mt i lng
trung bnh ca cp ch s sang mt i lng trung bnh ca cp phn t tng
ng ca hm s. Cc bi ton ny lin quan cht ch n vic chuyn tip cc
cp s; n s m phng cc cp s tng qut, chng hn, ta c th chuyn mt
cp s cng sang mt cp s nhn, cp s iu ho,...
Di y ta xt mt s bi ton chuyn tip cc i lng trung bnh c bn
trong chng trnh ph thng.
1) Php chuyn cc i lng trung bnh cng
Bi ton 8.31. Xc nh hm s u(), sao cho




+
u() + u()
+
u
=
, ,
Q
2
2
2
Gii. t u(1) = , u(2) = . Ta c


3+1
u(3) + u(1)
u(2) = u
=
.
2
2
Suy ra
u(3) = 2u(2) u(1) = 2 .
Tip tc qu trnh nh vy, ta c


4+2
u(4) + u(2)
u(3) = u
=
.
2
2
Suy ra
u(4) = 2u(3) u(2) = 2(2 ) = 3 2.
Bng phng php quy np, ta thu c
u() = (n 1) (n 2), n.
Vy

(
u() =
u(1) =

( )n + 2 , n,
, u(2) = .

t = a + b; = 2a + b, th a = v b = 2 .
Do , nghim ca phng trnh l u() = an + b; a, b tu .
2) Php chuyn i lng trung bnh cng sang trung bnh iu ho

188

8.3. Hm s xc nh trn tp cc s hu t

Bi ton 8.32. Xc nh hm s u() Q sao cho






+
2u()u()
+
u
=
, ,
Q .
2
u() + u()
2
Gii. Ta c
u

1
u()

+
2

2u()u()
u
=
u() + u()

+
2

2
.
1
1
+
u() u()

= v(), th phng trnh cho tng ng vi


v

+
2

v() + v()
.
2

Theo Bi ton 1, v() = a + b; a, b 0, a + b > 0.


Vy nghim ca phng trnh l
u() =

1
; a, b 0; a + b > 0.
a + b

3) Php chuyn i lng trung bnh cng sang trung buh nhn
Bi ton 8.33. Xc nh hm s u() sao cho




p
+
+
u
= u()u(); , ,
Q.
2
2
Gii. Ta c
u() = u

+
2

u()u() =

[u()]2 = |u()| 0.

t u(1) = , u(2) = ( 0, 0).


a) Nu = 0 th


p
1 + 2 1
u() = u
= u(1)u(2n 1) = 0, Q.
2
Vy u() 0 l nghim duy nht ca phng trnh .
b) Nu > 0 v = 0 th


p
2 + 2 2
u() = u
= u(2)u(2n 2) = 0, 2.
2

189

8.3. Hm s xc nh trn tp cc s hu t

Suy ra
u() =

, nu = 1
0 nu 2

l nghim ca phng trnh.


c) Xt trng hp > 0 v > 0. Gi s tn ti n0 3 sao cho u(n0 ) = 0.
Th th


p
n 0 + n0 2
u(n0 1) = u
= u(n0 )u(n0 2) = 0.
2
Chn n0 = 3 th u(n0 1) = u(2) = 0, hay = 0, mu thun.
Do , c th gi thit rng u() > 0 vi mi Q. Ta c


p
3+1
u(2) = u
= u(3)u(1).
2
Suy ra
u2(2)
2
=
.
u(1)

u(3) =
Mt khc
u(3) = u

4+2
2

Suy ra
u(4) =

u2 (3)
u(2)


=

u(4)u(2).

2

3
.
2

Bng phng php quy np ton hc, ta chng minh c rng


u() =

1
, 3.
2

m
1
=
2
t

  n

= ab,
(a > 0, b > 0).
= ab2

Suy ra
2

= a,
= b.

190

8.3. Hm s xc nh trn tp cc s hu t

Vy nghim ca phng trnh l


(
u() =

nu = 1
( 0)
0 nu 2

hoc u() = a.b (a > 0, b > 0).


4) Php chuyn i lng trung bnh cng sang trung bnh bc hai
Bi ton 8.34. Xc nh hm s u(), sao cho

 r 2


+
u () + u2 ()
+
u
=
, ,
Q .
2
2
2
Gii. Ta c
u() = u

+
2

u2 () + u2 () p 2
= u () = |u()| 0, Q.
2

t u(1) = 0 ; u(2) = 0. Ta c
u(2) = u

3+1
2

u2 (3) + u2 (1)
.
2

Suy ra
u2(3) = 2u2(2) u2 (1) = 2 2 2
p

u(3) = 2 2 2 ( 2).
Tng t
u(3) = u

4+2
2

u2 (4) + u2 (2)
.
2

Suy ra
u2(4) = 2u2 (3) u2 (2) = 2(2 2 2 ) 2 = 3 2 22
hay
u(4) =

3 2

22

r 
3

.
2

Bng quy np ton hc, ta chng minh c h thc


p
u() = (n 1) 2 (n 2)2, n 3.
Nhn xt rng, ta lun c
p
p
( 1) 2 ( 2)2 = ( 2 2 ) + 22 2 .

8.4. Phng trnh trong hm s vi cp bin t do

Suy ra

191

2 = a + b
2 = 2a + b.

a = 2 2
b = 22 2 .

Vy nghim ca phng trnh l u() = a + b ; a 0, a + b 0.


Nhn xt 8.2. Trong c bn bi ton nu trn, nu ta thay m bi (n + 1)
v n bi (n 1) th ta c th a chng c v cc phng trnh sai phn quen
bit.

8.4

Phng trnh trong hm s vi cp bin t do

Trong mc ny, ta i tm nhng hm s thc hin php chuyn tip mt biu


thc i s ca cp ch s sang mt i lng khc ca cp phn t tng ng
ca dy s. Cc bi ton ny lin quan cht ch n vic chuyn tip cc hm
s; n s m phng cc hm s c bit trong s hc, i s,...
Bi ton 8.35. Tm hm f : Q Q tha mn cc iu kin f (1) = a Q v
f ( + n) + f (m ) = 2f ()f (), m, Q.
Gii. Cho m = n = 0 ta c f (0) = 0 hoc f (0) = 1. Nu f (0) = 0 th thay
n = 0 ta c 2f () = 0 vi mi m Q. Do vy f () 0 v ng vi a = 0.
Nu f (0) = 1, cho m = n = 1 ta thu c f (2) = 2a2 1.
Tip tc thay m = 2; n = 1 vo iu kin bi ra ta c f (3) = 4a3 3a. T
ta c d on f () = Tn (a) vi mi 1.
D on c chng minh d dng bng phng php quy np.
mt khc, cho m = 0 ta c f ()+f () = 2f (0)f () = 2f () nn f () =
f (). Vy f () l hm chn. Vy ta c

1 khi m = 0,
f () =
a khi m = 1,

T|m|(a) khi |m| 1, m Q.


Bi ton 8.36. Tm hm f : Q R tha mn cc iu kin f (0) 6= 0, f (1) =
v
f ( + n) + f (m ) = f ()f (), m, Q.

5
2

8.4. Phng trnh trong hm s vi cp bin t do

192

Gii. Cho m = n = 0 ta c, do f (0) 6= 0, f (0) = 2. Tip theo, theo quy np


ta c
f () = 2 + 2 , Q.
Th li ta thy hm ny tho mn iu kin bi ra.
Bi ton 8.37. Tm hm f : Q [0, +) tha mn cc iu kin f (1) = 1 v
1
f ( + n) + f (m ) = [f (2m) + f (2n)], m, Q, m n.
2
Gii. Cho m = n = 0 ta c f (0) = 0. Cho m = 1, = 0 th
1
f (1) + f (1) = [f (2) + f (0)].
2
Suy ra f (2) = 4.
Chng minh bng quy np ta c f () = 2 .
Tht vy, do f (k) + f (k) = 12 [f (2k) + f (0)] nn c ngay f (2k) = 4k2 .
Cng vy, do f (k + 1) + f (k 1) = 12 [f (2k) + f (2)] nn ta c
1
f (k + 1) = f (2k) + 2 f (k 1) = (k + 1)2.
2
Bi ton 8.38. Tm cc a thc hai bin P (m, n) (m, Q) tho mn iu
kin
a) P (am, an) = a2 P (m, n) vi mi m, n, a Q,
b) P (b + c, a) + P (c + a, b) + P (a + b, c) = 0 vi mi a, b, c Q,
c) P (1, 0) = 1.
Gii. Trong b) t b = 1 a; c = 0 ta c
P (1 a, a) = 1 P (a, 1 a).

(1)

Li t c = 1 a b v kt hp vi a) ta c
P (a + b, 1 a b) = P (a, 1 a) + P (b, 1 b) + 2.

(2)

t f () = P (m, 1 ) + 2. Khi f (1) = P (1, 0) + 2 = 3 v (5) tr thnh


f (m + n) = f () + f (). l phng trnh dy chuyn i php cng
(
f (m + n) = f () + f (),
(3)
f (1) = 3.
Phng trnh (3) c nghim duy nht f () = 3n. Vy nn
P (n, 1 ) = 3n 2.

(4)

8.4. Phng trnh trong hm s vi cp bin t do

193

Bng phng php quy np ta s thu c



a
P (a, b) = (a + b)2 3
2 = (a + b)(a 2b), a, b Q.
a+b
Tm li P (, ) = ( + n)2 (m 2n).
Bi ton 8.39. Cho a thc Chebyshev Tn (x) = cos( arccos x). Chng minh
rng vi m, n Q; m v x R th Tn (x) l nghim ca phng trnh dy
sau
Tn+m (x) + Tnm (x) = 2Tn (x)Tm(x).
Gii. S ng nh ngha Tn (x) v phng php quy np hoc s dng cc cng
thc
cos(n + m)x + cos(n m)x = 2 cos nx cos mx
v
cosh(n + m)x + cosh(n m)x = 2 cosh(nx) cosh(mx),
ta c ngay iu phi chng minh.
Bi ton 8.40. Tm hm f Q Q tha mn cc iu kin
Q : < f () < Q,
f ( + n) + f (m ) = 2f ()f () m, Q.

(1)

Gii. Cho m = n = 0 ta c f (0) {0, 1}. Gi s f (0) = 0. Cho = 0 trong


(1) ta c 2f () = 2f ()f (0) = 0 v f 0.
Gi s f (0) = 1. Cho m = 0 trong (1) ta thu c f () = f () vi mi
Q. Vy ch cn xt Q. Cho = 1 trong (1), ta c
f ( + 1) = 2f ()f (1) f (m 1)
v thu c cng thc truy hi theo f (1). Nu |f (1)| 2 th t gi thit ta c
f (2n) = 2[f ()]2 1
tng v khng gii ni, tri vi gi thit. Vy f (1) {1, 0, 1}.
Vi f (1) = 1 th f () = (1)n (quy np).
Vi f (1) = 1 th f () 1.
Vi f (1) = 0 ta c dy tun hon (quy np)
f (4m) = 1, f (4 + 1) = 0, f (4 + 2) = 1, f (4 + 3) = 0.
Suy ra f (2) = 4. Chng minh bng quy np ta c f () = 2 . Tht vy, do
f (k) + f (k) = (1/2)[f (2k) + f (0)] nn c ngay f (2k) = 4k2 . Cng vy, do f (k +
1) + f (k 1) = (1/2)[f (2k) + f (2)] nn ta c
1
f (k + 1) = f (2k) + 2 f (k 1) = (k + 1)2.
2

194

8.4. Phng trnh trong hm s vi cp bin t do

Bi ton 8.41. K hiu


u =

sinn xdx, Q.

Xc nh hm s f : Q R theo cng thc


f () = ( + 1)un g( + 1), Q.
Gii. S dng cng thc tch phn tng phn, ta thu c
1

u = cos x sin


x 02 +

( 1) sin2 x cos2 xdx

( 1) sin2 x(1 sin2 x)dx

= ( 1)(u1 un ).
T y suy ra
g( + 2) =

+1
un , Q.
+2

T (1) ta nhn c
f ( + 1) = ( + 2)g( + 1)g( + 2)
= ( + 2)g( + 1)

+1
u
+2

= ( + 1)g( + 1)u = f ().


Vy nn
f () = f (0) =

.
2

Bi ton 8.42. K hiu


u =

cos x cos nxdx, Q.


0

Xc nh hm s f : Q R theo cng thc


f () = 2 u , Q.

(1)

8.4. Phng trnh trong hm s vi cp bin t do

195

Gii. t cosn x = u v cos nxdx = dv th theo cng thc tch phn tng phn,
ta thu c
Z

1
u = cosn x sin x 0 +
cos1 x sin x sin nxdx

0
Z
1
=
cos1 x[cos( 1)x + cos( + 1)x]dx
2 0
Z
1
1
1
= u1 u +
cos1 x sin x + sin nxdx
2
2
2 0
1
1
1
= u1 u + u .
2
2
2
Vy nn
u =

1
1
1
1
u1 = u2 = = 1 u1 = 1 .
2
4
2
2
2

Bi ton 8.43. 3. Xc nh hm s {u } c tnh theo cng thc


u =

tan2 xdx

Gii. Ta vit un di dng sau


u =

tan2 xdx

tan22 x[(tan2 x + 1) 1]dx

tan22 xd tan x u1

tan21 x 4
u1
2 1 0
1
=
u1 .
2 1

Do vy
1
,
2 1
1
=
,
2 3

u + u1 =
u1 + u2

196

8.4. Phng trnh trong hm s vi cp bin t do

u1 + u 0 =
Suy ra
u = (1)

1
.
21

X
(1)k i
+
.
4
2k 1
k=1

Bi ton 8.44. Xc nh hm s f : Q R c tnh theo cng thc


f () =

Z1

x 1 xdx, Q.

Gii. t x = u,

1 xdx = dv th
f () =

Z1

x 1 xdx

3  1
2
2
= xn (1 x) 2 0 +
3
3

Z1

x1 1 x(1 x)dx

2
=
3

Z1

(x1 xn ) 1 xdx

2
=
3

Z1

(x

2
1 xdx
3

Z1

x 1 xdx

2
2
x1
f ().
3
3
Vy nn
f () =
V f (0) =

2
x1 .
2 3

2
nn ta c ngay
3
f () =

Z1

(2n)!!
x 1 xdx = 2
, Q.
(2 + 3)!!

Bi ton 8.45. Xc nh hm f : Q Q tho mn cc iu kin


f (0) = 1, f (f () = f (f ( + 2) + 2) = Q.

8.4. Phng trnh trong hm s vi cp bin t do

197

Gii. Nhn xt rng f l nh x 1-1


f () = f () f (f ()) = f (f ()) m = n.
Vy nn
f () = f ( + 2) + 2 Q.
Suy ra
f ( + 2) = f () 2, f (0) = 1, f (1) = f (f (0)) = 0.
Vy nn
f (2) = f (0) 2 = 1,
f (3) = f (1) 2 = 2,
f () = ( 1).
Tng t
f (1) = f (1) + 2 = 2,
f (2) = f (0) + 2 = 3,
f (3) = f (1) + 2 = 5,
f () = ( 1).
Bi ton 8.46. Cho gc vi 0 < < . Xc nh cp s a, b sao cho dy hm
{Pn (x)} c tnh theo cng thc
Pn (x) = x sin x sin() + sin( 1)
lun lun chia ht cho f (x) = x2 + ax + b.
Gii. Vi = 3 th
P3 (x) = x3 sin x sin(3) + sin 2 = sin (x + 2 cos )(x2 2x cos + 1).
.
T suy ra vi f (x) = x2 + 2x cos + 1 th P3(x)..f (x). Vi 3 th
P+1 (x) = xPn (x) + (x2 2x cos + 1) sin .
Suy ra f (x) = x2 + 2x cos + 1.

198

8.5. S dng gii hn gii phng trnh hm

Bi tp
Bi 1. Xc nh hm s f : Q R nu bit: f (1) = a, f ( + n) = f () + f ();
Bi 2. Xc nh hm s f : Q R nu bit: f (1) = a, f (m ) = f () + f ()
(, , m Q).
 
Bi 3. Xc nh hm s f : Q R nu bit f = f () + f () (, , m
Q).
 
Bi 4. Xc nh hm s f : Q R tho mn iu kin f = f () f ()
(, , m
Q).
Bi 5. Xc nh hm s f : Q R tho mn iu kin f ( + n) = f ()f ()
(m, Q).
Bi 6. Xc nh hm s f : Q R nu bit f (+n)+f (m) = 12 (f (2m)+f (2n)
, (, , m Q).
Bi 7. Xc nh hm s f : Q R+ tho mn iu kin f ( + ) = ff ()
()
(m, Q).
 p

Bi 8. Xc nh hm s f : Q R+ tho mn iu kin f +
= f ()f ()
2
(, , +
2 Q).
Bi 9 . Xc nh hm s f : Q R+ tho mn iu kin x + =
2

(, , +
2 Q).

Bi 10 . Xc nh hm s f : Q R+ tho mn iu kin f
q
f ()2 +f ()2
(, , +
2
2 Q).

2f ()f ()
f ()+f ()

+
2

Bi 11. Xc nh cc hm f : Q Q tho mn cc iu kin f (1) = 2 v


f (xy) = f (x)f (y) f (x + y) + 1, x, y Q.

8.5

S dng gii hn gii phng trnh hm

Mt trong nhng tnh cht cn thit s dng gii hn l tnh lin tc ca


hm s. Khi s dng gii hn gii phng trnh hm ngi ta thng lm nh
sau.
1. Xy dng mt ng thc ng vi mi gi tr ca n sau ly gii hn hai
v nh s dng tnh cht lin tc ca hm s.
2. Tnh lin tc khng c tc dng i vi phng trnh hm trong tp hu t
Q. Tuy nhin nu bit chc chn l hm lin tuc, ta c th thit lp cng
thc cho hm trong Q v suy ra cng thc phi tm tng t trong tp R.
Bi ton 8.47. Tm tt c cc hm f : R R lin tc, tho mn iu kin
f (x + y) = f (x) + f (y), x, y R.

8.5. S dng gii hn gii phng trnh hm

199

Gii. Cho x = y = 0, suy ra f (0) = 2f (0, suy ra f (0) = 0. Cho x = y = 1, th


c f (2) = 2f (1). Cho x = 2, y = 1 th c f (3) = f (2) + f (1) = 3f (1). Quy
np ta c f (n) = nf (1), vi mi n N.
K hiu f (1) = a, suy ra f (n) = na, vi mi n N. Cho x = n, y = n
ta c 0 = f (0) = f (n) + f (n). Suy ra f (n) = f (n), f (n) = a(n), v
f (n) = an vi mi n Z.
t x = y, ta c f (2x) = 2f (x), f (3x) 6 f (2x + x) = 2f (x) + f (x), f (3x) =
3f (x). Suy ra
f (mx) = mf (x), m N, x R.
T


n
n
an = f (n) = f m
= mf
m
m

suy ra f (n/m) = an/m. Suy ra f (x) = ax, vi mi x Q. T suy ra vi mi


x R lun tn ti {xn }
1 , xn Q sao cho lim xn = x. Ta c
n

f (xn ) = axn .
Ly gii hn ta c
lim f (xn ) = lim (axn ).

T f (x) = ax, vi mi x R.
Bi ton 8.48. Tm tt c cc hm lin tc f : R R tho mn iu kin
f (x + y) + f (x y) = 2f (x) + 2f (y).
Gii. Cho x = y = 0, suy ra f (0) = 0. Cho x = 0 suy ra f (y) + f (y) = 2f (y).
Do f (y) = f (y), tc l f (x) l hm s chn trn R.
K hiu f (1) = a. t x = y = 1, suy ra f (2) + f (0) = 4f (1) = 4a. T
f (2) = 4a. t x = 2, y = 1 suy ra f (3) + f (1) = 2f (2) + 2f (1), f (3) =
2f (2) + f (1) = 9a.
Ta chng minh quy np f (n) = an2 . Ta gi s f (n) = an2 , phi chng minh
f (n + 1) = a(n + 1)2. Cho x = n, y = 1, ta c f (n + 1) + f (n 1) = 2f (n) + 2f (1).
Suy ra f (n + 1) = 2an2 a(n 1)2 + 2a = a(2n2 (n 1)2 + 2). Tip tc khai
trin cho ta
f (n + 1) = a(n + 1)2.
Do f l hm s chn nn f (n) = f (n) = an2 = a(n)2 . Ta c f (n) = an2 vi
mi n Z.
By gi ta chng minh cng thc f (nx) = n2 f (x), vi mi n N. Vi x = y
ta c f (2x) + f (0) = 2f (x) + 2f (x) = 22 f (x). Suy ra f (2x) = 22f (x).

200

8.5. S dng gii hn gii phng trnh hm

Gi s f (nx) = n2 f (x), ta phi chng minh f ((n + 1)x) = (n + 1)2 f (x). Tht
vy f ((n + 1)x) + f ((n 1)x) = 2f (nx) + 2f (x). Suy ra
f ((n + 1)x) = (n 1)2f (x) + 2n2 f (x) + 2f (x)
T y tip tc khai trin cho ta f ((n + 1)x) = (n + 1)2f (x). Khi an2 =
n
f (n) = f (m. m
) = m2f (n/m). Suy ra f (n/m) = a(n/m)2, vi mi m, n N. Suy
ra ta c f (x) = ax2 , vi mi x Q.
Vy vi mi x R lun tn ti {xn }
n , xn Q sao cho lim xn = x. T
n

f (xn ) = ax2n , hay l

lim f (xn ) = lim ax2n .

T f (x) = ax , vi mi x R.
Gii (2). Ta c f (0) = 0, cho x = y ta c f (2x) = 4f (x). Suy ra
f (2x)
f (x)
= 2 , x R {0}.
2
(2x)
x
Suy ra
g(2x) = g(x), g(x) =
Suy ra
g(x) = g

x
2

=g

x
22

f (x)
.
x2

= = g

(8.1)

x
,
2n

Do f lin tc nn g lin tc trn R{0}. Suy ra g(x) = lim g(x/2n) = g(0) = a.


n
Suy ra
g(x) = a, x R {0}.
Vy f (x) = ax2 , vi mi x R {0}. Do f (0) = 0 nn f (x) = ax2 , vi mi
x R.
Bi ton 8.49. Gi s f : R R v tha mn cc iu kin
1. f (1) = 1,
2. f (x + y) = f (x) + f (y), x, y R,
3. f (x).f (1/x) = 1, x 6= 0.
Gii. Ta chng t rng hm cn tm l f (x) = x, vi mi x R. Theo bi ton
(8.47), t iu kin th nht v th hai ta suy ra f (x) = x, vi mi x Q.
By gi ta chng t f (x l hm lin tc. Cn chng minh lim f (x+h) = f (x)
h

hay l lim (f (x) + f (h)) = f (x). Tc l cn chng t lim f (h) = 0.


h0

h0

8.5. S dng gii hn gii phng trnh hm

201

Nu a, b > 0 th |a + b| = |a| + |b|, t iu kin th ba suy ra rng vi x 6= 0 ta


c f (x) v f (1/x) cng du. Ch rng theo bt ng thc gia trung bnh cng
v trung bnh nhn, ta c

 

 



f x + 1 = f (x) + f 1 = |f (x)| + f 1





x
x
x
r
1
> 2 f (x).f
= 2.
x
Suy ra
f (y) > 2 vi mi y > 2.

(8.2)

Vy nu |y| 6 12 hay |1/y| > 2 th theo (8.2) ta c f (1/y) > 2.


T iu kin th ba ta c
1 = |f (y).f (1/y)| > |f (y)|.2.
Suy ra
|f (y)| 6

1
.
2

(8.3)

Nu |y| 6 14 , suy ra |2y| 6 12 , suy ra |f (2y)| 6 12 . Do


|f (y)| 6

1
.
4

(8.4)

Bng quy np, ta chng minh c


1
1
, vi mi |y| 6 n1 .
n
2
2

|f (y)| 6
Tht vy, gi s |f (y)| 6
gi thit quy np ta c

1
2n1

vi |y| 6 1/2n1 . Khi nu |y| 6 1/2n th theo


1
2n1

> |2y|.

Suy ra 1/2n1 > |f (2y)|, hay l




1


2n1 > 2f (y).
Tc l |f (y)| 6 1/2n vi |y| 6 1/2n . Vy lim f (h) = 0, suy ra f (x) l hm lin
h0

tc trn R, m f (x) = x, vi mi x Q. Vy
f (x) = x, vi mi x R.

202

8.5. S dng gii hn gii phng trnh hm

Bi ton 8.50. Cho f : R R lin tc tha mn iu kin f (1) = 1, f (x + y) +


f (xy) = f (x) + f (y) + f (x).f (y).
(a) Chng minh rng f (x) cng tnh: f (x + y) = f (x) + f (y).
(b) Hy tm tt c cc hm f tha mn iu kin
f (x + y) + f (xy) = f (x) + f (y) + f (x).f (y).
Gii. Cu (a). Cho y = 1, th ta c
f (x + 1) = f (x) + 1.

(8.5)

Thay x bi x + 1, ta c
f (x + 1 + y) + f ((x + 1)y) = f (x + 1) + f (y) + f (x + 1)f (y).
V f (x + 1) = f (x) + 1 nn ta c
f (x + y) + 1 + f (xy + y) = f (x) + 1 + f (y) + f (x + 1).f (y)
= f (x) + f (y) + f (x)f (y) + 1 + f (y)
= f (x + y) + f (xy) + f (y) + 1.
T suy ra
f (xy + y) = f (xy) + f (y)
Vy vi u, v bt k, tn ti x sao cho u = vx, suy ra x =

(8.6)
u
v

6= 0. Suy ra

f (u + v) = f (vx + v) = f (vx) + f (v) = f (u) + f (v).

(8.7)

Nu v = 0 th f (u) = f (u) + f (0), suy ra f (0) = 0. Thay x = y = 0 vo


f (x + y) + f (xy) = f (x) + f (y) + f (x).f (y),
ta c
f (0) + f (0) = f (0) + f (0) + f (0).f (0).
Suy ra
f (0) = 0

(8.8)

T (8.7) v (8.8) suy ra f cng tnh.


Cu (b). T cu (a) suy ra f (x + y) = f (x) + f (y), v f (xy) = f (x).f (y).
Tc l hm f (x) va cng tnh va nhn tnh. Suy ra f (x) = x, x = m
n , thnh

203

8.5. S dng gii hn gii phng trnh hm

th x Q. Suy ra vi mi x R, lun tn ti xn Q : xn x khi n . Do


f x) lin tc trn R nn limn f (xn ) = f (x), hay l x = f (x).
Gi s hm s khng gim. Cho x = y, ta c f (x2) = f 2 (x) > 0, suy ra
f (u) > 0 vi mi u > 0. Nu x > y th f (x) = f (y + x y) = f (y) + f (x y) >
f (y), v f (x y) > 0 vi x y > 0.
Vi mi x R Q, tn ti rn : rn Q sao cho
rn > x :

lim rn = x.

(8.9)

lim sn = x.

(8.10)

Tn ti sn : sn Q sao cho
sn < x :

Suy ra sn < x < rn . Do f l hm s khng gim nn f (sn ) 6 f (x) 6 f (rn ), hay


l
sn 6 f (x) 6 rn .
(8.11)
Ly gii hn cho ta
lim sn 6 f (x) 6 lim rn .

T (8.9), (8.10) v (8.11), theo nguyn l kp ta c x 6 f (x) 6 x. Suy ra


f (x) = x. Cho x = y = 1, t ta c f (2) = 2. Thay x = x 1 v y = y 1 ta
c
f (x + y) + f ((x y)(y + 1)) = f (x 1) + f (y + 1) + f (x 1).f (y + 1). (8.12)
Cho y = 1, ta c
f (x + 1) + f (x) = f (x) + f (1) + f (x).f (1).
ng thc ny tng ng vi
f (x + 1) = f (x) + 1.

(8.13)

Cho x = 2, t (8.12) cho ta


f (2 + y) + f (y + 1) = f (1) + f (y + 1) + f (1).f (y + 1).
V (8.13) nn t y ta c
f (2 + y) = f (1)(1 + f (y + 1)) = 1 + f (y + 1),
hay
f (2 + y) = f (2) + f (y).

(8.14)

204

8.5. S dng gii hn gii phng trnh hm

Cho x = 2, t ng thc
f (x + y) + f (xy) = f (x) + f (y) + f (x).f (y),
ta c
f (2 + y) + f (2y) = f (2) + f (y) + f (2).f (y).
Phng trnh ny tng ng vi
f (2y) = 2f (y),

(8.15)

f (2y)
f (y)
=
.
2y
y

(8.16)

hay l

t g(x) = f (x)/x, x 6= 0, ta c (8.16) tng ng vi


g(x) = g(2x).
Suy ra

x

x
.
(8.17)
2
4
2n
Suy ra tn ti (xn )
0 vi mi x R sao cho xn x, g(x) l hm s lin tc trn
R {0}. Thnh ra, lim g(xn) = g(x). Tc l
g(x) = g

=g

x

= = g

lim g

1
= g(x).
2n
f (1)

V (8.17) nn t y ta c g(1) = g(x), hay 1 = g(x), f (1) = g(x) = f (x)/x.


Cui cng ta c f (x) = x.
Suy ra f cng tnh.
Ch rng trong (b) nu cn xt ring x Q th ta c
f (n) = f (1
| + 1 +{z + 1}) = nf (1) = n.
n s 1

Ta c
f (n.

m
) = f (n).f (m/n)
n

tng ng vi mi
f (m) = n.f (n/m)
f (m) = nf (m/n)
Suy ra

m

f (m)
mf (1)
m
=
=
= .
n
n
n
n
Vy f (m/n) = m/n, hay f (x) = x, vi mi x Q.
f

8.5. S dng gii hn gii phng trnh hm

205

Bi ton 8.51. Tm hm f (x) xc nh R R (0 6 x < +) tha mn cc


iu kin
1. f (x.f (y)).f (y) = f (x + y) x, y > 0,
2. f (2) = 0
3. f (x) 6= 0 x [0, 2).
Gii. Cho y = 2 th t iu kin th nht ta c f (x.f (2)).f (2) = f (x + 2). T
iu kin th hai suy ra f (x + 2) = 0 vi mi x > 0. V iu kin th ba nn
f (x) 6= 0 vi mi x [0, 2), suy ra t = x + 2 > 2, suy ra f (t) = 0 vi mi t > 2.
Suy ra
f (x) = 0, x > 2.
(8.18)
Vy
f (x) =

0
6 0
=

nu x [2, +)
nu x [0, 2)

(8.19)

Ta ch cn tm hm f (x) vi 0 6 x 6 2 th f ((2 x).f (x)).f (x) = f (2 x + x) =


f (2) = 0.
Suy ra f ((2 x).f (x)) = 0. Kt hp vi (8.18) ta c (2 x)f (x) > 2. Suy ra
1/f (x) 6= 0 v f (x) > 2/(2 x). Do
1
2x
6
.
f (x)
2

(8.20)

Mt khc, f ((y x).f (x)) 6= 0, nn (y x).f (x) < 2 theo (8.19). Ta c nh x


v cho y 2, do tnh lin tc ca f nn (y x)f (x) (2 x)f (x) 6 2. Vi
2 x > 0 th f (x) 6 2/(2 x). Suy ra
1
2x
>
.
f (x)
x
T (8.20) v (8.21) suy ra
2x
1
2x
6
6
.
2
f (x)
x
Do f (x) = 2/(2 x). Tm li
f (x) =

2
2x

khi 0 6 x < 2
khi x > 2

(8.21)

8.5. S dng gii hn gii phng trnh hm

206

Cch 2. Khi 0 6 x < 2, ta c 2 x > 0,


f ((2 x).f (x)).f (x) = f (2 x + x) = f (2) = 0.
Do f (x) 6= 0 nn f ((2 x)f (x)) = 0, suy ra (2 x)f (x) > 2. Thnh th,
f (x) >

2
2x

(8.22)

Mt khc, f ((y x)f (x)) 6= 0 nn (y x)f (x) < 2. Ta c nh x v cho y 2,


do tnh lin tc ca f nn (y x)f (x) (2 x)f (x) 6 2. Suy ra
f (x) 6
T (8.22) v (8.23) suy ra f (x) =
f (x) =

2
2x .
2
2x

2
.
2x

(8.23)

Tm li
khi 0 6 x < 2
khi x > 2

Bi ton 8.52. Tm hm f (x) xc nh v lin tc vi mi x > 0 v tho mn


iu kin
1. f (uv) = f (v) + f (u), u, v > 0
2. lim f (x) = 0,
x1

3. f (x) 6= 0, x > 0.
Gii. Gi s f (x) 6 0, vi mi x > 0.
f (x) = f (1.x) = f (1) + f (x),
suy ra f (1) = 0. Ly x > 0, xn x0 khi n . Suy ra xn /x0 1 khi n .
Suy ra
 
xn
lim f
= 0.
x0
Vy f (xn ) = f (x0, xxn0 ) = f (x0) + f ( xxn0 ) f (x0) + 0. Tc l
lim f (xn ) = f (x0 ).

xn x0

f (uv) = f (u) + f (v), u, v > 0.

(8.24)

8.5. S dng gii hn gii phng trnh hm

207

Suy ra f (xn ) = nf (x), vi x > 0, n N . Do f (1) = 0 nn f (xn ) = nf (x)


ng khi n = 0. Hn na, vi mi n N th 0 = f (1) = f (xn , xn ) = f (xn ) +
f (x1 ) = nf (x) + f (xn ). Suy ra f (xn ) = nf (x), v
 m
 1 m 
1
m
f x n = f xn
= mf (x1/n) = m f (x) = f (x).
n
n
Suy ra f (xm/n ) =

m
n f (x),

m, n Z. Do

f (xr ) = rf (x), x > 0 r Q.


c bit f (2r ) = r.f (2) = r.A, r Q, A = f (2). Nu x > 0 th x = 2log2 x suy
ra vi mi x > 0 th tn ti mt dy s hu t {rn}
0 sao cho
lim rn = log2 x.

Suy ra
lim 2rn = 2log2 x = x

dn n lim f (2rn ) = f (x), m f (2rn ) = A.rn A. log2 x khi n .


n
Suy ra
lim f (2rn ) = A. log2 x = f (x).
n

Do f (x) 6 0 vi x > 0 nn vi mi x > 0 ta c A 6= 0, t A = loga 2, a > 0 v


a 6= 1, f (x) A log2 x = loga 2. log2 x = loga x. Vy
f (x) = loga x, x > 0, 0 < a 6= 1.
Bi ton 8.53. Tm f (x) xc nh v lin tc vi mi x > 0 v tho mn iu
kin
1. f (uv) = f (u).f (v), u, v > 0
2. lim f (x) = 1.
x1

Gii. Ta c vi mi x > 0

f (x) = f ( x. x) = f 2 ( x) > 0.
Suy ra f (x) > x, x > 0.
Nu tn ti x0 > 0 f (x0) = 0 th vi mi x > 0 ta c f (x) = f (x0 . xx0 ) =
 
f (x0 ).f xx0 = 0. Theo gi thit ta cng c lim f (x) = 1. Vy f (x) > 0, vi mi
x1
x > 0.

208

8.5. S dng gii hn gii phng trnh hm

Xt hm s g(x) = lim f (x), vi mi x > 0 tho mn iu kin


g(u, v) = ln f (uv) = ln(f (u).f (v)) =
= ln f (u) + ln f (v)
= g(u) + g(v)
v lim g(x) = 0.
x1

Vy g(x) tha mn iu kin bi ton 8.52 trn. Suy ra


g(x) = loga x, 0 < a 6= 1, x > 0.
Suy ra ln f (x) = log x = loge. ln x. Do
f (x) = x
vi = loga e, (0 < a 6= 1)
Bi ton 8.54. Cho f : (1, 1) R lin tc v


2x
f (x) = f
, x [1, 1].
1 + x2
Chng minh rng f (x) l hm hng.
Gii. Ta c nh x, xt dy s (xn )
1 xc nh bi
x0 = x > 0, xn+1 =

2x
.
1 + x2

(8.25)

Dy (xn ) l dy s tng. Suy ra xn+1 > xn , hay 2xn /(1 + x2n ) > xn . T y ta c
1 6 xn 6 1. Theo bt ng thc gia trung bnh cng v trung bnh nhn, ta c
xn+1 =

2xn
6 1.
1 + x2n

Do dy s (xn ) tng v b chn di bi 1 nn tn ti gii hn limn xn = l > 0.


Ta c (8.25) tng ng vi
2l
l=
,
1 + l2
t ta c l = 1.
Dy s
f (xn+1 ) = f

2xn
1 + x2n

= f (xn ).

Ly f (x) = f (x0 ) = = f (xn ). Do f (x) lin tc trn [1, 1] nn lim f (xn )


n

= f (1), hay f (x) = f (1) = c, vi c l hng s.


(trng hp x0 = x < 0 xt tng t)

209

8.5. S dng gii hn gii phng trnh hm

Bi ton 8.55. Tm tt c cc hm f (x) xc nh v lin tc trn R {0} v


tha mn iu kin sau
(f (x2 ) x2 )(f (x) x) =

1
, x 6= 0.
x3

(8.26)

Gii. Phng trnh (8.26) tng ng vi


(x2f (x2 ) x4 )(xf (x) x2 ) = 1.
t xf (x) x2 = g(x), ta thu c
g(x2)g(x) = 1.

(8.27)

Suy ra g(x) 6= 0, vi mi x R. Ta c g(0) = 1 v g(1) = 1. Thay x bi x


vo trong (8.27) ta thu c
g(x2)g(x) = 1 = g(x2)g(x).
Suy ra g(x) = g(x) trn tp i xng qua gc to R. Suy ra g(x) l hm s
chn trn R, nn ta ch cn xt g(x) trn tp x > 0 l .
Xt 0 6 x 6 1, ta c
g(x) =

1
=
g(x2)

1
1
g(x4 )

= g(x4).

Suy ra g(x) = g(x4).


Li c
g(x4) =

1
=
g((x4)2 )

1
1
g((x4 )2 )

= g((x4)4 ) = g(x4 ).

Suy ra g(x4) = g(x4 ). Vy ta thu c


n

g(x) = g(x4) = g((x4)4) = = g(( (x4 )4)4 )4) = g(x4 ), n +.


Qua gii hn ta thu c
n

g(x) = lim g(x(1/4) ),


n+

do lim x
0 x 1

4n

= 0. Suy ra
n

g(x) = lim g(x4 ) = g(0),


n+

m g(0) = 1. Suy ra g(x) = 1 = c.

210

8.5. S dng gii hn gii phng trnh hm

Xt x > 1, ta c
1

g(x) =

1
2

g(x )

1 n

= g(x 4 ) = = g(x( 4 ) ).

1
1

g(x 4 )
1 n

Qua gii hn, ta thu c g(x) = lim x( 4 ) = g(1) v vi x > 1 th limn+ =


n+

1, suy ra g(x) = g(1) = 1 = c.


Vy c = xf (x) x2 , hay f (x) = c/x + x, c l mt hng s.
Bi ton 8.56. Cho f : (1, 1) R lin tc tho mn iu kin


2x
, x (1, 1).
f (x) = f
1 + x2

(8.28)

Chng minh rng f (x) l hm hng.


Gii. Xt 0 < x < 1. Ta c nh x, xt dy s (xn )+
nh sau
1
p
1 1 x2n
x0 = x, xn+1 =
.
xn

(8.29)

Dy ny c suy ra t vic xt dy s
xn =

2xn + 1
.
1 + x2n+1

Ta chng minh rng (xn )


1 xc nh vi mi n v
lim xn = 0.

(8.30)

T (8.28) suy ra f (x) = f (x0 ) = f (x1 ) = = f (xn ). Do f (x) lin tc trn


(1, 1) nn f (x) = lim f (xn ) = f (0).
n

Ta chng minh dy s (xn )


1 b chn.

D thy (xn )1 lun dng vi mi n N . Ta chng minh xn 6 1, vi mi


n N .
Nu n = 0 th x0 = x < 1, ng theo gi thit.
Gi s xk < 1, ta c
p
1 1 x2n
xk+1 =
6 1.
xk
q
Bt ng thc ny tng ng vi 1xk 6 1 x2k . T y ta c xk (xk 1) 6 0,
iu ny lun ng vi xk < 1.

8.5. S dng gii hn gii phng trnh hm

211

Suy ra (xn )
1 l dy s gim.

By gi ta chng minh dy s (xn )


1 b chn bi s 0. Tht vt, v (xn )1 l
dy s gim nn tn ti lim xn = c (ta i chng minh c = 0).
n
Suy ra
2xn + 1
2xn
lim xn = lim
= lim
.
2
n
n 1 + x
n 1 + x2
n
n+1
T c = c/(1 + c2 ), suy ra c = 0 hoc c = 1. Do dy s gim nn c = 0, suy

ra limn xn = 0. Vy dy s (xn )
1 gim v b chn bi 0 vy dy s (xn )1 b
chn.
Ta c


2xn+1
f (xn ) = f
= f (xn+1 ).
1 + x2n+1
Suy ra f (x) = f (xn+1 ) = f (xn ) = = f (x1 ) = f (0). Do f (x) lin tc trn
(1, 1) nn lim f (xn ) = f (0), hay l f (x) = f (0) = c vi c l mt hng s.
n

Trng hp 1 < x < 0. Ta chng minh dy s (xn )


1 n iu tng v b
chn bi s 0.
Nhn xt. Bi ton 8.54 v 8.56 khc nhau c bn iu kin nn bi 8.54
2xn
2xn +1
xt dy s xn+1 = 1+x
2 , bi ton 8.56 xt dy s xn = 1+x2 sao cho dy s
n
n
2
1 1xn
khng th bng 1 c, suy ra xn =
.
xn
Bi ton 8.57. Cho f : R R lin tc, 0 < c < 14 . Gi s f tho mn iu
kin f (x) = f (x2 + c), x R. Chng minh rng f (x) l hm hng.
Gii. Nhn xt rng f l hm s chn trn R. Suy ra, ta ch cn xt x > 0. Xt
x2 + c = x, tc vi 0 < c < 14 th tn ti hai nghim phn bit , ca phng
trnh
x2 x + c = 0.
(8.31)
Trng hp 1. Xt x [0, ]. C nh x0, xt dy s
x0 = x, xn+1 = x2n + c.
Ta c
f (xn+1 ) = f (x2n + c) = f (xn ), 0, 1, 2, ..., .
Suy ra
f (x) = f (x1 ) = f (x2 ) = = f (xn ),
v f (x) lin tc trn R. Suy ra lim f (xn ) = f (x).
n

(8.32)

212

8.5. S dng gii hn gii phng trnh hm

Ta chng minh (xn )


1 : lim xn = . Suy ra lim f (xn ) = f () v f (x) lin
n

tc trn R, hay l f (x) = f (), vi mi x [0, ], hay f (x) = c, vi c l mt


hng s.
a) Chng minh lim = vi (xn )
1 xc nh bi (8.32).
n

2
0
Ta c (xn )
1 l dy s tng. Xt g(x) = x + c, g (x) = 2x > 0 vi x [0, ].
Suy ra g(x) ng bin trn [0, ]. Do , g(x1) > g(x0), tng t vi x2 > x1 ta
c g(x2) > g(x1). Vy (xn )
1 l dy s tng.
b) Chng minh (xn )
b
chn bi (bng phng php quy np). Vi x0 =
1
x < . Gi s (8.32) ng vi n = k: xk < . Suy ra xk+1 = x2k + c < 2 + c =
v l nghim ca (8.31). Suy ra (8.32) ng vi n = k + 1.
T a) v b) suy ra lim xn = .
n

Trng hp 2. Xt x [, ], xt dy s
x0 = x, xn+1 = x2n + c.

(8.33)

Chng minh tng t nh trng hp 1, (xn )


1 l dy s gim., xn > , suy ra
lim xn = .
n
Suy ra
f (x) = f (), x [, ].
Trng hp 3. x [, +), xt dy s xc nh bi

x = x0 , xn+1 = xn c vi xn = x2n+1 + c.

(8.34)

Chng minh rng lim xn = .


n
Xt g(x) = x c. Tnh o hm cho ta
1
g 0(x) =
> 0, vi x [, +).
2 xc
Suy ra g(x) ng bin trn [, +).

Ta c x1 = x0 c < x0 , hay x20 x0 + c > 0 lun ng do x0 [, +).


Nu x1 < x0 th dy s (xn )
1 gim.
Ta chng minh (xn )
b
chn di bi
bng phng php quy np. Nu
1

x0 = x > , gi s xk > , xk+1 = xk + c > c = . iu ny lun ng


v l mt nghim ca 2 + c = 0. Suy ra tn ti lim xn = l (l > ). T
n

lim xn = .

T dy s (8.32) suy ra f (xn ) = f (x2n+1 + c) = f (xn+1 ). Ly


f (x0) = f (x1) = = f (xn ).

(8.35)

213

8.5. S dng gii hn gii phng trnh hm

f (x) lin tc trn [, +). Do


lim f (xn ) = f (),

hay l f (x) = f () = c, vi c l hng s.


Php chng minh hon tt.
Bi ton 8.58. Chng minh rng khng tn ti hm s f : (0, ) (0, +)
tho mn iu kin
1. f (x) l hm s tng: x > y > 0 th f (x) > f (y).


1
2. f x + f (x)
> 2f (x).
Gii. Gi s f (x) tn ti, ta c nh x0 > 0. Xt dy s xc nh bi
x0 = x

(8.36)

xn+1 = xn +

1
.
f (xn )

(8.37)

iu kin th nht trong bi cho ta


f (xn+1 ) > 2f (xn ) > 2(2f (xn1)) = 22 f (xn1 ) > 22 .2f (xn2 )
> > 2n f (x1 ) > 2n+1 f (x0).
Vy ta c
f (xk ) > 2k f (x0 ) vi mi k = 0, 1, 2, ...
1
,
f (x0 )
1
x 2 = x1 +
,
f (x1 )
1
x 3 = x2 +
,
f (x2 )
...
1
xn = xn1 +
,
f (xn1 )
x 1 = x0 +

Suy ra
xn = x0 +

n
X
k=0

1
.
f (xk )

(8.38)

214

8.5. S dng gii hn gii phng trnh hm

T (8.38) suy ra
1
1
6 k
.
f (xk )
2 f (x0 )
Suy ra
x n 6 x0 +

n
X
k=0

1
1 X 1
2
+
= x0 +
=
x
.
.
0
k
k
2 f (x0)
f (x0)
2
f (x0 )
k=0

n
X
1
1
=
2k
1
k=0

nn

xn 6 a

1
2


2
= x0 +
, n.
f (x0 )

Do f l hm s tng nn 2n f (x0) 6 f (x0 ) < f (a), vi mi a. T y v (8.38)


ta suy ra f (a) > 2n f (x0) vi mi n. Suy ra f (a) > , mu thun vi gi thit
rng f (x) tn ti. iu mu thun ny cho ta iu phi chng minh.
Bi ton 8.59. Tm tt c cc hm f : R R lin tc ti x = 0 v tho mn
iu kin nf (nx) = f (x) + nx, trong n > 1 l s t nhin c nh no .
Gii. Cho n = 0, t thay gi tr ny vo biu thc cho, ta c nf (0) =
f (0) + 0, hay (n 1)f (0) = 0. Suy ra f (0) = 0, v n > 1. Cng t biu thc
cho, thay x bi x/n th
 x
x x
nf n = f
+n ,
n
n
n
hay
x
nf (x) = f
+x.
n
Suy ra
1 x x
f (x) = .f
+ .
(8.39)
n
n
n
Trong (8.39) thay x bi x/n, ta c
f
Suy ra
1
f (x) =
n

x
n

1 x x
+ 2
f
n n2
n

x
1x
+ 2.
2
n n
n


x
x
x
1 x
+ 3+ .
= 2f
2
n
n
n
n
n

(8.40)

215

8.5. S dng gii hn gii phng trnh hm

Trong (8.39) li thay x bi x/n2 th ta c


f

x 1 x x
= f 3 + 3.
n2
n n
n

(8.41)

T (8.40) suy ra
1 x
x
x
+ f racxn5 + 3 + .
f
n3
n3
n
n

f (x) =

T , ta c th chng minh quy np theo k rng


1 x
x
x
x
f (x) = k f
+ 2k1 + 2k3 + + .
k
n
n
n
n
n
Ta c
Sk =

x
n2k1

x
n2k3

++

(8.42)

x
n

l tng cp s nhn hu han. Suy ra


Sk =

x. n1
nx
= 2
.
1
n 1
1 n2

Suy ra
lim Sk =

nx
,
n2 + 1

1 x
= 0.f (0),
f
k nk
nk
lim

v f (x) lin tc ti x = 0 suy ra f (x) = nx/(n2 1).


Th li, ta c kt qu ng. Vy
f (x) =

nx
, n > 1, n N , x R.
1

n2

Bi tp
Bi 8.1. Tm tt c cc hm f xc nh v lin tc trn R tha mn iu kin
f (x3) x2f (x) =

1
x, x 6= 0.
x3

Bi 8.2. Gi s f : R R lin tc v f (x + y).f (x y) = f 2 (x) vi mi x, y R.


Chng minh rng f = 0 hoc f khng c khng im.

8.5. S dng gii hn gii phng trnh hm

216

Bi 8.3. Gi s f : R R lin tc v f (x y).f (x + y) = f 2 (x).f 2(y), vi mi


x, y R. Chng minh rng f (x) = 0 hoc f (x) khng c khng im.
Bi 8.4. Tm hm lin tc tho mn
f (x + y) = f (x) + f (y) + ax2 + bxy + cy 2 , x, y R.
Bi 8.5. Tm tt c cc hm f : R R lin tc v tha mn iu kin
f (x2) + f (x) = x2 + x, x R.
Bi 8.6. Gi s a R, f (x) lin tc trn [0.1] tho mn iu kin
1. f (0) = 0
2. f (1) = 1
3. f ( x+y
2 ) = (1 a)f (x) + af (y), vi mi x, y [0, 1], x 6 slanty.
Tm cc gi tr c th ca a.

Ti liu tham kho


[1] Nguyn Vn Mu, "a thc i s v phn thc hu t", NXB Gio dc
2002.
[2] Nguyn Vn Mu,"Phng trnh hm", NXB Gio Dc, 1996.
[3] B.J.Venkatachala, "Functional Equations - A problem Solving Approach",
PRISM 2002.
[4] Cc tp ch Kvant, Ton hc v tui tr, t liu Internet.

217

You might also like